*NURSING > QUESTIONS & ANSWERS > TEST BANK for Brunner & Suddarth's Medical-Surgical Nursing Primary Concepts Of Adult Nursing. All 7 (All)

TEST BANK for Brunner & Suddarth's Medical-Surgical Nursing Primary Concepts Of Adult Nursing. All 73 Chapters 1386 Pages

Document Content and Description Below

TEST BANK for Brunner & Suddarth's Medical-Surgical Nursing Primary Concepts Of Adult Nursing. All 73 Chapters 1386 Pages Chapter 01: Health Care Delivery and Evidence-Based Nursing Practice Chapter ... 02: Community-Based Nursing Practice Chapter 03: Critical Thinking, Ethical Decision Making and the Nursing Process Chapter 04: Health Education and Promotion Chapter 05: Adult Health and Nutritional Assessment Chapter 06: Individual and Family Homeostasis, Stress, and Adaptation Chapter 07: Overview of Transcultural Nursing Chapter 08: Overview of Genetics and Genomics in Nursing Chapter 09: Chronic Illness and Disability Chapter 10: Principles and Practices of Rehabilitation Chapter 11: Health Care of the Older Adult Chapter 12: Pain Management Chapter 13: Fluid and Electrolytes: Balance and Disturbance Chapter 14: Shock and Multiple Organ Dysfunction Syndrome Chapter 15: Management of Patients with Oncologic Disorders Chapter 16: End-of-Life Care Chapter 17: Preoperative Nursing Management Chapter 18: Intraoperative Nursing Management Chapter 19: Postoperative Nursing Management Chapter 20: Assessment of Respiratory Function Chapter 21: Respiratory Care Modalities Chapter 22: Management of Patients With Upper Respiratory Tract Disorders Chapter 23: Management of Patients with Chest and Lower Respiratory Tract Disorders Chapter 24: Management of Patients With Chronic Pulmonary Disease Chapter 25: Assessment of Cardiovascular Function Chapter 26: Management of Patients With Dysrhythmias and Conduction Problems Chapter 27: Management of Patients With Coronary Vascular Disorders Chapter 28: Management of Patients With Structural, Infectious, and Inflammatory Cardiac Disorders Chapter 29: Management of Patients With Complications from Heart Disease Chapter 30: Assessment and Management of Patients With Vascular Disorders and Problems of Peripheral Circulation Chapter 31: Assessment and Management of Patients With Hypertension Chapter 32: Assessment of Hematologic Function and Treatment Modalities Chapter 33: Management of Patients With Nonmalignant Hematologic Disorders Chapter 34: Management of Patients With Hematologic Neoplasms Chapter 35: Assessment of Immune Function Chapter 36: Management of Patients With Immune Deficiency Disorders Chapter 37: Assessment and Management of Patients With Allergic Disorders Chapter 38: Assessment and Management of Patients With Rheumatic Disorders Chapter 39: Assessment of Musculoskeletal Function Chapter 40: Musculoskeletal Care Modalities Chapter 41: Management of Patients With Musculoskeletal Disorders Chapter 42: Management of Patients With Musculoskeletal Trauma Chapter 43: Assessment of Digestive and Gastrointestinal Function Chapter 44: Digestive and Gastrointestinal Treatment Modalities Chapter 45: Management of Patients with Oral and Esophageal Disorders Chapter 46: Management of Patients with Gastric and Duodenal Disorders Chapter 47: Management of Patients With Intestinal and Rectal Disorders Chapter 48: Assessment and Management of Patients with Obesity Chapter 49: Assessment and Management of Patients with Hepatic Disorders Chapter 50: Assessment and Management of Patients with Biliary Disorders Chapter 51: Assessment and Management of Patients with Diabetes Chapter 52: Assessment and Management of Patients with Endocrine Disorders Chapter 53: Assessment of Kidney and Urinary Function Chapter 54: Management of Patients with Kidney Disorders Chapter 55: Management of Patients with Urinary Disorders Chapter 56: Assessment and Management of Patients With Female Physiologic Processes Chapter 57: Management of Patients with Female Reproductive Disorders Chapter 58: Assessment and Management of Patients with Breast Disorders Chapter 59: Assessment and Management of Patients With Male Reproductive Disorders Chapter 60: Assessment of Integumentary Function Chapter 61: Managements of Patients with Dermatologic Problems Chapter 62: Managements of Patients with Burn Injury Chapter 63: Assessment and Management of Patients with Eye and Vision Disorders Chapter 64: Assessment and Management of Patients with Hearing and Balance Disorders Chapter 65: Assessment of Neurologic Function Chapter 66: Management of Patients with Neurologic Dysfunction Chapter 67: Management of Patients with Cerebrovascular Disorders Chapter 68: Management of Patients with Neurologic Trauma Chapter 69: Management of Patients with Neurologic Infections, Autoimmune Disorders, and Neuropathies 1294 Chapter 70: Management of Patients With Oncologic or Degenerative Neurologic Disorders 1312 Chapter 71: Management of Patients With Infectious Diseases Chapter 72: Emergency Nursing Chapter 73: Terrorism, Mass Casualty, and Disaster Nursing Chapter 21: Respiratory Care Modalities The nurse is caring for a patient with chronic obstructive pulmonary disease (COPD). The patient has been receiving high-flow oxygen therapy for an extended time. What symptoms should the nurse anticipate if the patient were experiencing oxygen toxicity? Bradycardia and frontal headache Dyspnea and substernal pain Peripheral cyanosis and restlessness Hypotension and tachycardia The nurse caring for a patient with an endotracheal tube recognizes several disadvantages of an endotracheal tube. What would the nurse recognize as a disadvantage of endotracheal tubes? Cognition is decreased. Daily arterial blood gases (ABGs) are necessary. Slight tracheal bleeding is anticipated. The cough reflex is depressed. What would the critical care nurse recognize as a condition that may indicate a patients need to have a tracheostomy? A patient has a respiratory rate of 10 breaths per minute. A patient requires permanent ventilation. A patient exhibits symptoms of dyspnea. A patient has respiratory acidosis. The medical nurse is creating the care plan of an adult patient requiring mechanical ventilation. What nursing action is most appropriate? Keep the patient in a low Fowlers position. Perform tracheostomy care at least once per day. Maintain continuous bedrest. Monitor cuff pressure every 8 hours. The nurse is caring for a patient who is scheduled to have a thoracotomy. When planning preoperative teaching, what information should the nurse communicate to the patient? How to milk the chest tubing How to splint the incision when coughing How to take prophylactic antibiotics correctly How to manage the need for fluid restriction A nurse is educating a patient in anticipation of a procedure that will require a water-sealed chest drainage system. What should the nurse tell the patient and the family that this drainage system is used for? Maintaining positive chest-wall pressure Monitoring pleural fluid osmolarity Providing positive intrathoracic pressure Removing excess air and fluid A patient is exhibiting signs of a pneumothorax following tracheostomy. The surgeon inserts a chest tube into the anterior chest wall. What should the nurse tell the family is the primary purpose of this chest tube? To remove air from the pleural space To drain copious sputum secretions To monitor bleeding around the lungs To assist with mechanical ventilation A patients plan of care specifies postural drainage. What action should the nurse perform when providing this noninvasive therapy? Administer the treatment with the patient in a high Fowlers or semi-Fowlers position. Perform the procedure immediately following the patients meals. Apply percussion firmly to bare skin to facilitate drainage. Assist the patient into a position that will allow gravity to move secretions. a meal. The critical care nurse is precepting a new nurse on the unit. Together they are caring for a patient who has a tracheostomy tube and is receiving mechanical ventilation. What action should the critical care nurse recommend when caring for the cuff? Deflate the cuff overnight to prevent tracheal tissue trauma. Inflate the cuff to the highest possible pressure in order to prevent aspiration. Monitor the pressure in the cuff at least every 8 hours Keep the tracheostomy tube plugged at all times. minimal occlusion volume technique. Plugging is only used when weaning the patient from tracheal support. Deflating the cuff overnight would be unsafe and inappropriate. High cuff pressure can cause tissue trauma. The acute medical nurse is preparing to wean a patient from the ventilator. Which assessment parameter is most important for the nurse to assess? Fluid intake for the last 24 hours Baseline arterial blood gas (ABG) levels Prior outcomes of weaning Electrocardiogram (ECG) results . While assessing the patient, the nurse observes constant bubbling in the water-seal chamber of the patients closed chest-drainage system. What should the nurse conclude? The system is functioning normally. The patient has a pneumothorax. The system has an air leak. The chest tube is obstructed. A patient recovering from thoracic surgery is on long-term mechanical ventilation and becomes very frustrated when he tries to communicate. What intervention should the nurse perform to assist the patient? Assure the patient that everything will be all right and that remaining calm is the best strategy. Ask a family member to interpret what the patient is trying to communicate. Ask the physician to wean the patient off the mechanical ventilator to allow the patient to speak freely. Express empathy and then encourage the patient to write, use a picture board, or spell words with an alphabet board. The physician has ordered continuous positive airway pressure (CPAP) with the delivery of a patients high-flow oxygen therapy. The patient asks the nurse what the benefit of CPAP is. What would be the nurses best response? CPAP allows a higher percentage of oxygen to be safely used. CPAP allows a lower percentage of oxygen to be used with a similar effect. CPAP allows for greater humidification of the oxygen that is administered. CPAP allows for the elimination of bacterial growth in oxygen delivery systems. The home care nurse is assessing a patient who requires home oxygen therapy. What criterion indicates that an oxygen concentrator will best meet the needs of the patient in the home environment? The patient desires a low-maintenance oxygen delivery system that delivers oxygen flow rates up to 6 L/min. The patient requires a high-flow system for use with a tracheostomy collar. The patient desires a portable oxygen delivery system that can deliver 2 L/min. The patients respiratory status requires a system that provides an FiO2 of 65%. While caring for a patient with an endotracheal tube, the nurses recognizes that suctioning is required how often? Every 2 hours when the patient is awake When adventitious breath sounds are auscultated When there is a need to prevent the patient from coughing When the nurse needs to stimulate the cough reflex B The nurse is caring for a patient who is ready to be weaned from the ventilator. In preparing to assist in the collaborative process of weaning the patient from a ventilator, the nurse is aware that the weaning of the patient will progress in what order? Removal from the ventilator, tube, and then oxygen Removal from oxygen, ventilator, and then tube Removal of the tube, oxygen, and then ventilator Removal from oxygen, tube, and then ventilator The nurse has admitted a patient who is scheduled for a thoracic resection. The nurse is providing preoperative teaching and is discussing several diagnostic studies that will be required prior to surgery. Which study will be performed to determine whether the planned resection will leave sufficient functioning lung tissue? Pulmonary function studies Exercise tolerance tests Arterial blood gas values Chest x-ray The nurse is discussing activity management with a patient who is postoperative following thoracotomy. What instructions should the nurse give to the patient regarding activity immediately following discharge? Walk 1 mile 3 to 4 times a week. Use weights daily to increase arm strength. Walk on a treadmill 30 minutes daily. Perform shoulder exercises five times daily. A patient with a severe exacerbation of COPD requires reliable and precise oxygen delivery. Which mask will the nurse expect the physician to order? Non-rebreather air mask Tracheostomy collar Venturi mask Face tent The nurse is caring for a patient who is experiencing mild shortness of breath during the immediate postoperative period, with oxygen saturation readings between 89% and 91%. What method of oxygen delivery is most appropriate for the patients needs? Non-rebreathing mask Nasal cannula Simple mask Partial-rebreathing mask A critical care nurse is caring for a client with an endotracheal tube who is on a ventilator. The nurse knows that meticulous airway management of this patient is necessary. What is the main rationale for this? Maintaining a patent airway Preventing the need for suctioning Maintaining the sterility of the patients airway Increasing the patients lung compliance A Feedback: The nurse is preparing to suction a patient with an endotracheal tube. What should be the nurses first step in the suctioning process? Explain the suctioning procedure to the patient and reposition the patient. Turn on suction source at a pressure not exceeding 120 mm Hg. Assess the patients lung sounds and SAO2 via pulse oximeter. Perform hand hygiene and don nonsterile gloves, goggles, gown, and mask. The critical care nurse and the other members of the care team are assessing the patient to see if he is ready to be weaned from the ventilator. What are the most important predictors of successful weaning that the nurse should identify? Stable vital signs and ABGs Pulse oximetry above 80% and stable vital signs Stable nutritional status and ABGs Normal orientation and level of consciousness The OR nurse is setting up a water-seal chest drainage system for a patient who has just had a thoracotomy. The nurse knows that the amount of suction in the system is determined by the water level. At what suction level should the nurse set the system? 20 cm H2O 15 cm H2O 10 cm H2O 5 cm H2O The nurse is preparing to discharge a patient after thoracotomy. The patient is going home on oxygen therapy and requires wound care. As a result, he will receive home care nursing. What should the nurse include in discharge teaching for this patient? Safe technique for self-suctioning of secretions Technique for performing postural drainage Correct and safe use of oxygen therapy equipment How to provide safe and effective tracheostomy care The nurse is performing patient education for a patient who is being discharged on mini-nebulizer treatments. What information should the nurse prioritize in the patients discharge teaching? How to count her respirations accurately How to collect serial sputum samples How to independently wean herself from treatment How to perform diaphragmatic breathing The nurse is caring for a client with an endotracheal tube who is on a ventilator. When assessing the client, the nurse knows to maintain what cuff pressure to maintain appropriate pressure on the tracheal wall? Between 10 and 15 mm Hg Between 15 and 20 mm Hg Between 20 and 25 mm Hg Between 25 and 30 mm Hg The decision has been made to discharge a ventilator-dependent patient home. The nurse is developing a teaching plan for this patient and his family. What would be most important to include in this teaching plan? Administration of inhaled corticosteroids Assessment of neurologic status Turning and coughing Signs of pulmonary infection The nurse has explained to the patient that after his thoracotomy, it will be important to adhere to a coughing schedule. The patient is concerned about being in too much pain to be able to cough. What would be an appropriate nursing intervention for this client? Teach him postural drainage. Teach him how to perform huffing. Teach him to use a mini-nebulizer. Teach him how to use a metered dose inhaler. A nurse educator is reviewing the indications for chest drainage systems with a group of medical nurses. What indications should the nurses identify? Select all that apply. Post thoracotomy Spontaneous pneumothorax Need for postural drainage Chest trauma resulting in pneumothorax Pleurisy The home care nurse is visiting a patient newly discharged home after a lobectomy. What would be most important for the home care nurse to assess? Resumption of the patients ADLs The familys willingness to care for the patient Nutritional status and fluid balance Signs and symptoms of respiratory complications A patient has been discharged home after thoracic surgery. The home care nurse performs the initial visit and finds the patient discouraged and saddened. The client states, I am recovering so slowly. I really thought I would be better by now. What nursing action should the nurse prioritize? Provide emotional support to the patient and family. Schedule a visit to the patients primary physician within 24 hours. Notify the physician that the patient needs a referral to a psychiatrist. Place a referral for a social worker to visit the patient. A patient is being admitted to the preoperative holding area for a thoracotomy. Preoperative teaching includes what? Correct use of a ventilator Correct use of incentive spirometry Correct use of a mini-nebulizer Correct technique for rhythmic breathing A patient in the ICU has had an endotracheal tube in place for 3 weeks. The physician has ordered that a tracheostomy tube be placed. The patients family wants to know why the endotracheal tube cannot be left in place. What would be the nurses best response? The physician may feel that mechanical ventilation will have to be used long-term. Long-term use of an endotracheal tube diminishes the normal breathing reflex. When an endotracheal tube is left in too long it can damage the lining of the windpipe. It is much harder to breathe through an endotracheal tube than a tracheostomy. The home care nurse is planning to begin breathing retraining exercises with a client newly admitted to the home health service. The home care nurse knows that breathing retraining is especially indicated if the patient has what diagnosis? Asthma Pneumonia Lung cancer COPD e nurse is performing nasotracheal suctioning on a medical patient and obtains copious amounts of secretions from the patients airway, even after inserting and withdrawing the catheter several times. How should the nurse proceed? Continue suctioning the patient until no more secretions are obtained. Perform chest physiotherapy rather than nasotracheal suctioning. Wait several minutes and then repeat suctioning. Perform postural drainage and then repeat suctioning. A nurse has performed tracheal suctioning on a patient who experienced increasing dyspnea prior to a procedure. When applying the nursing process, how can the nurse best evaluate the outcomes of this intervention? Determine whether the patient can now perform forced expiratory technique (FET). Percuss the patients lungs and thorax. Measure the patients oxygen saturation. Have the patient perform incentive spirometry. Postural drainage has been ordered for a patient who is having difficulty mobilizing her bronchial secretions. Before repositioning the patient and beginning treatment, the nurse should perform what health assessment? Chest auscultation Pulmonary function testing Chest percussion Thoracic palpation A nurse is teaching a patient how to perform flow type incentive spirometry prior to his scheduled thoracic surgery. What instruction should the nurse provide to the patient? Hold the spirometer at your lips and breathe in and out like you normally would. When youre ready, blow hard into the spirometer for as long as you can. Take a deep breath and then blow short, forceful breaths into the spirometer. Breathe in deeply through the spirometer, hold your breath briefly, and then exhale. The nurse is assessing a patient who has a chest tube in place for the treatment of a pneumothorax. The nurse observes that the water level in the water seal rises and falls in rhythm with the patients respirations. How should the nurse best respond to this assessment finding? Gently reinsert the chest tube 1 to 2 cm and observe if the water level stabilizes. Inform the physician promptly that there is in imminent leak in the drainage system. Encourage the patient to do deep breathing and coughing exercises. Document that the chest drainage system is operating as it is intended. Chapter 22: Management of Patients With Upper Respiratory Tract Disorders The nurse is providing patient teaching to a young mother who has brought her 3-month-old infant to the clinic for a well-baby checkup. What action should the nurse recommend to the woman to prevent the transmission of organisms to her infant during the cold season? Take preventative antibiotics, as ordered. Gargle with warm salt water regularly. Dress herself and her infant warmly. Wash her hands frequently. A patient visiting the clinic is diagnosed with acute sinusitis. To promote sinus drainage, the nurse should instruct the patient to perform which of the following? Apply a cold pack to the affected area. Apply a mustard poultice to the forehead. Perform postural drainage. Increase fluid intake. The nurse is creating a plan of car for a patient diagnosed with acute laryngitis. What intervention should be included in the patients plan of care? Place warm cloths on the patients throat, as needed. Have the patient inhale warm steam three times daily. Encourage the patient to limit speech whenever possible. Limit the patients fluid intake to 1.5 L/day. . A patient comes to the ED and is admitted with epistaxis. Pressure has been applied to the patients midline septum for 10 minutes, but the bleeding continues. The nurse should anticipate using what treatment to control the bleeding? Irrigation with a hypertonic solution Nasopharyngeal suction Normal saline application Silver nitrate application The nurse is planning the care of a patient who is scheduled for a laryngectomy. The nurse should assign the highest priority to which postoperative nursing diagnosis? Anxiety related to diagnosis of cancer Altered nutrition related to swallowing difficulties Ineffective airway clearance related to airway alterations Impaired verbal communication related to removal of the larynx The home care nurse is assessing the home environment of a patient who will be discharged from the hospital shortly after his laryngectomy. The nurse should inform the patient that he may need to arrange for the installation of which system in his home? A humidification system An air conditioning system A water purification system A radiant heating system The nurse is caring for a patient whose recent unexplained weight loss and history of smoking have prompted diagnostic testing for cancer. What symptom is most closely associated with the early stages of laryngeal cancer? Hoarseness Dyspnea Dysphagia Frequent nosebleeds The nurse is caring for a patient who needs education on his medication therapy for allergic rhinitis. The patient is to take cromolyn (Nasalcrom) daily. In providing education for this patient, how should the nurse describe the action of the medication? It inhibits the release of histamine and other chemicals. It inhibits the action of proton pumps. It inhibits the action of the sodium-potassium pump in the nasal epithelium. It causes bronchodilation and relaxes smooth muscle in the bronchi. The campus nurse at a university is assessing a 21-year-old student who presents with a severe nosebleed. The site of bleeding appears to be the anterior portion of the nasal septum. The nurse instructs the student to tilt her head forward and the nurse applies pressure to the nose, but the students nose continues to bleed. Which intervention should the nurse next implement? Apply ice to the bridge of her nose Lay the patient down on a cot Arrange for transfer to the local ED Insert a tampon in the affected nare The ED nurse is assessing a young gymnast who fell from a balance beam. The gymnast presents with a clear fluid leaking from her nose. What should the ED nurse suspect? Fracture of the cribriform plate Rupture of an ethmoid sinus Abrasion of the soft tissue Fracture of the nasal septum A 42-year-old patient is admitted to the ED after an assault. The patient received blunt trauma to the face and has a suspected nasal fracture. Which of the following interventions should the nurse perform? Administer nasal spray and apply an occlusive dressing to the patients face. Position the patients head in a dependent position. Irrigate the patients nose with warm tap water. Apply ice and keep the patients head elevated. The occupational health nurse is obtaining a patient history during a pre-employment physical. During the history, the patient states that he has hereditary angioedema. The nurse should identify what implication of this health condition? It will result in increased loss of work days. It may cause episodes of weakness due to reduced cardiac output. It can cause life-threatening airway obstruction. It is unlikely to interfere with the individuals health. The nurse is conducting a presurgical interview for a patient with laryngeal cancer. The patient states that he drinks approximately six to eight shots of vodka per day. It is imperative that the nurse inform the surgical team so the patient can be assessed for what? Increased risk for infection Delirium tremens Depression Nonadherence to postoperative care The nurse is explaining the safe and effective administration of nasal spray to a patient with seasonal allergies. What information is most important to include in this teaching? Finish the bottle of nasal spray to clear the infection effectively. Nasal spray can only be shared between immediate family members. Nasal spray should be administered in a prone position. Overuse of nasal spray may cause rebound congestion. As a clinic nurse, you are caring for a patient who has been prescribed an antibiotic for tonsillitis and has been instructed to take the antibiotic for 10 days. When you do a follow-up call with this patient, you are informed that the patient is feeling better and is stopping the medication after taking it for 4 days. What information should you provide to this patient? Keep the remaining tablets for an infection at a later time. Discontinue the medications if the fever is gone. Dispose of the remaining medication in a biohazard receptacle. Finish all the antibiotics to eliminate the organism completely. A nurse practitioner has provided care for three different patients with chronic pharyngitis over the past several months. Which patients are at greatest risk for developing chronic pharyngitis? Patients who are habitual users of alcohol and tobacco Patients who are habitual users of caffeine and other stimulants Patients who eat a diet high in spicy foods Patients who have gastrointestinal reflux disease (GERD) The perioperative nurse has admitted a patient who has just underwent a tonsillectomy. The nurses postoperative assessment should prioritize which of the following potential complications of this surgery? Difficulty ambulating Hemorrhage Infrequent swallowing Bradycardia A 45-year-old obese man arrives in a clinic with complaints of daytime sleepiness, difficulty going to sleep at night, and snoring. The nurse should recognize the manifestations of what health problem? Adenoiditis Chronic tonsillitis Obstructive sleep apnea Laryngeal cancer cancer. The nurse is caring for a patient in the ED for epistaxis. What information should the nurse include in patient discharge teaching as a way to prevent epistaxis? Keep nasal passages clear. Use decongestants regularly. Humidify the indoor environment. Use a tissue when blowing the nose. The nurse is caring for a patient who is postoperative day 2 following a total laryngectomy for supraglottic cancer. The nurse should prioritize what assessment? Assessment of body image Assessment of jugular venous pressure Assessment of carotid pulse Assessment of swallowing ability The nurse is performing the health interview of a patient with chronic rhinosinusitis who experiences frequent nose bleeds. The nurse asks the patient about her current medication regimen. Which medication would put the patient at a higher risk for recurrent epistaxis? Afrin Beconase Sinustop Pro Singulair The nurse is performing an assessment on a patient who has been diagnosed with cancer of the larynx. Part of the nurses assessment addresses the patients general state of nutrition. Which laboratory values would be assessed when determining the nutritional status of the patient? Select all that apply. White blood cell count Protein level Albumin level Platelet count Glucose level The nurse is teaching a patient with allergic rhinitis about the safe and effective use of his medications. What would be the most essential information to give this patient about preventing possible drug interactions? Prescription medications can be safely supplemented with OTC medications. Use only one pharmacy so the pharmacist can check drug interactions. Read drug labels carefully before taking OTC medications. Consult the Internet before selecting an OTC medication. interactions is important. Not all OTC medications are safe additions to prescription medication regimens. The nurse is caring for a patient who has just been diagnosed with chronic rhinosinusitis. While being admitted to the clinic, the patient asks, Will this chronic infection hurt my new kidney? What should the nurse know about chronic rhinosinusitis in patients who have had a transplant? The patient will have exaggerated symptoms of rhinosinusitis due to immunosuppression. Taking immunosuppressive drugs can contribute to chronic rhinosinusitis. Chronic rhinosinusitis can damage the transplanted organ. Immunosuppressive drugs can cause organ rejection. The nurse is caring for a patient with a severe nosebleed. The physician inserts a nasal sponge and tells the patient it may have to remain in place up to 6 days before it is removed. The nurse should identify that this patient is at increased risk for what? Viral sinusitis Toxic shock syndrome Pharyngitis Adenoiditis A nursing student is discussing a patient with viral pharyngitis with the preceptor at the walk-in clinic. What should the preceptor tell the student about nursing care for patients with viral pharyngitis? Teaching focuses on safe and effective use of antibiotics. The patient should be preliminarily screened for surgery. Symptom management is the main focus of medical and nursing care. The focus of care is resting the voice to prevent chronic hoarseness. The nurse is providing patient teaching to a patient diagnosed with acute rhinosinusitis. For what possible complication should the nurse teach the patient to seek immediate follow-up? Periorbital edema Headache unrelieved by OTC medications Clear drainage from nose Blood-tinged mucus when blowing the nose A A patient states that her family has had several colds during this winter and spring despite their commitment to handwashing. The high communicability of the common cold is attributable to what factor? Cold viruses are increasingly resistant to common antibiotics. The virus is shed for 2 days prior to the emergence of symptoms. A genetic predisposition to viral rhinitis has recently been identified. Overuse of OTC cold remedies creates a rebound susceptibility to future colds. It is cold season and the school nurse been asked to provide an educational event for the parent teacher organization of the local elementary school. What should the nurse include in teaching about the treatment of pharyngitis? Pharyngitis is more common in children whose immunizations are not up to date. There are no effective, evidence-based treatments for pharyngitis. Use of warm saline gargles or throat irrigations can relieve symptoms. Heat may increase the spasms in pharyngeal muscles. The nurse is doing discharge teaching in the ED with a patient who had a nosebleed. What should the nurse include in the discharge teaching of this patient? Avoid blowing the nose for the next 45 minutes. In case of recurrence, apply direct pressure for 15 minutes. Do not take aspirin for the next 2 weeks. Seek immediate medical attention if the nosebleed recurs. The nurse recognizes that aspiration is a potential complication of a laryngectomy. How should the nurse best manage this risk? Facilitate total parenteral nutrition (TPN). Keep a complete suction setup at the bedside. Feed the patient several small meals daily. Refer the patient for occupational therapy. A patient has had a nasogastric tube in place for 6 days due to the development of paralytic ileus after Sinus infections Esophageal strictures Pharyngitis Laryngitis A mother calls the clinic asking for a prescription for Amoxicillin for her 2-year-old son who has what the nurse suspects to be viral rhinitis. What should the nurse explain to this mother? I will relay your request promptly to the doctor, but I suspect that she wont get back to you if its a cold. Ill certainly inform the doctor, but if it is a cold, antibiotics wont be used because they do not affect the virus. Ill phone in the prescription for you since it can be prescribed by the pharmacist. Amoxicillin is not likely the best antibiotic, but Ill call in the right prescription for you. The nurse is providing care for a patient who has just been admitted to the postsurgical unit following a laryngectomy. What assessment should the nurse prioritize? The patients swallowing ability The patients airway patency The patients carotid pulses Signs and symptoms of infection The nurse has noted the emergence of a significant amount of fresh blood at the drain site of a patient who is postoperative day 1 following total laryngectomy. How should the nurse respond to this development? Remove the patients drain and apply pressure with a sterile gauze. Assess the patient, reposition the patient supine, and apply wall suction to the drain. Rapidly assess the patient and notify the surgeon about the patients bleeding. Administer a STAT dose of vitamin K to aid coagulation. The nurse is creating a care plan for a patient who is status post-total laryngectomy. Much of the plan consists of a long-term postoperative communication plan for alaryngeal communication. What form of alaryngeal communication will likely be chosen? Esophageal speech Electric larynx Tracheoesophageal puncture American sign language (ASL) A patient is being treated for bacterial pharyngitis. Which of the following should the nurse recommend when promoting the patients nutrition during treatment? A 1.5 L/day fluid restriction A high-potassium, low-sodium diet A liquid or soft diet A high-protein diet A patient has just been diagnosed with squamous cell carcinoma of the neck. While the nurse is doing health education, the patient asks, Does this kind of cancer tend to spread to other parts of the body? What is the nurses best response? In many cases, this type of cancer spreads to other parts of the body. This cancer usually does not spread to distant sites in the body. You will have to speak to your oncologist about that. Squamous cell carcinoma is nothing to be concerned about, so try to focus on your health. The nurse is performing preoperative teaching with a patient who has cancer of the larynx. After completing patient teaching, what would be most important for the nurse to do? Give the patient his or her cell phone number. Refer the patient to a social worker or psychologist. Provide the patient with audiovisual materials about the surgery. Reassure the patient and family that everything will be alright. A patients total laryngectomy has created a need for alaryngeal speech which will be achieved through the use of tracheoesophageal puncture. What action should the nurse describe to the patient when teaching him about this process? Training on how to perform controlled belching Use of an electronically enhanced artificial pharynx Insertion of a specialized nasogastric tube Fitting for a voice prosthesis Chapter 23: Management of Patients with Chest and Lower Respiratory Tract Disorders A perioperative nurse is caring for a postoperative patient. The patient has a shallow respiratory pattern and is reluctant to cough or to begin mobilizing. The nurse should address the patients increased risk for what complication? Acute respiratory distress syndrome (ARDS) Atelectasis Aspiration Pulmonary embolism A critical-care nurse is caring for a patient diagnosed with pneumonia as a surgical complication. The nurses assessment reveals that the patient has an increased work of breathing due to copious tracheobronchial secretions. What should the nurse encourage the patient to do? Increase oral fluids unless contraindicated. Call the nurse for oral suctioning, as needed. Lie in a low Fowlers or supine position. Increase activity. The public health nurse is administering Mantoux tests to children who are being registered for kindergarten in the community. How should the nurse administer this test? Administer intradermal injections into the childrens inner forearms. Administer intramuscular injections into each childs vastus lateralis. Administer a subcutaneous injection into each childs umbilical area. Administer a subcutaneous injection at a 45-degree angle into each childs deltoid. The nurse is caring for a patient who has been in a motor vehicle accident and the care team suspects that the patient has developed pleurisy. Which of the nurses assessment findings would best corroborate this diagnosis? The patient is experiencing painless hemoptysis. The patients arterial blood gases (ABGs) are normal, but he demonstrates increased work of breathing. The patients oxygen saturation level is below 88%, but he denies shortness of breath. The patients pain intensifies when he coughs or takes a deep breath. The nurse caring for a patient recently diagnosed with lung disease encourages the patient not to smoke. What is the primary rationale behind this nursing action? Smoking decreases the amount of mucus production. Smoke particles compete for binding sites on hemoglobin. Smoking causes atrophy of the alveoli. Smoking damages the ciliary cleansing mechanism. A patient has been brought to the ED by the paramedics. The patient is suspected of having ARDS. What intervention should the nurse first anticipate? Preparing to assist with intubating the patient Setting up oxygen at 5 L/minute by nasal cannula Performing deep suctioning Setting up a nebulizer to administer corticosteroids The nurse is caring for a patient who is scheduled for a lobectomy for a diagnosis of lung cancer. While assisting with a subclavian vein central line insertion, the nurse notes the clients oxygen saturation rapidly dropping. The patient complains of shortness of breath and becomes tachypneic. The nurse suspects a pneumothorax has developed. Further assessment findings supporting the presence of a pneumothorax include what? Diminished or absent breath sounds on the affected side Paradoxical chest wall movement with respirations Sudden loss of consciousness Muffled heart sounds The nurse is providing discharge teaching for a patient who developed a pulmonary embolism after total knee surgery. The patient has been converted from heparin to sodium warfarin (Coumadin) anticoagulant therapy. What should the nurse teach the client? Coumadin will continue to break up the clot over a period of weeks Coumadin must be taken concurrent with ASA to achieve anticoagulation. Anticoagulant therapy usually lasts between 3 and 6 months. He should take a vitamin supplement containing vitamin K A new employee asks the occupational health nurse about measures to prevent inhalation exposure of the substances. Which statement by the nurse will decrease the patients exposure risk to toxic substances? Position a fan blowing on the toxic substances to prevent the substance from becoming stagnant in the air. Wear protective attire and devices when working with a toxic substance. Make sure that you keep your immunizations up to date to prevent respiratory diseases resulting from toxins. Always wear a disposable paper face mask when you are working with inhalable toxins. An x-ray of a trauma patient reveals rib fractures and the patient is diagnosed with a small flail chest injury. Which intervention should the nurse include in the patients plan of care? Suction the patients airway secretions. Immobilize the ribs with an abdominal binder. Prepare the patient for surgery. Immediately sedate and intubate the patient. for severe flail chest injuries, and surgery is required only in rare circumstances to stabilize the flail segment. The nurse is caring for a patient who is receiving oxygen therapy for pneumonia. How should the nurse best assess whether the patient is hypoxemic? Assess the patients level of consciousness (LOC). Assess the patients extremities for signs of cyanosis. Assess the patients oxygen saturation level. Review the patients hemoglobin, hematocrit, and red blood cell levels. related to oxygenation. Hemoglobin, hematocrit, and red blood cell levels do not directly reflect current oxygenation status. An adult patient has tested positive for tuberculosis (TB). While providing patient teaching, what information should the nurse prioritize? The importance of adhering closely to the prescribed medication regimen The fact that the disease is a lifelong, chronic condition that will affect ADLs The fact that TB is self-limiting, but can take up to 2 years to resolve The nurse is assessing an adult patient following a motor vehicle accident. The nurse observes that the patient has an increased use of accessory muscles and is complaining of chest pain and shortness of breath. The nurse should recognize the possibility of what condition? Pneumothorax Anxiety Acute bronchitis Aspiration The nurse at a long-term care facility is assessing each of the residents. Which resident most likely faces the greatest risk for aspiration? A resident who suffered a severe stroke several weeks ago A resident with mid-stage Alzheimers disease A 92-year-old resident who needs extensive help with ADLs A resident with severe and deforming rheumatoid arthritis The nurse is caring for a patient suspected of having ARDS. What is the most likely diagnostic test ordered in the early stages of this disease to differentiate the patients symptoms from those of a cardiac etiology? Carboxyhemoglobin level Brain natriuretic peptide (BNP) level C-reactive protein (CRP) level Complete blood count The nurse is caring for a patient at risk for atelectasis. The nurse implements a first-line measure to prevent atelectasis development in the patient. What is an example of a first-line measure to minimize atelectasis? Incentive spirometry Intermittent positive-pressure breathing (IPPB) Positive end-expiratory pressure (PEEP) Bronchoscopy While planning a patients care, the nurse identifies nursing actions to minimize the patients pleuritic pain. Which intervention should the nurse include in the plan of care? Avoid actions that will cause the patient to breathe deeply. Ambulate the patient at least three times daily. Arrange for a soft-textured diet and increased fluid intake. Encourage the patient to speak as little as possible The perioperative nurse is writing a care plan for a patient who has returned from surgery 2 hours prior. Which measure should the nurse implement to most decrease the patients risk of developing pulmonary emboli (PE)? Early ambulation Increased dietary intake of protein Maintaining the patient in a supine position Administering aspirin with The school nurse is presenting a class on smoking cessation at the local high school. A participant in the class asks the nurse about the risk of lung cancer in those who smoke. What response related to risk for lung cancer in smokers is most accurate? The younger you are when you start smoking, the higher your risk of lung cancer. The risk for lung cancer never decreases once you have smoked, which is why smokers need annual chest x-rays. The risk for lung cancer is determined mostly by what type of cigarettes you smoke. The risk for lung cancer depends primarily on the other risk factors for cancer that you have. The nurse is assessing a patient who has a 35 pack-year history of cigarette smoking. In light of this known risk factor for lung cancer, what statement should prompt the nurse to refer the patient for further assessment? Lately, I have this cough that just never seems to go away. I find that I dont have nearly the stamina that I used to. I seem to get nearly every cold and flu that goes around my workplace. I never used to have any allergies, but now I think Im developing allergies to dust and pet hair. A client presents to the walk-in clinic complaining of a dry, irritating cough and production of a minute amount of mucus-like sputum. The patient complains of soreness in her chest in the sternal area. The nurse should suspect that the primary care provider will assess the patient for what health problem? Pleural effusion Pulmonary embolism Tracheobronchitis Tuberculosis A hospital has been the site of an increased incidence of hospital-acquired pneumonia (HAP). What is an important measure for the prevention of HAP? Administration of prophylactic antibiotics Administration of pneumococcal vaccine to vulnerable individuals Obtaining culture and sensitivity swabs from all newly admitted patients Administration of antiretroviral medications to patients over age 65 B When assessing for substances that are known to harm workers lungs, the occupational health nurse should assess their potential exposure to which of the following? Organic acids Propane Asbestos Gypsum A patient presents to the ED stating she was in a boating accident about 3 hours ago. Now the patient has complaints of headache, fatigue, and the feeling that he just cant breathe enough. The nurse notes that the patient is restless and tachycardic with an elevated blood pressure. This patient may be in the early stages of what respiratory problem? Pneumoconiosis Pleural effusion Acute respiratory failure Pneumonia hypoxemia progresses, more obvious signs may be present, including confusion, lethargy, tachycardia, tachypnea, central cyanosis, diaphoresis, and, finally, respiratory arrest. Pneumonia is infectious and would not result from trauma. Pneumoconiosis results from exposure to occupational toxins. A pleural effusion does not cause this constellation of symptoms. The nurse is caring for a 46-year-old patient recently diagnosed with the early stages of lung cancer. The nurse is aware that the preferred method of treating patients with nonsmall cell tumors is what? Chemotherapy Radiation Surgical resection Bronchoscopic opening of the airway C A patient is receiving thrombolytic therapy for the treatment of pulmonary emboli. What is the best way for the nurse to assess the patients oxygenation status at the bedside? Obtain serial ABG samples. Monitor pulse oximetry readings. Test pulmonary function. Monitor incentive spirometry volumes. The nurse is caring for an 82-year-old patient with a diagnosis of tracheobronchitis. The patient begins complaining of right-sided chest pain that gets worse when he coughs or breathes deeply. Vital signs are within normal limits. What would you suspect this patient is experiencing? Traumatic pneumothorax Empyema Pleuritic pain Myocardial infarction . A patient with thoracic trauma is admitted to the ICU. The nurse notes the patients chest and neck are swollen and there is a crackling sensation when palpated. The nurse consequently identifies the presence of subcutaneous emphysema. If this condition becomes severe and threatens airway patency, what intervention is indicated? A chest tube A tracheostomy An endotracheal tube A feeding tube The nurse is caring for a patient in the ICU admitted with ARDS after exposure to toxic fumes from a hazardous spill at work. The patient has become hypotensive. What is the cause of this complication to the ARDS treatment? Pulmonary hypotension due to decreased cardiac output Severe and progressive pulmonary hypertension Hypovolemia secondary to leakage of fluid into the interstitial spaces Increased cardiac output from high levels of PEEP therapy The home care nurse is monitoring a patient discharged home after resolution of a pulmonary embolus. For what potential complication would the home care nurse be most closely monitoring this patient? Signs and symptoms of pulmonary infection Swallowing ability and signs of aspiration Activity level and role performance Residual effects of compromised oxygenation The occupational health nurse is assessing new employees at a company. What would be important to assess in employees with a potential occupational respiratory exposure to a toxin? Select all that apply. Time frame of exposure Type of respiratory protection used Immunization status Breath sounds Intensity of exposure A 54-year-old man has just been diagnosed with small cell lung cancer. The patient asks the nurse why the doctor is not offering surgery as a treatment for his cancer. What fact about lung cancer treatment should inform the nurses response? The cells in small cell cancer of the lung are not large enough to visualize in surgery. Small cell lung cancer is self-limiting in many patients and surgery should be delayed. Patients with small cell lung cancer are not normally stable enough to survive surgery. Small cell cancer of the lung grows rapidly and metastasizes early and extensively. A patient who involved in a workplace accident suffered a penetrating wound of the chest that led to acute respiratory failure. What goal of treatment should the care team prioritize when planning this patients care? Facilitation of long-term intubation Restoration of adequate gas exchange Attainment of effective coping Self-management of oxygen therapy A patient is brought to the ED by ambulance after a motor vehicle accident in which the patient received blunt trauma to the chest. The patient is in acute respiratory failure, is intubated, and is transferred to the ICU. What parameters of care should the nurse monitor most closely? Select all that apply. Coping Level of consciousness Oral intake Arterial blood gases Vital signs A gerontologic nurse is teaching a group of medical nurses about the high incidence and mortality of pneumonia in older adults. What is a contributing factor to this that the nurse should describe? Older adults have less compliant lung tissue than younger adults. Older adults are not normally candidates for pneumococcal vaccination. Older adults often lack the classic signs and symptoms of pneumonia. Older adults often cannot tolerate the most common antibiotics used to treat pneumonia. A patient has just been diagnosed with lung cancer. After the physician discusses treatment options and leaves the room, the patient asks the nurse how the treatment is decided upon. What would be the nurses best response? The type of treatment depends on the patients age and health status. The type of treatment depends on what the patient wants when given the options. The type of treatment depends on the cell type of the cancer, the stage of the cancer, and the patients health status. The type of treatment depends on the discussion between the patient and the physician of which treatment is best. A patient in the ICU is status post embolectomy after a pulmonary embolus. What assessment parameter does the nurse monitor most closely on a patient who is postoperative following an embolectomy? Pupillary response Pressure in the vena cava White blood cell differential Pulmonary arterial pressure A firefighter was trapped in a fire and is admitted to the ICU for smoke inhalation. After 12 hours, the firefighter is exhibiting signs of ARDS and is intubated. What other supportive measures are initiated in a patient with ARDS? Psychological counseling Nutritional support High-protein oral diet Occupational therapy The nurse is reviewing the electronic health record of a patient with an empyema. What health problem in the patients history is most likely to have caused the empyema? Smoking Asbestosis Pneumonia Lung cancer An 87-year-old patient has been hospitalized with pneumonia. Which nursing action would be a priority in this patients plan of care? Nasogastric intubation Administration of probiotic supplements Bedrest Cautious hydration Chapter 24: Management of Patients With Chronic Pulmonary Disease A clinic nurse is caring for a patient who has just been diagnosed with chronic obstructive pulmonary disease (COPD). The patient asks the nurse what he could have done to minimize the risk of contracting this disease. What would be the nurses best answer? The most important risk factor for COPD is exposure to occupational toxins. The most important risk factor for COPD is inadequate exercise. The most important risk factor for COPD is exposure to dust and pollen. The most important risk factor for COPD is cigarette smoking. A nurse is creating a health promotion intervention focused on chronic obstructive pulmonary disease (COPD). What should the nurse identify as a complication of COPD? Lung cancer Cystic fibrosis Respiratory failure Hemothorax A nurse is caring for a young adult patient whose medical history includes an alpha1-antitrypsin deficiency. This deficiency predisposes the patient to what health problem? Pulmonary edema Lobular emphysema Cystic fibrosis (CF) Empyema The nurse is assessing a patient whose respiratory disease in characterized by chronic hyperinflation of the lungs. What would the nurse most likely assess in this patient? Signs of oxygen toxicity Chronic chest pain A barrel chest Long, thin fingers A patient with emphysema is experiencing shortness of breath. To relieve this patients symptoms, the nurse should assist her into what position? Sitting upright, leaning forward slightly Low Fowlers, with the neck slightly hyperextended Prone Trendelenburg A Feedback: The typical posture of a person with COPD is to lean forward and use the accessory muscles of respiration to breathe. Low Fowlers positioning would be less likely to aid oxygenation. Prone or Trendelenburg positioning would exacerbate shortness of breath. A nurse is evaluating the diagnostic study data of a patient with suspected cystic fibrosis (CF). Which of the following test results is associated with a diagnosis of cystic fibrosis? Elevated sweat chloride concentration Presence of protein in the urine Positive phenylketonuria Malignancy on lung biopsy A Feedback: Gene mutations affect transport of chloride ions, leading to CF, which is characterized by thick, viscous secretions in the lungs, pancreas, liver, intestine, and reproductive tract as well as increased salt content in sweat gland secretions. Proteinuria, positive phenylketonuria, and malignancy are not diagnostic for CF. A school nurse is caring for a 10-year-old girl who is having an asthma attack. What is the preferred intervention to alleviate this clients airflow obstruction? Administer corticosteroids by metered dose inhaler Administer inhaled anticholinergics Administer an inhaled beta-adrenergic agonist Utilize a peak flow monitoring device C Feedback: Asthma exacerbations are best managed by early treatment and education of the patient. Quick-acting beta-adrenergic medications are the first used for prompt relief of airflow obstruction. Systemic corticosteroids may be necessary to decrease airway inflammation in patients who fail to respond to inhaled beta-adrenergic medication. A peak flow device will not resolve short-term shortness of breath. A student nurse is developing a teaching plan for an adult patient with asthma. Which teaching point should have the highest priority in the plan of care that the student is developing? Gradually increase levels of physical exertion. Change filters on heaters and air conditioners frequently. Take prescribed medications as scheduled. Avoid goose-down pillows. C Feedback: Although all of the measures are appropriate for a client with asthma, taking prescribed medications on time is the most important measure in preventing asthma attacks. A student nurse is preparing to care for a patient with bronchiectasis. The student nurse should recognize that this patient is likely to experience respiratory difficulties related to what pathophysiologic process? Intermittent episodes of acute bronchospasm Alveolar distention and impaired diffusion Dilation of bronchi and bronchioles Excessive gas exchange in the bronchioles C Feedback: Bronchiectasis is a chronic, irreversible dilation of the bronchi and bronchioles that results from destruction of muscles and elastic connective tissue. It is not characterized by acute bronchospasm, alveolar distention, or excessive gas exchange. A nurse is caring for a 6-year-old patient with cystic fibrosis. In order to enhance the childs nutritional status, what intervention should most likely be included in the plan of care? Pancreatic enzyme supplementation with meals Provision of five to six small meals per day rather than three larger meals Total parenteral nutrition (TPN) Magnesium, thiamine, and iron supplementation A Feedback: Nearly 90% of patients with CF have pancreatic exocrine insufficiency and require oral pancreatic enzyme supplementation with meals. Frequent, small meals or TPN are not normally indicated. Vitamin supplements are required, but specific replacement of magnesium, thiamine, and iron is not typical. A patient arrives in the emergency department with an attack of acute bronchiectasis. Chest auscultation reveals the presence of copious secretions. What intervention should the nurse prioritize in this patients care? Oral administration of diuretics Intravenous fluids to reduce the viscosity of secretions Postural chest drainage Pulmonary function testing C Feedback: Postural drainage is part of all treatment plans for bronchiectasis, because draining of the bronchiectatic areas by gravity reduces the amount of secretions and the degree of infection. Diuretics and IV fluids will not aid in the mobilization of secretions. Lung function testing may be indicated, but this assessment will not relieve the patients symptoms. A nurse is completing a focused respiratory assessment of a child with asthma. What assessment finding is most closely associated with the characteristic signs and symptoms of asthma? Shallow respirations Increased anterior-posterior (A-P) diameter Bilateral wheezes Bradypnea C Feedback: The three most common symptoms of asthma are cough, dyspnea, and wheezing. There may be generalized wheezing (the sound of airflow through narrowed airways), first on expiration and then, possibly, during inspiration as well. Respirations are not usually slow and the childs A-P diameter does not normally change. A nurse is developing the teaching portion of a care plan for a patient with COPD. What would be the most important component for the nurse to emphasize? Smoking up to one-half of a pack of cigarettes weekly is allowable. Chronic inhalation of indoor toxins can cause lung damage. Minor respiratory infections are considered to be self-limited and are not treated. Activities of daily living (ADLs) should be clustered in the early morning hours. B Feedback: Environmental risk factors for COPD include prolonged and intense exposure to occupational dusts and chemicals, indoor air pollution, and outdoor air pollution. Smoking cessation should be taught to all patients who are currently smoking. Minor respiratory infections that are of no consequence to the person with normal lungs can produce fatal disturbances in the lungs of the person with emphysema. ADLs should be paced throughout the day to permit patients to perform these without excessive distress. A nursing is planning the care of a patient with emphysema who will soon be discharged. What teaching should the nurse prioritize in the plan of care? Taking prophylactic antibiotics as ordered Adhering to the treatment regimen in order to cure the disease Avoiding airplanes, buses, and other crowded public places Setting realistic short-term and long-range goals D Feedback: A major area of teaching involves setting and accepting realistic short-term and long-range goals. Emphysema is not considered curable and antibiotics are not used on a preventative basis. The patient does not normally need to avoid public places. A nurse is documenting the results of assessment of a patient with bronchiectasis. What would the nurse most likely include in documentation? Sudden onset of pleuritic chest pain Wheezes on auscultation Increased anterior-posterior (A-P) diameter Clubbing of the fingers D Feedback: Characteristic symptoms of bronchiectasis include chronic cough and production of purulent sputum in copious amounts. Clubbing of the fingers also is common because of respiratory insufficiency. Sudden pleuritic chest pain is a common manifestation of a pulmonary embolism. Wheezes on auscultation are common in patients with asthma. An increased A-P diameter is noted in patients with COPD. A patient is having pulmonary-function studies performed. The patient performs a spirometry test, revealing an FEV1/FVC ratio of 60%. How should the nurse interpret this assessment finding? Strong exercise tolerance Exhalation volume is normal Respiratory infection Obstructive lung disease D Feedback: Spirometry is used to evaluate airflow obstruction, which is determined by the ratio of forced expiration volume in 1 second to forced vital capacity. Obstructive lung disease is apparent when an FEV1/FVC ratio is less than 70%. A nurse has been asked to give a workshop on COPD for a local community group. The nurse emphasizes the importance of smoking cessation because smoking has what pathophysiologic effect? Increases the amount of mucus production Destabilizes hemoglobin Shrinks the alveoli in the lungs Collapses the alveoli in the lungs A Feedback: Smoking irritates the goblet cells and mucous glands, causing an increased accumulation of mucus, which, in turn, produces more irritation, infection, and damage to the lung. A pediatric nurse practitioner is caring for a child who has just been diagnosed with asthma. The nurse has provided the parents with information that includes potential causative agents for an asthmatic reaction. What potential causative agent should the nurse describe? Pets Lack of sleep Psychosocial stress Bacteria A Feedback: Common causative agents that may trigger an asthma attack are as follows: dust, dust mites, pets, soap, certain foods, molds, and pollens. Lack of sleep, stress, and bacteria are not common triggers for asthma attacks. A nurse is providing discharge teaching for a client with COPD. When teaching the client about breathing exercises, what should the nurse include in the teaching? Lie supine to facilitate air entry Avoid pursed lip breathing Use diaphragmatic breathing Use chest breathing C Feedback: Inspiratory muscle training and breathing retraining may help improve breathing patterns in patients with COPD. Training in diaphragmatic breathing reduces the respiratory rate, increases alveolar ventilation, and, sometimes, helps expel as much air as possible during expiration. Pursed-lip breathing helps slow expiration, prevents collapse of small airways, and controls the rate and depth of respiration. Diaphragmatic breathing, not chest breathing, increases lung expansion. Supine positioning does not aid breathing. A nurse is caring for a patient who has been admitted with an exacerbation of chronic bronchiectasis. The nurse should expect to assess the patient for which of the following clinical manifestations? Copious sputum production Pain on inspiration Pigeon chest Dry cough A Feedback: Clinical manifestations of bronchiectasis include hemoptysis, chronic cough, copious purulent sputum, and clubbing of the fingers. Because of the copious production of sputum, the cough is rarely dry. A pigeon chest is not associated with the disease and patients do not normally experience pain on inspiration. A nurse is reviewing the pathophysiology of cystic fibrosis (CF) in anticipation of a new admission. The nurse should identify what characteristic aspects of CF? Alveolar mucus plugging, infection, and eventual bronchiectasis Bronchial mucus plugging, inflammation, and eventual bronchiectasis Atelectasis, infection, and eventual COPD Bronchial mucus plugging, infection, and eventual COPD B Feedback: The hallmark pathology of CF is bronchial mucus plugging, inflammation, and eventual bronchiectasis. Commonly, the bronchiectasis begins in the upper lobes and progresses to involve all lobes. Infection, atelectasis, and COPD are not hallmark pathologies of CF. An older adult patient has been diagnosed with COPD. What characteristic of the patients current health status would preclude the safe and effective use of a metered-dose inhaler (MDI)? The patient has not yet quit smoking. The patient has severe arthritis in her hands. The patient requires both corticosteroids and beta2-agonists. The patient has cataracts. B Feedback: Safe and effective MDI use requires the patient to be able to manipulate the device independently, which may be difficult if the patient has arthritis. Smoking does not preclude MDI use. A modest loss of vision does not preclude the use of an MDI and a patient can safely use more than one MDI. A nurse is preparing to perform an admission assessment on a patient with COPD. It is most important for the nurse to review which of the following? Social work assessment Insurance coverage Chloride levels Available diagnostic tests D Feedback: In addition to the patients history, the nurse reviews the results of available diagnostic tests. Social work assessment is not a priority for the majority of patients. Chloride levels are relevant to CF, not COPD. Insurance coverage is not normally the domain of the nurse. An admitting nurse is assessing a patient with COPD. The nurse auscultates diminished breath sounds, which signify changes in the airway. These changes indicate to the nurse to monitor the patient for what? Kyphosis and clubbing of the fingers Dyspnea and hypoxemia Sepsis and pneumothorax Bradypnea and pursed lip breathing B Feedback: These changes in the airway require that the nurse monitor the patient for dyspnea and hypoxemia. Kyphosis is a musculoskeletal problem. Sepsis and pneumothorax are atypical complications. Tachypnea is much more likely than bradypnea. Pursed lip breathing can relieve dyspnea. A nurse is caring for a patient with COPD. The patients medication regimen has been recently changed and the nurse is assessing for therapeutic effect of a new bronchodilator. What assessment parameters suggest a consequent improvement in respiratory status? Select all that apply. Negative sputum culture Increased viscosity of lung secretions Increased respiratory rate Increased expiratory flow rate Relief of dyspnea D, E Feedback: The relief of bronchospasm is confirmed by measuring improvement in expiratory flow rates and volumes (the force of expiration, how long it takes to exhale, and the amount of air exhaled) as well as by assessing the dyspnea and making sure that it has lessened. Increased respiratory rate and viscosity of secretions would suggest a worsening of the patients respiratory status. Bronchodilators would not have a direct result on the patients infectious process. A nurses assessment reveals that a client with COPD may be experiencing bronchospasm. What assessment finding would suggest that the patient is experiencing bronchospasm? Fine or coarse crackles on auscultation Wheezes or diminished breath sounds on auscultation Reduced respiratory rate or lethargy Slow, deliberate respirations B Feedback: Wheezing and diminished breath sounds are consistent with bronchospasm. Crackles are usually attributable to other respiratory or cardiac pathologies. Bronchospasm usually results in rapid, inefficient breathing and agitation. The case manager for a group of patients with COPD is providing health education. What is most important for the nurse to assess when providing instructions on self-management to these patients? Knowledge of alternative treatment modalities Family awareness of functional ability and activities of daily living (ADLs) Knowledge of the pathophysiology of the disease process Knowledge about self-care and their therapeutic regimen D Feedback: When providing instructions about self-management, it is important for the nurse to assess the knowledge of patients and family members about self-care and the therapeutic regimen. This supersedes knowledge of alternative treatments or the pathophysiology of the disease, neither of which is absolutely necessary for patients to know. The patients own knowledge is more important than that of the family. A nurse is developing a teaching plan for a patient with COPD. What should the nurse include as the most important area of teaching? Avoiding extremes of heat and cold Setting and accepting realistic short- and long-range goals Adopting a lifestyle of moderate activity Avoiding emotional disturbances and stressful situations B Feedback: A major area of teaching involves setting and accepting realistic short-term and long-range goals. The other options should also be included in the teaching plan, but they are not areas that are as high a priority as setting and accepting realistic goals. A nurse is assessing a patient who is suspected of having bronchiectasis. The nurse should consider which of the following potential causes? Select all that apply. A) Pulmonary hypertension B) Airway obstruction C) Pulmonary infections D) Genetic disorders E) Atelectasis B, C, D Feedback: Bronchiectasis is a chronic, irreversible dilation of the bronchi and bronchioles. Under the new definition of COPD, it is considered a disease process separate from COPD. Bronchiectasis may be caused by a variety of conditions, including airway obstruction, diffuse airway injury, pulmonary infections and obstruction of the bronchus or complications of long-term pulmonary infections, or genetic disorders such as cystic fibrosis. Bronchiectasis is not caused by pulmonary hypertension or atelectasis. A nurse is planning the care of a client with bronchiectasis. What goal of care should the nurse prioritize? A) The patient will successfully mobilize pulmonary secretions. B) The patient will maintain an oxygen saturation level of 98%. C) The patients pulmonary blood pressure will decrease to within reference ranges. D) The patient will resume prediagnosis level of function within 72 hours. A Feedback: Nursing management focuses on alleviating symptoms and helping patients clear pulmonary secretions. Pulmonary pressures are not a central focus in the care of the patient with bronchiectasis. Rapid resumption of prediagnosis function and oxygen saturation above 98% are unrealistic goals. An interdisciplinary team is planning the care of a patient with bronchiectasis. What aspects of care should the nurse anticipate? Select all that apply. A) Occupational therapy B) Antimicrobial therapy C) Positive pressure isolation D) Chest physiotherapy E) Smoking cessation B, D, E Feedback: Chest physiotherapy, antibiotics, and smoking cessation are cornerstones of the care of patients with bronchiectasis. Occupational therapy and isolation are not normally indicated. A patients severe asthma has necessitated the use of a long-acting beta2-agonist (LABA). Which of the patients statements suggests a need for further education? A) I know that these drugs can sometimes make my heart beat faster. B) Ive heard that this drug is particularly good at preventing asthma attacks during exercise. C) Ill make sure to use this each time I feel an asthma attack coming on. D) Ive heard that this drug sometimes gets less effective over time. C Feedback: LABAs are not used for management of acute asthma symptoms. Tachycardia is a potential adverse effect and decreased protection against exercise-induced bronchospasm may occur with regular use. A nurse is providing health education to the family of a patient with bronchiectasis. What should the nurse teach the patients family members? A) The correct technique for chest palpation and auscultation B) Techniques for assessing the patients fluid balance C) The technique for providing deep nasotracheal suctioning D) The correct technique for providing postural drainage D Feedback: A focus of the care of bronchiectasis is helping patients clear pulmonary secretions; consequently, patients and families are taught to perform postural drainage. Chest palpation and auscultation and assessment of fluid balance are not prioritized over postural drainage. Nasotracheal suctioning is not normally necessary. A nurse is working with a child who is undergoing a diagnostic workup for suspected asthma. What are the signs and symptoms that are consistent with a diagnosis of asthma? Select all that apply. A) Chest tightness B) Crackles C) Bradypnea D) Wheezing E) Cough A, D, E Feedback: Asthma is a chronic inflammatory disease of the airways that causes airway hyperresponsiveness, mucosal edema, and mucus production. This inflammation ultimately leads to recurrent episodes of asthma symptoms: cough, chest tightness, wheezing, and dyspnea. Crackles and bradypnea are not typical symptoms of asthma. A nurse is caring for a patient who has been hospitalized with an acute asthma exacerbation. What drugs should the nurse expect to be ordered for this patient to gain underlying control of persistent asthma? A) Rescue inhalers B) Anti-inflammatory drugs C) Antibiotics D) Antitussives B Feedback: Because the underlying pathology of asthma is inflammation, control of persistent asthma is accomplished primarily with regular use of anti-inflammatory medications. Rescue inhalers, antibiotics, and antitussives do not aid in the first-line control of persistent asthma. A nurse is teaching a patient with asthma about Azmacort, an inhaled corticosteroid. Which adverse effects should the nurse be sure to address in patient teaching? A) Dyspnea and increased respiratory secretions B) Nausea and vomiting C) Cough and oral thrush D) Fatigue and decreased level of consciousness C Feedback: Azmacort has possible adverse effects of cough, dysphonia, oral thrush (candidiasis), and headache. In high doses, systemic effects may occur (e.g., adrenal suppression, osteoporosis, skin thinning, and easy bruising). The other listed adverse effects are not associated with this drug. A nurse is explaining to a patient with asthma what her new prescription for prednisone is used for. What would be the most accurate explanation that the nurse could give? A) To ensure long-term prevention of asthma exacerbations B) To cure any systemic infection underlying asthma attacks C) To prevent recurrent pulmonary infections D) To gain prompt control of inadequately controlled, persistent asthma D Feedback: Prednisone is used for a short-term (310 days) burst to gain prompt control of inadequately controlled, persistent asthma. It is not used to treat infection or to prevent exacerbations in the long term. An asthma nurse educator is working with a group of adolescent asthma patients. What intervention is most likely to prevent asthma exacerbations among these patients? A) Encouraging patients to carry a corticosteroid rescue inhaler at all times B) Educating patients about recognizing and avoiding asthma triggers C) Teaching patients to utilize alternative therapies in asthma management D) Ensuring that patients keep their immunizations up to date B Feedback: Asthma exacerbations are best managed by early treatment and education, including the use of written action plans as part of any overall effort to educate patients about self-management techniques, especially those with moderate or severe persistent asthma or with a history of severe exacerbations. Corticosteroids are not used as rescue inhalers. Alternative therapies are not normally a high priority, though their use may be appropriate in some cases. Immunizations should be kept up to date, but this does not necessarily prevent asthma exacerbations. An asthma educator is teaching a patient newly diagnosed with asthma and her family about the use of a peak flow meter. The educator should teach the patient that a peak flow meter measures what value? A) Highest airflow during a forced inspiration B) Highest airflow during a forced expiration C) Airflow during a normal inspiration D) Airflow during a normal expiration B Feedback: Peak flow meters measure the highest airflow during a forced expiration. A nurse is admitting a new patient who has been admitted with a diagnosis of COPD exacerbation. How can the nurse best help the patient achieve the goal of maintaining effective oxygenation? A) Teach the patient strategies for promoting diaphragmatic breathing. B) Administer supplementary oxygen by simple face mask. C) Teach the patient to perform airway suctioning. D) Assist the patient in developing an appropriate exercise program. A Feedback: The breathing pattern of most people with COPD is shallow, rapid, and inefficient; the more severe the disease, the more inefficient the breathing pattern. With practice, this type of upper chest breathing can be changed to diaphragmatic breathing, which reduces the respiratory rate, increases alveolar ventilation, and sometimes helps expel as much air as possible during expiration. Suctioning is not normally necessary in patients with COPD. Supplementary oxygen is not normally delivered by simple face mask and exercise may or may not be appropriate. Chapter 25: Assessment of Cardiovascular Function A nurse is describing the process by which blood is ejected into circulation as the chambers of the heart become smaller. The instructor categorizes this action of the heart as what? A) Systole B) Diastole C) Repolarization D) Ejection fraction A Feedback: Systole is the action of the chambers of the heart becoming smaller and ejecting blood. This action of the heart is not diastole (relaxations), ejection fraction (the amount of blood expelled), or repolarization (electrical charging). 2. During a shift assessment, the nurse is identifying the clients point of maximum impulse (PMI). Where will the nurse best palpate the PMI? A) Left midclavicular line of the chest at the level of the nipple B) Left midclavicular line of the chest at the fifth intercostal space C) Midline between the xiphoid process and the left nipple D) Two to three centimeters to the left of the sternum B Feedback: The left ventricle is responsible for the apical beat or the point of maximum impulse, which is normally palpated in the left midclavicular line of the chest wall at the fifth intercostal space. 3. The nurse is calculating a cardiac patients pulse pressure. If the patients blood pressure is 122/76 mm Hg, what is the patients pulse pressure? A) 46 mm Hg B) 99 mm Hg C) 198 mm Hg D) 76 mm Hg A Feedback: Pulse pressure is the difference between the systolic and diastolic pressure. In this case, this value is 46 mm Hg. 4. The nurse is caring for a patient admitted with unstable angina. The laboratory result for the initial troponin I is elevated in this patient. The nurse should recognize what implication of this assessment finding? A) This is only an accurate indicator of myocardial damage when it reaches its peak in 24 hours. B) Because the patient has a history of unstable angina, this is a poor indicator of myocardial injury. C) This is an accurate indicator of myocardial injury. D) This result indicates muscle injury, but does not specify the source. C Feedback: Troponin I, which is specific to cardiac muscle, is elevated within hours after myocardial injury. Even with a diagnosis of unstable angina, this is an accurate indicator of myocardial injury. 5. The nurse is conducting patient teaching about cholesterol levels. When discussing the patients elevated LDL and lowered HDL levels, the patient shows an understanding of the significance of these levels by stating what? A) Increased LDL and decreased HDL increase my risk of coronary artery disease. B) Increased LDL has the potential to decrease my risk of heart disease. C) The decreased HDL level will increase the amount of cholesterol moved away from the artery walls. D) The increased LDL will decrease the amount of cholesterol deposited on the artery walls. A Feedback: Elevated LDL levels and decreased HDL levels are associated with a greater incidence of coronary artery disease. 6. The physician has placed a central venous pressure (CVP) monitoring line in an acutely ill patient so right ventricular function and venous blood return can be closely monitored. The results show decreased CVP. What does this indicate? A) Possible hypovolemia B) Possible myocardial infarction (MI) C) Left-sided heart failure D) Aortic valve regurgitation A Feedback: Hypovolemia may cause a decreased CVP. MI, valve regurgitation and heart failure are less likely causes of decreased CVP. 7. While auscultating a patients heart sounds, the nurse hears an extra heart sound immediately after the second heart sound (S2). An audible S3 would be considered an expected finding in what patient? A) An older adult B) A 20-year-old patient C) A patient who has undergone valve replacement D) A patient who takes a beta-adrenergic blocker B Feedback: S3 represents a normal finding in children and adults up to 35 or 40 years of age. In these cases, it is called a physiologic S3. It is an abnormal finding in a patient with an artificial valve, an older adult, or a patient who takes a beta blocker. 8. The physical therapist notifies the nurse that a patient with coronary artery disease (CAD) experiences a much greater-than-average increase in heart rate during physical therapy. The nurse recognizes that an increase in heart rate in a patient with CAD may result in what? A) Development of an atrial-septal defect B) Myocardial ischemia C) Formation of a pulmonary embolism D) Release of potassium ions from cardiac cells B Feedback: Unlike other arteries, the coronary arteries are perfused during diastole. An increase in heart rate shortens diastole and can decrease myocardial perfusion. Patients, particularly those with CAD, can develop myocardial ischemia. An increase in heart rate will not usually result in a pulmonary embolism or create electrolyte imbalances. Atrial-septal defects are congenital. 9. The nurse is caring for a patient who has a history of heart disease. What factor should the nurse identify as possibly contributing to a decrease in cardiac output? A) A change in position from standing to sitting B) A heart rate of 54 bpm C) A pulse oximetry reading of 94% D) An increase in preload related to ambulation B Feedback: Cardiac output is computed by multiplying the stroke volume by the heart rate. Cardiac output can be affected by changes in either stroke volume or heart rate, such as a rate of 54 bpm. An increase in preload will lead to an increase in stroke volume. A pulse oximetry reading of 94% does not indicate hypoxemia, as hypoxia can decrease contractility. Transitioning from standing to sitting would more likely increase rather than decrease cardiac output. 10. The nurse is caring for an 82-year-old patient. The nurse knows that changes in cardiac structure and function occur in older adults. What is a normal change expected in the aging heart of an older adult? A) Decreased left ventricular ejection time B) Decreased connective tissue in the SA and AV nodes and bundle branches C) Thinning and flaccidity of the cardiac values D) Widening of the aorta D Feedback: Changes in cardiac structure and function are clearly observable in the aging heart. Aging results in decreased elasticity and widening of the aorta, thickening and rigidity of the cardiac valves, increased connective tissue in the SA and AV nodes and bundle branches, and an increased left ventricular ejection time (prolonged systole). 11. A resident of a long-term care facility has complained to the nurse of chest pain. What aspect of the residents pain would be most suggestive of angina as the cause? A) The pain is worse when the resident inhales deeply. B) The pain occurs immediately following physical exertion. C) The pain is worse when the resident coughs. D) The pain is most severe when the resident moves his upper body. B Feedback: Chest pain associated with angina is often precipitated by physical exertion. The other listed aspects of chest pain are more closely associated with noncardiac etiologies. 12. The critical care nurse is caring for a patient with a central venous pressure (CVP) monitoring system. The nurse notes that the patients CVP is increasing. Of what may this indicate? A) Psychosocial stress B) Hypervolemia C) Dislodgment of the catheter D) Hypomagnesemia B Feedback: CVP is a useful hemodynamic parameter to observe when managing an unstable patients fluid volume status. An increasing pressure may be caused by hypervolemia or by a condition, such as heart failure, that results in decreased myocardial contractility. Stress, dislodgement of the catheter, and low magnesium levels would not typically result in increased CVP. 13. The critical care nurse is caring for a patient with a pulmonary artery pressure monitoring system. The nurse is aware that pulmonary artery pressure monitoring is used to assess left ventricular function. What is an additional function of pulmonary artery pressure monitoring systems? A) To assess the patients response to fluid and drug administration B) To obtain specimens for arterial blood gas measurements C) To dislodge pulmonary emboli D) To diagnose the etiology of chronic obstructive pulmonary disease A Feedback: Pulmonary artery pressure monitoring is an important tool used in critical care for assessing left ventricular function (cardiac output), diagnosing the etiology of shock, and evaluating the patients response to medical interventions, such as fluid administration and vasoactive medications. Pulmonary artery monitoring is preferred for the patient with heart failure over central venous pressure monitoring. Arterial catheters are useful when arterial blood gas measurements and blood samples need to be obtained frequently. Neither intervention is used to clear pulmonary emboli. 14. The cardiac care nurse is reviewing the conduction system of the heart. The nurse is aware that electrical conduction of the heart usually originates in the SA node and then proceeds in what sequence? A) SA node to bundle of His to AV node to Purkinje fibers B) SA node to AV node to Purkinje fibers to bundle of His C) SA node to bundle of His to Purkinje fibers to AV node D) SA node to AV node to bundle of His to Purkinje fibers D Feedback: The normal electrophysiological conduction route is SA node to AV node to bundle of HIS to Purkinje fibers. 15. A patient has had a myocardial infarction and has been diagnosed as having damage to the layer of the heart responsible for the pumping action. You are aware that the damage occurred where? A) Endocardium B) Pericardium C) Myocardium D) Visceral pericardium C Feedback: The myocardium is the layer of the heart responsible for the pumping action. 16. The nurse working on a cardiac care unit is caring for a patient whose stroke volume has increased. The nurse is aware that afterload influences a patients stroke volume. The nurse recognizes that afterload is increased when there is what? A) Arterial vasoconstriction B) Venous vasoconstriction C) Arterial vasodilation D) Venous vasodilation A Feedback: Arterial vasoconstriction increases the systemic vascular resistance, which increases the afterload. Venous vasoconstriction decreases preload thereby decreasing stroke volume. Venous vasodilation increases preload. 17. A nurse is preparing a patient for scheduled transesophageal echocardiography. What action should the nurse perform? A) Instruct the patient to drink 1 liter of water before the test. B) Administer IV benzodiazepines and opioids. C) Inform the patient that she will remain on bed rest following the procedure. D) Inform the patient that an access line will be initiated in her femoral artery. C Feedback: During the recovery period, the patient must maintain bed rest with the head of the bed elevated to 45 degrees. The patient must be NPO 6 hours preprocedure. The patient is sedated to make him or her comfortable, but will not be heavily sedated, and opioids are not necessary. Also, the patient will have a peripheral IV line initiated preprocedure. 18. The nurse is caring for a patient admitted with angina who is scheduled for cardiac catheterization. The patient is anxious and asks the reason for this test. What is the best response? A) Cardiac catheterization is usually done to assess how blocked or open a patients coronary arteries are. B) Cardiac catheterization is most commonly done to detect how efficiently a patients heart muscle contracts. C) Cardiac catheterization is usually done to evaluate cardiovascular response to stress. D) Cardiac catheterization is most commonly done to evaluate cardiac electrical activity. A Feedback: Cardiac catheterization is usually used to assess coronary artery patency to determine if revascularization procedures are necessary. A thallium stress test shows myocardial ischemia after stress. An ECG shows the electrical activity of the heart. 19. The critical care nurse is caring for a patient who has had an MI. The nurse should expect to assist with establishing what hemodynamic monitoring procedure to assess the patients left ventricular function? A) Central venous pressure (CVP) monitoring B) Pulmonary artery pressure monitoring (PAPM) C) Systemic arterial pressure monitoring (SAPM) D) Arterial blood gases (ABG) B Feedback: PAPM is used to assess left ventricular function. CVP is used to assess right ventricular function; SAPM is used for continual assessment of BP. ABG are used to assess for acidic and alkalotic levels in the blood. 20. A critically ill patient is admitted to the ICU. The physician decides to use intra-arterial pressure monitoring. After this intervention is performed, what assessment should the nurse prioritize in the plan of care? A) Fluctuations in core body temperature B) Signs and symptoms of esophageal varices C) Signs and symptoms of compartment syndrome D) Perfusion distal to the insertion site D Feedback: The radial artery is the usual site selected. However, placement of a catheter into the radial artery can further impede perfusion to an area that has poor circulation. As a result, the tissue distal to the cannulated artery can become ischemic or necrotic. Vigilant assessment is thus necessary. Alterations in temperature and the development of esophageal varices or compartment syndrome are not high risks. 21. The nurse is caring for an acutely ill patient who has central venous pressure monitoring in place. What intervention should be included in the care plan of a patient with CVP in place? A) Apply antibiotic ointment to the insertion site twice daily. B) Change the site dressing whenever it becomes visibly soiled. C) Perform passive range-of-motion exercises to prevent venous stasis. D) Aspirate blood from the device once daily to test pH. B Feedback: Gauze dressings should be changed every 2 days or transparent dressings at least every 7 days and whenever dressings become damp, loosened, or visibly soiled. Passive ROM exercise is not indicated and it is unnecessary and inappropriate to aspirate blood to test it for pH. Antibiotic ointments are contraindicated. 22. A patient is brought into the ED by family members who tell the nurse the patient grabbed his chest and complained of substernal chest pain. The care team recognizes the need to monitor the patients cardiac function closely while interventions are performed. What form of monitoring should the nurse anticipate? A) Left-sided heart catheterization B) Cardiac telemetry C) Transesophageal echocardiography D) Hardwire continuous ECG monitoring D Feedback: Two types of continuous ECG monitoring techniques are used in health care settings: hardwire cardiac monitoring, found in EDs, critical care units, and progressive care units; and telemetry, found in general nursing care units or outpatient cardiac rehabilitation programs. Cardiac catheterization and transesophageal echocardiography would not be used in emergent situations to monitor cardiac function. 23. The nurse is performing an intake assessment on a patient with a new diagnosis of coronary artery disease. What would be the most important determination to make during this intake assessment? A) Whether the patient and involved family members understand the role of genetics in the etiology of the disease B) Whether the patient and involved family members understand dietary changes and the role of nutrition C) Whether the patient and involved family members are able to recognize symptoms of an acute cardiac problem and respond appropriately D) Whether the patient and involved family members understand the importance of social support and community agencies C Feedback: During the health history, the nurse needs to determine if the patient and involved family members are able to recognize symptoms of an acute cardiac problem, such as acute coronary syndrome (ACS) or HF, and seek timely treatment for these symptoms. Each of the other listed topics is valid, but the timely and appropriate response to a cardiac emergency is paramount. 24. The nurse is relating the deficits in a patients synchronization of the atrial and ventricular events to his diagnosis. What are the physiologic characteristics of the nodal and Purkinje cells that provide this synchronization? Select all that apply. A) Loop connectivity B) Excitability C) Automaticity D) Conductivity E) Independence B, C, D Feedback: Three physiologic characteristics of two types of specialized electrical cells, the nodal cells and the Purkinje cells, provide this synchronization: automaticity, or the ability to initiate an electrical impulse; excitability, or the ability to respond to an electrical impulse; and conductivity, the ability to transmit an electrical impulse from one cell to another. Loop connectivity is a distracter for this question. Independence of the cells has nothing to do with the synchronization described in the scenario. 25. The nurses assessment of an older adult client reveals the following data: Lying BP 144/82 mm Hg; sitting BP 121/69 mm Hg; standing BP 98/56 mm Hg. The nurse should consequently identify what nursing diagnosis in the patients plan of care? A) Risk for ineffective breathing pattern related to hypotension B) Risk for falls related to orthostatic hypotension C) Risk for ineffective role performance related to hypotension D) Risk for imbalanced fluid balance related to hemodynamic variability B Feedback: Orthostatic hypotension creates a significant risk for falls due to the dizziness and lightheadedness that accompanies it. It does not normally affect breathing or fluid balance. The patients ability to perform normal roles may be affected, but the risk for falls is the most significant threat to safety. 26. A brain (B-type) natriuretic peptide (BNP) sample has been drawn from an older adult patient who has been experienced vital fatigue and shortness of breath. This test will allow the care team to investigate the possibility of what diagnosis? A) Pleurisy B) Heart failure C) Valve dysfunction D) Cardiomyopathy B Feedback: The level of BNP in the blood increases as the ventricular walls expand from increased pressure, making it a helpful diagnostic, monitoring, and prognostic tool in the setting of HF. It is not specific to cardiomyopathy, pleurisy, or valve dysfunction. 27. A lipid profile has been ordered for a patient who has been experiencing cardiac symptoms. When should a lipid profile be drawn in order to maximize the accuracy of results? A) As close to the end of the day as possible B) After a meal high in fat C) After a 12-hour fast D) Thirty minutes after a normal meal C Feedback: Although cholesterol levels remain relatively constant over 24 hours, the blood specimen for the lipid profile should be obtained after a 12-hour fast. 28. When hemodynamic monitoring is ordered for a patient, a catheter is inserted into the appropriate blood vessel or heart chamber. When assessing a patient who has such a device in place, the nurse should check which of the following components? Select all that apply. A) A transducer B) A flush system C) A leveler D) A pressure bag E) An oscillator A, B, D Feedback: To perform hemodynamic monitoring, a CVP, pulmonary artery, or arterial catheter is introduced into the appropriate blood vessel or heart chamber. It is connected to a pressure monitoring system that has several components. Included among these are a transducer, a flush system, and a pressure bag. A pressure monitoring system does not have a leveler or an oscillator. 29. The critical care nurse is caring for a patient who has been experiencing bradycardia after cardiovascular surgery. The nurse knows that the heart rate is determined by myocardial cells with the fastest inherent firing rate. Under normal circumstances where are these cells located? A) SA node B) AV node C) Bundle of His D) Purkinje cells A Feedback: The heart rate is determined by the myocardial cells with the fastest inherent firing rate. Under normal circumstances, the SA node has the highest inherent rate (60 to 100 impulses per minute). 30. The nurse is doing discharge teaching with a patient who has coronary artery disease. The patient asks why he has to take an aspirin every day if he doesnt have any pain. What would be the nurses best response? A) Taking an aspirin every day is an easy way to help restore the normal function of your heart. B) An aspirin a day can help prevent some of the blockages that can cause chest pain or heart attacks. C) Taking an aspirin every day is a simple way to make your blood penetrate your heart more freely. D) An aspirin a day eventually helps your blood carry more oxygen that it would otherwise. B Feedback: An aspirin a day is a common nonprescription medication that improves outcomes in patients with CAD due to its antiplatelet action. It does not affect oxygen carrying capacity or perfusion. Aspirin does not restore cardiac function. 31. The physician has ordered a high-sensitivity C-reactive protein (hs-CRP) drawn on a patient. The results of this test will allow the nurse to evaluate the role of what process that is implicated in the development of atherosclerosis? A) Immunosuppression B) Inflammation C) Infection D) Hemostasis B Feedback: High-sensitivity CRP is a protein produced by the liver in response to systemic inflammation. Inflammation is thought to play a role in the development and progression of atherosclerosis. 32. The patient has a homocysteine level ordered. What aspects of this test should inform the nurses care? Select all that apply. A) A 12-hour fast is necessary before drawing the blood sample. B) Recent inactivity can depress homocysteine levels. C) Genetic factors can elevate homocysteine levels. D) A diet low in folic acid elevates homocysteine levels. E) An ECG should be performed immediately before drawing a sample. A, C, D Feedback: Genetic factors and a diet low in folic acid, vitamin B6, and vitamin B12 are associated with elevated homocysteine levels. A 12-hour fast is necessary before drawing a blood sample for an accurate serum measurement. An ECG is unnecessary and recent inactivity does not influence the results of the test. 33. A patient with a complex cardiac history is scheduled for transthoracic echocardiography. What should the nurse teach the patient in anticipation of this diagnostic procedure? A) The test is noninvasive, and nothing will be inserted into the patients body. B) The patients pain will be managed aggressively during the procedure. C) The test will provide a detailed profile of the hearts electrical activity. D) The patient will remain on bed rest for 1 to 2 hours after the test. A Feedback: Before transthoracic echocardiography, the nurse informs the patient about the test, explaining that it is painless. The test does not evaluate electrophysiology and bed rest is unnecessary after the procedure. 34. A critical care nurse is caring for a patient with a hemodynamic monitoring system in place. For what complications should the nurse assess? Select all that apply. A) Pneumothorax B) Infection C) Atelectasis D) Bronchospasm E) Air embolism A, B, E Feedback: Complications from use of hemodynamic monitoring systems are uncommon, but can include pneumothorax, infection, and air embolism. Complications of hemodynamic monitoring systems do not include atelectasis or bronchospasm. 35. The nurse is caring for a patient who has central venous pressure (CVP) monitoring in place. The nurses most recent assessment reveals that CVP is 7 mm Hg. What is the nurses most appropriate action? A) Arrange for continuous cardiac monitoring and reposition the patient. B) Remove the CVP catheter and apply an occlusive dressing. C) Assess the patient for fluid overload and inform the physician. D) Raise the head of the patients bed and have the patient perform deep breathing exercise, if possible. C Feedback: The normal CVP is 2 to 6 mm Hg. Many problems can cause an elevated CVP, but the most common is due to hypervolemia. Assessing the patient and informing the physician are the most prudent actions. Repositioning the patient is ineffective and removing the device is inappropriate. 36. A critical care nurse is caring for a patient with a pulmonary artery catheter in place. What does this catheter measure that is particularly important in critically ill patients? A) Pulmonary artery systolic pressure B) Right ventricular afterload C) Pulmonary artery pressure D) Left ventricular preload D Feedback: Monitoring of the pulmonary artery diastolic and pulmonary artery wedge pressures is particularly important in critically ill patients because it is used to evaluate left ventricular filling pressures (i.e., left ventricular preload). This device does not directly measure the other listed aspects of cardiac function. 37. A patients declining cardiac status has been attributed to decreased cardiac action potential. Interventions will be aimed at restoring what aspect of cardiac physiology? A) The cycle of depolarization and repolarization B) The time it takes from the firing of the SA node to the contraction of the ventricles C) The time between the contraction of the atria and the contraction of the ventricles D) The cycle of the firing of the AV node and the contraction of the myocardium A Feedback: This exchange of ions creates a positively charged intracellular space and a negatively charged extracellular space that characterizes the period known as depolarization. Once depolarization is complete, the exchange of ions reverts to its resting state; this period is known as repolarization. The repeated cycle of depolarization and repolarization is called the cardiac action potential. 38. A patient has been scheduled for cardiovascular computed tomography (CT) with contrast. To prepare the patient for this test, what action should the nurse perform? A) Keep the patient NPO for at least 6 hours prior to the test. B) Establish peripheral IV access. C) Limit the patients activity for 2 hours before the test. D) Teach the patient to perform incentive spirometry. B Feedback: An IV is necessary if contrast is to be used to enhance the images of the CT. The patient does not need to fast or limit his or her activity. Incentive spirometry is not relevant to this diagnostic test. 39. The student nurse is preparing a teaching plan for a patient being discharged status post MI. What should the student include in the teaching plan? (Mark all that apply.) A) Need for careful monitoring for cardiac symptoms B) Need for carefully regulated exercise C) Need for dietary modifications D) Need for early resumption of prediagnosis activity E) Need for increased fluid intake A, B, C Feedback: Dietary modifications, exercise, weight loss, and careful monitoring are important strategies for managing three major cardiovascular risk factors: hyperlipidemia, hypertension, and diabetes. There is no need to increase fluid intake and activity should be slowly and deliberately increased. 40. The nurse is caring for a patient who is undergoing an exercise stress test. Prior to reaching the target heart rate, the patient develops chest pain. What is the nurses most appropriate response? A) Administer sublingual nitroglycerin to allow the patient to finish the test. B) Initiate cardiopulmonary resuscitation. C) Administer analgesia and slow the test. D) Stop the test and monitor the patient closely. D Feedback: Signs of myocardial ischemia would necessitate stopping the test. CPR would only be necessary if signs of cardiac or respiratory arrest were evident. Chapter 26: Management of Patients With Dysrhythmias and Conduction Problems 1. The nurse is caring for a patient who has had an ECG. The nurse notes that leads I, II, and III differ from one another on the cardiac rhythm strip. How should the nurse best respond? A) Recognize that the view of the electrical current changes in relation to the lead placement. B) Recognize that the electrophysiological conduction of the heart differs with lead placement. C) Inform the technician that the ECG equipment has malfunctioned. D) Inform the physician that the patient is experiencing a new onset of dysrhythmia. A Feedback: Each lead offers a different reference point to view the electrical activity of the heart. The lead displays the configuration of electrical activity of the heart. Differences between leads are not necessarily attributable to equipment malfunction or dysrhythmias. 2. The nurse is analyzing a rhythm strip. What component of the ECG corresponds to the resting state of the patients heart? A) P wave B) T wave C) U wave D) QRS complex B Feedback: The T wave specifically represents ventricular muscle depolarization, also referred to as the resting state. Ventricular muscle depolarization does not result in the P wave, U wave, or QRS complex. 3. The nursing educator is presenting a case study of an adult patient who has abnormal ventricular depolarization. This pathologic change would be most evident in what component of the ECG? A) P wave B) T wave C) QRS complex D) U wave C Feedback: The QRS complex represents the depolarization of the ventricles and, as such, the electrical activity of that ventricle. 4. An adult patient with third-degree AV block is admitted to the cardiac care unit and placed on continuous cardiac monitoring. What rhythm characteristic will the ECG most likely show? A) PP interval and RR interval are irregular. B) PP interval is equal to RR interval. C) Fewer QRS complexes than P waves D) PR interval is constant. C Feedback: In third-degree AV block, no atrial impulse is conducted through the AV node into the ventricles. As a result, there are impulses stimulating the atria and impulses stimulating the ventricles. Therefore, there are more P waves than QRS complexes due to the difference in the natural pacemaker (nodes) rates of the heart. The other listed ECG changes are not consistent with this diagnosis. 5. The nurse is writing a plan of care for a patient with a cardiac dysrhythmia. What would be the most appropriate goal for the patient? A) Maintain a resting heart rate below 70 bpm. B) Maintain adequate control of chest pain. C) Maintain adequate cardiac output. D) Maintain normal cardiac structure. C Feedback: For patient safety, the most appropriate goal is to maintain cardiac output to prevent worsening complications as a result of decreased cardiac output. A resting rate of less than 70 bpm is not appropriate for every patient. Chest pain is more closely associated with acute coronary syndrome than with dysrhythmias. Nursing actions cannot normally influence the physical structure of the heart. 6. A patient has returned to the cardiac care unit after having a permanent pacemaker implantation. For which potential complication should the nurse most closely assess this patient? A) Chest pain B) Bleeding at the implantation site C) Malignant hyperthermia D) Bradycardia B Feedback: Bleeding, hematomas, local infections, perforation of the myocardium, and tachycardia are complications of pacemaker implantations. The nurse should monitor for chest pain and bradycardia, but bleeding is a more common immediate complication. Malignant hyperthermia is unlikely because it is a response to anesthesia administration. 7. A patient the nurse is caring for has a permanent pacemaker implanted with the identification code beginning with VVI. What does this indicate? A) Ventricular paced, ventricular sensed, inhibited B) Variable paced, ventricular sensed, inhibited C) Ventricular sensed, ventricular situated, implanted D) Variable sensed, variable paced, inhibited A Feedback: The identification of VVI indicates ventricular paced, ventricular sensed, inhibited. 8. The nurse is caring for an adult patient who has gone into ventricular fibrillation. When assisting with defibrillating the patient, what must the nurse do? A) Maintain firm contact between paddles and patient skin. B) Apply a layer of water as a conducting agent. C) Call all clear once before discharging the defibrillator. D) Ensure the defibrillator is in the sync mode. A Feedback: When defibrillating an adult patient, the nurse should maintain good contact between the paddles and the patients skin to prevent arcing, apply an appropriate conducting agent (not water) between the skin and the paddles, and ensure the defibrillator is in the nonsync mode. Clear should be called three times before discharging the paddles. 9. A patient who is a candidate for an implantable cardioverter defibrillator (ICD) asks the nurse about the purpose of this device. What would be the nurses best response? A) To detect and treat dysrhythmias such as ventricular fibrillation and ventricular tachycardia B) To detect and treat bradycardia, which is an excessively slow heart rate C) To detect and treat atrial fibrillation, in which your heart beats too quickly and inefficiently D) To shock your heart if you have a heart attack at home A Feedback: The ICD is a device that detects and terminates life-threatening episodes of ventricular tachycardia and ventricular fibrillation. It does not treat atrial fibrillation, MI, or bradycardia. 10. A nurse is providing health education to a patient scheduled for cryoablation therapy. The nurse should describe what aspect of this treatment? A) Peeling away the area of endocardium responsible for the dysrhythmia B) Using electrical shocks directly to the endocarduim to eliminate the source of dysrhythmia C) Using high-frequency sound waves to eliminate the source of dysrhythmia D) Using a cooled probe to eliminate the source of dysrhythmia D Feedback: Cryoablation therapy involves using a cooled probe to create a small scar on the endocardium to eliminate the source of the dysrhythmias. Endocardium resection involves peeling away a specified area of the endocardium. Electrical ablation involves using shocks to eliminate the area causing the dysrhythmias. Radio frequency ablation uses high-frequency sound waves to destroy the area causing the dysrhythmias. 11. The nurse is caring for a patient who has just had an implantable cardioverter defibrillator (ICD) placed. What is the priority area for the nurses assessment? A) Assessing the patients activity level B) Facilitating transthoracic echocardiography C) Vigilant monitoring of the patients ECG D) Close monitoring of the patients peripheral perfusion C Feedback: After a permanent electronic device (pacemaker or ICD) is inserted, the patients heart rate and rhythm are monitored by ECG. This is a priority over peripheral circulation and activity. Echocardiography is not indicated. 12. During a patients care conference, the team is discussing whether the patient is a candidate for cardiac conduction surgery. What would be the most important criterion for a patient to have this surgery? A) Angina pectoris not responsive to other treatments B) Decreased activity tolerance related to decreased cardiac output C) Atrial and ventricular tachycardias not responsive to other treatments D) Ventricular fibrillation not responsive to other treatments C Feedback: Cardiac conduction surgery is considered in patients who do not respond to medications and antitachycardia pacing. Angina, reduced activity tolerance, and ventricular fibrillation are not criteria. 13. A nurse is caring for a patient who is exhibiting ventricular tachycardia (VT). Because the patient is pulseless, the nurse should prepare for what intervention? A) Defibrillation B) ECG monitoring C) Implantation of a cardioverter defibrillator D) Angioplasty A Feedback: Any type of VT in a patient who is unconscious and without a pulse is treated in the same manner as ventricular fibrillation: Immediate defibrillation is the action of choice. ECG monitoring is appropriate, but this is an assessment, not an intervention, and will not resolve the problem. An ICD and angioplasty do not address the dysrhythmia. 14. A patient converts from normal sinus rhythm at 80 bpm to atrial fibrillation with a ventricular response at 166 bpm. Blood pressure is 162/74 mm Hg. Respiratory rate is 20 breaths per minute with normal chest expansion and clear lungs bilaterally. IV heparin and Cardizem are given. The nurse caring for the patient understands that the main goal of treatment is what? A) Decrease SA node conduction B) Control ventricular heart rate C) Improve oxygenation D) Maintain anticoagulation B Feedback: Treatment for atrial fibrillation is to terminate the rhythm or to control ventricular rate. This is a priority because it directly affects cardiac output. A rapid ventricular response reduces the time for ventricular filling, resulting in a smaller stroke volume. Control of rhythm is the initial treatment of choice, followed by anticoagulation with heparin and then Coumadin. 15. The nurse and the other members of the team are caring for a patient who converted to ventricular fibrillation (VF). The patient was defibrillated unsuccessfully and the patient remains in VF. According to national standards, the nurse should anticipate the administration of what medication? A) Epinephrine 1 mg IV push B) Lidocaine 100 mg IV push C) Amiodarone 300 mg IV push D) Sodium bicarbonate 1 amp IV push A Feedback: Epinephrine should be administered as soon as possible after the first unsuccessful defibrillation and then every 3 to 5 minutes. Antiarrhythmic medications such as amiodarone and licocaine are given if ventricular dysrhythmia persists. 16. The nurse is planning discharge teaching for a patient with a newly inserted permanent pacemaker. What is the priority teaching point for this patient? A) Start lifting the arm above the shoulder right away to prevent chest wall adhesion. B) Avoid cooking with a microwave oven. C) Avoid exposure to high-voltage electrical generators. D) Avoid walking through store and library antitheft devices. C Feedback: High-output electrical generators can reprogram pacemakers and should be avoided. Recent pacemaker technology allows patients to safely use most household electronic appliances and devices (e.g., microwave ovens). The affected arm should not be raised above the shoulder for 1 week following placement of the pacemaker. Antitheft alarms may be triggered so patients should be taught to walk through them quickly and avoid standing in or near these devices. These alarms generally do not interfere with pacemaker function. 17. A patient is brought to the ED and determined to be experiencing symptomatic sinus bradycardia. The nurse caring for this patient is aware the medication of choice for treatment of this dysrhythmia is the administration of atropine. What guidelines will the nurse follow when administering atropine? A) Administer atropine 0.5 mg as an IV bolus every 3 to 5 minutes to a maximum of 3.0 mg. B) Administer atropine as a continuous infusion until symptoms resolve. C) Administer atropine as a continuous infusion to a maximum of 30 mg in 24 hours. D) Administer atropine 1.0 mg sublingually. A Feedback: Atropine 0.5 mg given rapidly as an intravenous (IV) bolus every 3 to 5 minutes to a maximum total dose of 3.0 mg is the medication of choice in treating symptomatic sinus bradycardia. By this guideline, the other listed options are inappropriate. 18. An ECG has been ordered for a newly admitted patient. What should the nurse do prior to electrode placement? A) Clean the skin with providone-iodine solution. B) Ensure that the area for electrode placement is dry. C) Apply tincture of benzoin to the electrode sites and wait for it to become tacky. D) Gently abrade the skin by rubbing the electrode sites with dry gauze or cloth. D Feedback: An ECG is obtained by slightly abrading the skin with a clean dry gauze pad and placing electrodes on the body at specific areas. The abrading of skin will enhance signal transmission. Disinfecting the skin is unnecessary and conduction gel is used. 19. The nurse is caring for a patient who has just undergone catheter ablation therapy. The nurse in the step- down unit should prioritize what assessment? A) Cardiac monitoring B) Monitoring the implanted device signal C) Pain assessment D) Monitoring the patients level of consciousness (LOC) A Feedback: Following catheter ablation therapy, the patient is closely monitored to ensure the dysrhythmia does not reemerge. This is a priority over monitoring of LOC and pain, although these are valid and important assessments. Ablation does not involve the implantation of a device. 20. The ED nurse is caring for a patient who has gone into cardiac arrest. During external defibrillation, what action should the nurse perform? A) Place gel pads over the apex and posterior chest for better conduction. B) Ensure no one is touching the patient at the time shock is delivered. C) Continue to ventilate the patient via endotracheal tube during the procedure. D) Allow at least 3 minutes between shocks. B Feedback: In external defibrillation, both paddles may be placed on the front of the chest, which is the standard paddle placement. Whether using pads, or paddles, the nurse must observe two safety measures. First, maintain good contact between the pads or paddles and the patients skin to prevent leaking. Second, ensure that no one is in contact with the patient or with anything that is touching the patient when the defibrillator is discharged, to minimize the chance that electrical current will be conducted to anyone other than the patient. Ventilation should be stopped during defibrillation. 21. A group of nurses are participating in orientation to a telemetry unit. What should the staff educator tell this class about ST segments? A) They are the part of an ECG that reflects systole. B) They are the part of an ECG used to calculate ventricular rate and rhythm. C) They are the part of an ECG that reflects the time from ventricular depolarization through repolarization. D) They are the part of an ECG that represents early ventricular repolarization. D Feedback: ST segment is the part of an ECG that reflects the end of the QRS complex to the beginning of the T wave. The part of an ECG that reflects repolarization of the ventricles is the T wave. The part of an ECG used to calculate ventricular rate and rhythm is the RR interval. The part of an ECG that reflects the time from ventricular depolarization through repolarization is the QT interval. 22. The nurse is providing care to a patient who has just undergone an electrophysiologic (EP) study. The patient states that she is nervous about things going wrong during the procedure. What is the nurses best response? A) This is basically a risk-free procedure. B) Thousands of patients undergo EP every year. C) Remember that this is a step that will bring you closer to enjoying good health. D) The whole team will be monitoring you very closely for the entire procedure. D Feedback: Patients who are to undergo an EP study may be anxious about the procedure and its outcome. A detailed discussion involving the patient, the family, and the electrophysiologist usually occurs to ensure that the patient can give informed consent and to reduce the patients anxiety about the procedure. It is inaccurate to state that EP is risk-free and stating that it is common does not necessarily relieve the patients anxiety. Characterizing EP as a step toward good health does not directly address the patients anxiety. 23. New nurses on the telemetry unit have been paired with preceptors. One new nurse asks her preceptor to explain depolarization. What would be the best answer by the preceptor? A) Depolarization is the mechanical contraction of the heart muscles. B) Depolarization is the electrical stimulation of the heart muscles. C) Depolarization is the electrical relaxation of the heart muscles. D) Depolarization is the mechanical relaxation of the heart muscles. B Feedback: The electrical stimulation of the heart is called depolarization, and the mechanical contraction is called systole. Electrical relaxation is called repolarization, and mechanical relaxation is called diastole. 24. A cardiac care nurse is aware of factors that result in positive chronotropy. These factors would affect a patients cardiac function in what way? A) Exacerbating an existing dysrhythmia B) Initiating a new dysrhythmia C) Resolving ventricular tachycardia D) Increasing the heart rate D Feedback: Stimulation of the sympathetic system increases heart rate. This phenomenon is known as positive chronotropy. It does not influence dysrhythmias. 25. The nurse is caring for a patient with refractory atrial fibrillation who underwent the maze procedure several months ago. The nurse reviews the result of the patients most recent cardiac imaging, which notes the presence of scarring on the atria. How should the nurse best respond to this finding? A) Recognize that the procedure was unsuccessful. B) Recognize this as a therapeutic goal of the procedure. C) Liaise with the care team in preparation for repeating the maze procedure. D) Prepare the patient for pacemaker implantation. B Feedback: The maze procedure is an open heart surgical procedure for refractory atrial fibrillation. Small transmural incisions are made throughout the atria. The resulting formation of scar tissue prevents reentry conduction of the electrical impulse. Consequently, scar formation would constitute a successful procedure. There is no indication for repeating the procedure or implanting a pacemaker. 26. A patient is scheduled for catheter ablation therapy. When describing this procedure to the patients family, the nurse should address what aspect of the treatment? A) Resetting of the hearts contractility B) Destruction of specific cardiac cells C) Correction of structural cardiac abnormalities D) Clearance of partially occluded coronary arteries B Feedback: Catheter ablation destroys specific cells that are the cause or central conduction route of a tachydysrhythmia. It does not reset the hearts contractility and it does not address structural or vascular abnormalities. 27. A patient has undergone diagnostic testing and received a diagnosis of sinus bradycardia attributable to sinus node dysfunction. When planning this patients care, what nursing diagnosis is most appropriate? A) Acute pain B) Risk for unilateral neglect C) Risk for activity intolerance D) Risk for fluid volume excess C Feedback: Sinus bradycardia causes decreased cardiac output that is likely to cause activity intolerance. It does not typically cause pain, fluid imbalances, or neglect of a unilateral nature. 28. The nurse is caring for a patient on telemetry. The patients ECG shows a shortened PR interval, slurring of the initial QRS deflection, and prolonged QRS duration. What does this ECG show? A) Sinus bradycardia B) Myocardial infarction C) Lupus-like syndrome D) Wolf-Parkinson-White (WPW) syndrome D Feedback: In WPW syndrome there is a shortened PR interval, slurring (called a delta wave) of the initial QRS deflection, and prolonged QRS duration. These characteristics are not typical of the other listed cardiac anomalies. 29. A patient is undergoing preoperative teaching before his cardiac surgery and the nurse is aware that a temporary pacemaker will be placed later that day. What is the nurses responsibility in the care of the patients pacemaker? A) Monitoring for pacemaker malfunction or battery failure B) Determining when it is appropriate to remove the pacemaker C) Making necessary changes to the pacemaker settings D) Selecting alternatives to future pacemaker use A Feedback: Monitoring for pacemaker malfunctioning and battery failure is a nursing responsibility. The other listed actions are physician responsibilities. 30. The nurse caring for a patient whose sudden onset of sinus bradycardia is not responding adequately to atropine. What might be the treatment of choice for this patient? A) Implanted pacemaker B) Trancutaneous pacemaker C) ICD D) Asynchronous defibrillator B Feedback: If a patient suddenly develops a bradycardia, is symptomatic but has a pulse, and is unresponsive to atropine, emergency pacing may be started with transcutaneous pacing, which most defibrillators are now equipped to perform. An implanted pacemaker is not a time-appropriate option. An asynchronous defibrillator or ICD would not provide relief. 31. The nurse is caring for a patient who has had a dysrhythmic event. The nurse is aware of the need to assess for signs of diminished cardiac output (CO). What change in status may signal to the nurse a decrease in cardiac output? A) Increased blood pressure B) Bounding peripheral pulses C) Changes in level of consciousness D) Skin flushing C Feedback: The nurse conducts a physical assessment to confirm the data obtained from the history and to observe for signs of diminished cardiac output (CO) during the dysrhythmic event, especially changes in level of consciousness. Blood pressure tends to decrease with lowered CO and bounding peripheral pulses are inconsistent with this problem. Pallor, not skin flushing, is expected. 32. Following cardiac resuscitation, a patient has been placed in a state of mild hypothermia before being transferred to the cardiac intensive care unit. The nurses assessment reveals that the patient is experiencing neuromuscular paralysis. How should the nurse best respond? A) Administer hypertonic IV solution. B) Administer a bolus of warned normal saline. C) Reassess the patient in 15 minutes. D) Document this as an expected assessment finding. D Feedback: The nurse caring for a patient with hypothermia (passive or induced) needs to monitor for appropriate level of cooling, sedation, and neuromuscular paralysis to prevent seizures; myoclonus; and shivering. Neuromuscular paralysis is an expected finding and does not necessitate further interventions. 33. The nurse is caring for a patient who has had a biventricular pacemaker implanted. When planning the patients care, the nurse should recognize what goal of this intervention? A) Resynchronization B) Defibrillation C) Angioplasty D) Ablation A Feedback: Biventricular (both ventricles) pacing, also called resynchronization therapy, may be used to treat advanced heart failure that does not respond to medication. This type of pacing therapy is not called defibrillation, angioplasty, or ablation therapy. 34. When planning the care of a patient with an implanted pacemaker, what assessment should the nurse prioritize? A) Core body temperature B) Heart rate and rhythm C) Blood pressure D) Oxygen saturation level B Feedback: For patients with pacemakers, close monitoring of the heart rate and rhythm is a priority, even though each of the other listed vital signs must be assessed. 35. The nurse is assessing a patient who had a pacemaker implanted 4 weeks ago. During the patients most recent follow-up appointment, the nurse identifies data that suggest the patient may be socially isolated and depressed. What nursing diagnosis is suggested by these data? A) Decisional conflict related to pacemaker implantation B) Deficient knowledge related to pacemaker implantation C) Spiritual distress related to pacemaker implantation D) Ineffective coping related to pacemaker implantation D Feedback: Depression and isolation may be symptoms of ineffective coping with the implantation. These psychosocial symptoms are not necessarily indicative of issues related to knowledge or decisions. Further data would be needed to determine a spiritual component to the patients challenges. 36. The nurse is caring for a patient who is in the recovery room following the implantation of an ICD. The patient has developed ventricular tachycardia (VT). What should the nurse assess and document? A) ECG to compare time of onset of VT and onset of devices shock B) ECG so physician can see what type of dysrhythmia the patient has C) Patients level of consciousness (LOC) at the time of the dysrhythmia D) Patients activity at time of dysrhythmia A Feedback: If the patient has an ICD implanted and develops VT or ventricular fibrillation, the ECG should be recorded to note the time between the onset of the dysrhythmia and the onset of the devices shock or antitachycardia pacing. This is a priority over LOC or activity at the time of onset. 37. The staff educator is teaching a CPR class. Which of the following aspects of defibrillation should the educator stress to the class? A) Apply the paddles directly to the patients skin. B) Use a conducting medium between the paddles and the skin. C) Always use a petroleum-based gel between the paddles and the skin. D) Any available liquid can be used between the paddles and the skin. B Feedback: Use multifunction conductor pads or paddles with a conducting medium between the paddles and the skin (the conducting medium is available as a sheet, gel, or paste). Do not use gels or pastes with poor electrical conductivity. 38. During a CPR class, a participant asks about the difference between cardioversion and defibrillation. What would be the instructors best response? A) Cardioversion is done on a beating heart; defibrillation is not. B) The difference is the timing of the delivery of the electric current. C) Defibrillation is synchronized with the electrical activity of the heart, but cardioversion is not. D) Cardioversion is always attempted before defibrillation because it has fewer risks. B Feedback: One major difference between cardioversion and defibrillation is the timing of the delivery of electrical current. In cardioversion, the delivery of the electrical current is synchronized with the patients electrical events; in defibrillation, the delivery of the current is immediate and unsynchronized. Both can be done on beating heart (i.e., in a dysrhythmia). Cardioversion is not necessarily attempted first. 39. A patient is admitted to the cardiac care unit for an electrophysiology (EP) study. What goal should guide the planning and execution of the patients care? A) Ablate the area causing the dysrhythmia. B) Freeze hypersensitive cells. C) Diagnose the dysrhythmia. D) Determine the nursing plan of care. C Feedback: A patient may undergo an EP study in which electrodes are placed inside the heart to obtain an intracardiac ECG. This is used not only to diagnose the dysrhythmia but also to determine the most effective treatment plan. However, because an EP study is invasive, it is performed in the hospital and may require that the patient be admitted. 40. A patient calls his cardiologists office and talks to the nurse. He is concerned because he feels he is being defibrillated too often. The nurse tells the patient to come to the office to be evaluated because the nurse knows that the most frequent complication of ICD therapy is what? A) Infection B) Failure to capture C) Premature battery depletion D) Oversensing of dysrhythmias D Feedback: Inappropriate delivery of ICD therapy, usually due to oversensing of atrial and sinus tachycardias with a rapid ventricular rate response, is the most frequent complication of ICD. Infections, failure to capture, and premature battery failure are less common. Chapter 27: Management of Patients With Coronary Vascular Disorders 1. The nurse is caring for a patient who has been diagnosed with an elevated cholesterol level. The nurse is aware that plaque on the inner lumen of arteries is composed chiefly of what? A) Lipids and fibrous tissue B) White blood cells C) Lipoproteins D) High-density cholesterol A Feedback: As T-lymphocytes and monocytes infiltrate to ingest lipids on the arterial wall and then die, a fibrous tissue develops. This causes plaques to form on the inner lumen of arterial walls. These plaques do not consist of white cells, lipoproteins, or high-density cholesterol. 2. A patient presents to the walk-in clinic complaining of intermittent chest pain on exertion, which is eventually attributed to angina. The nurse should inform the patient that angina is most often attributable to what cause? A) Decreased cardiac output B) Decreased cardiac contractility C) Infarction of the myocardium D) Coronary arteriosclerosis D Feedback: In most cases, angina pectoris is due to arteriosclerosis. The disease is not a result of impaired cardiac output or contractility. Infarction may result from untreated angina, but it is not a cause of the disease. 3. The nurse is caring for an adult patient who had symptoms of unstable angina upon admission to the hospital. What nursing diagnosis underlies the discomfort associated with angina? A) Ineffective breathing pattern related to decreased cardiac output B) Anxiety related to fear of death C) Ineffective cardiopulmonary tissue perfusion related to coronary artery disease (CAD) D) Impaired skin integrity related to CAD C Feedback: Ineffective cardiopulmonary tissue perfusion directly results in the symptoms of discomfort associated with angina. Anxiety and ineffective breathing may result from angina chest pain, but they are not the causes. Skin integrity is not impaired by the effects of angina. 4. The triage nurse in the ED assesses a 66-year-old male patient who presents to the ED with complaints of midsternal chest pain that has lasted for the last 5 hours. If the patients symptoms are due to an MI, what will have happened to the myocardium? A) It may have developed an increased area of infarction during the time without treatment. B) It will probably not have more damage than if he came in immediately. C) It may be responsive to restoration of the area of dead cells with proper treatment. D) It has been irreparably damaged, so immediate treatment is no longer necessary. A Feedback: When the patient experiences lack of oxygen to myocardium cells during an MI, the sooner treatment is initiated, the more likely the treatment will prevent or minimize myocardial tissue necrosis. Delays in treatment equate with increased myocardial damage. Despite the length of time the symptoms have been present, treatment needs to be initiated immediately to minimize further damage. Dead cells cannot be restored by any means. 5. Family members bring a patient to the ED with pale cool skin, sudden midsternal chest pain unrelieved with rest, and a history of CAD. How should the nurse best interpret these initial data? A) The symptoms indicate angina and should be treated as such. B) The symptoms indicate a pulmonary etiology rather than a cardiac etiology. C) The symptoms indicate an acute coronary episode and should be treated as such. D) Treatment should be determined pending the results of an exercise stress test. C Feedback: Angina and MI have similar symptoms and are considered the same process, but are on different points along a continuum. That the patients symptoms are unrelieved by rest suggests an acute coronary episode rather than angina. Pale cool skin and sudden onset are inconsistent with a pulmonary etiology. Treatment should be initiated immediately regardless of diagnosis. 6. An OR nurse is preparing to assist with a coronary artery bypass graft (CABG). The OR nurse knows that the vessel most commonly used as source for a CABG is what? A) Brachial artery B) Brachial vein C) Femoral artery D) Greater saphenous vein D Feedback: The greater saphenous vein is the most commonly used graft site for CABG. The right and left internal mammary arteries, radial arteries, and gastroepiploic artery are other graft sites used, though not as frequently. The femoral artery, brachial artery, and brachial vein are never harvested. 7. A patient with an occluded coronary artery is admitted and has an emergency percutaneous transluminal coronary angioplasty (PTCA). The patient is admitted to the cardiac critical care unit after the PTCA. For what complication should the nurse most closely monitor the patient? A) Hyperlipidemia B) Bleeding at insertion site C) Left ventricular hypertrophy D) Congestive heart failure B Feedback: Complications of PTCA may include bleeding at the insertion site, abrupt closure of the artery, arterial thrombosis, and perforation of the artery. Complications do not include hyperlipidemia, left ventricular hypertrophy, or congestive heart failure; each of these problems takes an extended time to develop and none is emergent. 8. The nurse is caring for a patient who is scheduled for cardiac surgery. What should the nurse include in preoperative care? A) With the patient, clarify the surgical procedure that will be performed. B) Withhold the patients scheduled medications for at least 12 hours preoperatively. C) Inform the patient that health teaching will begin as soon as possible after surgery. D) Avoid discussing the patients fears as not to exacerbate them. A Feedback: Preoperatively, it is necessary to evaluate the patients understanding of the surgical procedure, informed consent, and adherence to treatment protocols. Teaching would begin on admission or even prior to admission. The physician would write orders to alter the patients medication regimen if necessary; this will vary from patient to patient. Fears should be addressed directly and empathically. 9. The OR nurse is explaining to a patient that cardiac surgery requires the absence of blood from the surgical field. At the same time, it is imperative to maintain perfusion of body organs and tissues. What technique for achieving these simultaneous goals should the nurse describe? A) Coronary artery bypass graft (CABG) B) Percutaneous transluminal coronary angioplasty (PTCA) C) Atherectomy D) Cardiopulmonary bypass D Feedback: Cardiopulmonary bypass is often used to circulate and oxygenate blood mechanically while bypassing the heart and lungs. PTCA, atherectomy, and CABG are all surgical procedures, none of which achieves the two goals listed. 10. The nurse has just admitted a 66-year-old patient for cardiac surgery. The patient tearfully admits to the nurse that she is afraid of dying while undergoing the surgery. What is the nurses best response? A) Explore the factors underlying the patients anxiety. B) Teach the patient guided imagery techniques. C) Obtain an order for a PRN benzodiazepine. D) Describe the procedure in greater detail. A Feedback: An assessment of anxiety levels is required in the patient to assist the patient in identifying fears and developing coping mechanisms for those fears. The nurse must further assess and explore the patients anxiety before providing interventions such as education or medications. 11. A patient with angina has been prescribed nitroglycerin. Before administering the drug, the nurse should inform the patient about what potential adverse effects? A) Nervousness or paresthesia B) Throbbing headache or dizziness C) Drowsiness or blurred vision D) Tinnitus or diplopia B Feedback: Headache and dizziness commonly occur when nitroglycerin is taken at the beginning of therapy. Nervousness, paresthesia, drowsiness, blurred vision, tinnitus, and diplopia do not typically occur as a result of nitroglycerin therapy. 12. The nurse is providing an educational workshop about coronary artery disease (CAD) and its risk factors. The nurse explains to participants that CAD has many risk factors, some that can be controlled and some that cannot. What risk factors would the nurse list that can be controlled or modified? A) Gender, obesity, family history, and smoking B) Inactivity, stress, gender, and smoking C) Obesity, inactivity, diet, and smoking D) Stress, family history, and obesity C Feedback: The risk factors for CAD that can be controlled or modified include obesity, inactivity, diet, stress, and smoking. Gender and family history are risk factors that cannot be controlled. 13. A 48-year-old man presents to the ED complaining of severe substernal chest pain radiating down his left arm. He is admitted to the coronary care unit (CCU) with a diagnosis of myocardial infarction (MI). What nursing assessment activity is a priority on admission to the CCU? A) Begin ECG monitoring. B) Obtain information about family history of heart disease. C) Auscultate lung fields. D) Determine if the patient smokes. A Feedback: The 12-lead ECG provides information that assists in ruling out or diagnosing an acute MI. It should be obtained within 10 minutes from the time a patient reports pain or arrives in the ED. By monitoring serial ECG changes over time, the location, evolution, and resolution of an MI can be identified and monitored; life-threatening arrhythmias are the leading cause of death in the first hours after an MI. Obtaining information about family history of heart disease and whether the patient smokes are not immediate priorities in the acute phase of MI. Data may be obtained from family members later. Lung fields are auscultated after oxygenation and pain control needs are met. 14. The public health nurse is participating in a health fair and interviews a patient with a history of hypertension, who is currently smoking one pack of cigarettes per day. She denies any of the most common manifestations of CAD. Based on these data, the nurse would expect the focuses of CAD treatment most likely to be which of the following? A) Drug therapy and smoking cessation B) Diet and drug therapy C) Diet therapy only D) Diet therapy and smoking cessation D Feedback: Due to the absence of symptoms, dietary therapy would likely be selected as the first-line treatment for possible CAD. Drug therapy would be determined based on a number of considerations and diagnostics findings, but would not be directly indicated. Smoking cessation is always indicated, regardless of the presence or absence of symptoms. 15. The nurse is working with a patient who had an MI and is now active in rehabilitation. The nurse should teach this patient to cease activity if which of the following occurs? A) The patient experiences chest pain, palpitations, or dyspnea. B) The patient experiences a noticeable increase in heart rate during activity. C) The patients oxygen saturation level drops below 96%. D) The patients respiratory rate exceeds 30 breaths/min. A Feedback: Any activity or exercise that causes dyspnea and chest pain should be stopped in the patient with CAD. Heart rate must not exceed the target rate, but an increase above resting rate is expected and is therapeutic. In most patients, a respiratory rate that exceeds 30 breaths/min is not problematic. Similarly, oxygen saturation slightly below 96% does not necessitate cessation of activity. 16. A patient with cardiovascular disease is being treated with amlodipine (Norvasc), a calcium channel blocking agent. The therapeutic effects of calcium channel blockers include which of the following? A) Reducing the hearts workload by decreasing heart rate and myocardial contraction B) Preventing platelet aggregation and subsequent thrombosis C) Reducing myocardial oxygen consumption by blocking adrenergic stimulation to the heart D) Increasing the efficiency of myocardial oxygen consumption, thus decreasing ischemia and relieving pain A Feedback: Calcium channel blocking agents decrease sinoatrial node automaticity and atrioventricular node conduction, resulting in a slower heart rate and a decrease in the strength of the heart muscle contraction. These effects decrease the workload of the heart. Antiplatelet and anticoagulation medications are administered to prevent platelet aggregation and subsequent thrombosis, which impedes blood flow. Beta-blockers reduce myocardial consumption by blocking beta-adrenergic sympathetic stimulation to the heart. The result is reduced myocardial contractility (force of contraction) to balance the myocardium oxygen needs and supply. Nitrates reduce myocardial oxygen consumption, which decreases ischemia and relieves pain by dilating the veins and, in higher doses, the arteries. 17. The nurse is providing care for a patient with high cholesterol and triglyceride values. In teaching the patient about therapeutic lifestyle changes such as diet and exercise, the nurse realizes that the desired goal for cholesterol levels is which of the following? A) High HDL values and high triglyceride values B) Absence of detectable total cholesterol levels C) Elevated blood lipids, fasting glucose less than 100 D) Low LDL values and high HDL values D Feedback: The desired goal for cholesterol readings is for a patient to have low LDL and high HDL values. LDL exerts a harmful effect on the coronary vasculature because the small LDL particles can be easily transported into the vessel lining. In contrast, HDL promotes the use of total cholesterol by transporting LDL to the liver, where it is excreted. Elevated triglycerides are also a major risk factor for cardiovascular disease. A goal is also to keep triglyceride levels less than 150 mg/dL. All individuals possess detectable levels of total cholesterol. 18. When discussing angina pectoris secondary to atherosclerotic disease with a patient, the patient asks why he tends to experience chest pain when he exerts himself. The nurse should describe which of the following phenomena? A) Exercise increases the hearts oxygen demands. B) Exercise causes vasoconstriction of the coronary arteries. C) Exercise shunts blood flow from the heart to the mesenteric area. D) Exercise increases the metabolism of cardiac medications. A Feedback: Physical exertion increases the myocardial oxygen demand. If the patient has arteriosclerosis of the coronary arteries, then blood supply is diminished to the myocardium. Exercise does not cause vasoconstriction or interfere with drug metabolism. Exercise does not shunt blood flow away from the heart. 19. The nurse is caring for a patient who is believed to have just experienced an MI. The nurse notes changes in the ECG of the patient. What change on an ECG most strongly suggests to the nurse that ischemia is occurring? A) P wave inversion B) T wave inversion C) Q wave changes with no change in ST or T wave D) P wave enlargement B Feedback: T-wave inversion is an indicator of ischemic damage to myocardium. Typically, few changes to P waves occur during or after an MI, whereas Q-wave changes with no change in the ST or T wave indicate an old MI. 20. An adult patient is admitted to the ED with chest pain. The patient states that he had developed unrelieved chest pain that was present for approximately 20 minutes before coming to the hospital. To minimize cardiac damage, the nurse should expect to administer which of the following interventions? A) Thrombolytics, oxygen administration, and nonsteroidal anti-inflammatories B) Morphine sulphate, oxygen, and bed rest C) Oxygen and beta-adrenergic blockers D) Bed rest, albuterol nebulizer treatments, and oxygen B Feedback: The patient with suspected MI should immediately receive supplemental oxygen, aspirin, nitroglycerin, and morphine. Morphine sulphate reduces preload and decreases workload of the heart, along with increased oxygen from oxygen therapy and bed rest. With decreased cardiac demand, this provides the best chance of decreasing cardiac damage. NSAIDs and beta-blockers are not normally indicated. Albuterol, which is a medication used to manage asthma and respiratory conditions, will increase the heart rate. 21. The nurse is assessing a patient who was admitted to the critical care unit 3 hours ago following cardiac surgery. The nurses most recent assessment reveals that the patients left pedal pulses are not palpable and that the right pedal pulses are rated at +2. What is the nurses best response? A) Document this expected assessment finding during the initial postoperative period. B) Reposition the patient with his left leg in a dependent position. C) Inform the patients physician of this assessment finding. D) Administer an ordered dose of subcutaneous heparin. C Feedback: If a pulse is absent in any extremity, the cause may be prior catheterization of that extremity, chronic peripheral vascular disease, or a thromboembolic obstruction. The nurse immediately reports newly identified absence of any pulse. 22. In preparation for cardiac surgery, a patient was taught about measures to prevent venous thromboembolism. What statement indicates that the patient clearly understood this education? A) Ill try to stay in bed for the first few days to allow myself to heal. B) Ill make sure that I dont cross my legs when Im resting in bed. C) Ill keep pillows under my knees to help my blood circulate better. D) Ill put on those compression stockings if I get pain in my calves. B Feedback: To prevent venous thromboembolism, patients should avoid crossing the legs. Activity is generally begun as soon as possible and pillows should not be placed under the popliteal space. Compression stockings are often used to prevent venous thromboembolism, but they would not be applied when symptoms emerge. 23. An ED nurse is assessing an adult woman for a suspected MI. When planning the assessment, the nurse should be cognizant of what signs and symptoms of MI that are particularly common in female patients? Select all that apply. A) Shortness of breath B) Chest pain C) Anxiety D) Numbness E) Weakness D, E Feedback: Although these symptoms are not wholly absent in men, many women have been found to have atypical symptoms of MI, including indigestion, nausea, palpitations, and numbness. Shortness of breath, chest pain, and anxiety are common symptoms of MI among patients of all ages and genders. 24. When assessing a patient diagnosed with angina pectoris it is most important for the nurse to gather what information? A) The patients activities limitations and level of consciousness after the attacks B) The patients symptoms and the activities that precipitate attacks C) The patients understanding of the pathology of angina D) The patients coping strategies surrounding the attacks B Feedback: The nurse must gather information about the patients symptoms and activities, especially those that precede and precipitate attacks of angina pectoris. The patients coping, understanding of the disease, and status following attacks are all important to know, but causative factors are a primary focus of the assessment interview. 25. You are writing a care plan for a patient who has been diagnosed with angina pectoris. The patient describes herself as being distressed and shocked by her new diagnosis. What nursing diagnosis is most clearly suggested by the womans statement? A) Spiritual distress related to change in health status B) Acute confusion related to prognosis for recovery C) Anxiety related to cardiac symptoms D) Deficient knowledge related to treatment of angina pectoris C Feedback: Although further assessment is warranted, it is not unlikely that the patient is experiencing anxiety. In patients with CAD, this often relates to the threat of sudden death. There is no evidence of confusion (i.e., delirium or dementia) and there may or may not be a spiritual element to her concerns. Similarly, it is not clear that a lack of knowledge or information is the root of her anxiety. 26. The nurse is caring for patient who tells the nurse that he has an angina attack beginning. What is the nurses most appropriate initial action? A) Have the patient sit down and put his head between his knees. B) Have the patient perform pursed-lip breathing. C) Have the patient stand still and bend over at the waist. D) Place the patient on bed rest in a semi-Fowlers position. D Feedback: When a patient experiences angina, the patient is directed to stop all activities and sit or rest in bed in a semi-Fowlers position to reduce the oxygen requirements of the ischemic myocardium. Pursed-lip breathing and standing will not reduce workload to the same extent. No need to have the patient put his head between his legs because cerebral perfusion is not lacking. 27. A patient presents to the ED in distress and complaining of crushing chest pain. What is the nurses priority for assessment? A) Prompt initiation of an ECG B) Auscultation of the patients point of maximal impulse (PMI) C) Rapid assessment of the patients peripheral pulses D) Palpation of the patients cardiac apex A Feedback: The 12-lead ECG provides information that assists in ruling out or diagnosing an acute MI. It should be obtained within 10 minutes from the time a patient reports pain or arrives in the ED. Each of the other listed assessments is valid, but ECG monitoring is the most time dependent priority. 28. The ED nurse is caring for a patient with a suspected MI. What drug should the nurse anticipate administering to this patient? A) Oxycodone B) Warfarin C) Morphine D) Acetaminophen C Feedback: The patient with suspected MI is given aspirin, nitroglycerin, morphine, an IV beta- blocker, and other medications, as indicated, while the diagnosis is being confirmed. Tylenol, warfarin, and oxycodone are not typically used. 29. The nurse is assessing a patient with acute coronary syndrome (ACS). The nurse includes a careful history in the assessment, especially with regard to signs and symptoms. What signs and symptoms are suggestive of ACS? Select all that apply. A) Dyspnea B) Unusual fatigue C) Hypotension D) Syncope E) Peripheral cyanosis A, B, D Feedback: Systematic assessment includes a careful history, particularly as it relates to symptoms: chest pain or discomfort, difficulty breathing (dyspnea), palpitations, unusual fatigue, faintness (syncope), or sweating (diaphoresis). Each symptom must be evaluated with regard to time, duration, and the factors that precipitate the symptom and relieve it, and in comparison with previous symptoms. Hypotension and peripheral cyanosis are not typically associated with ACS. 30. The nurse is creating a plan of care for a patient with acute coronary syndrome. What nursing action should be included in the patients care plan? A) Facilitate daily arterial blood gas (ABG) sampling. B) Administer supplementary oxygen, as needed. C) Have patient maintain supine positioning when in bed. D) Perform chest physiotherapy, as indicated. B Feedback: Oxygen should be administered along with medication therapy to assist with symptom relief. Administration of oxygen raises the circulating level of oxygen to reduce pain associated with low levels of myocardial oxygen. Physical rest in bed with the head of the bed elevated or in a supportive chair helps decrease chest discomfort and dyspnea. ABGs are diagnostic, not therapeutic, and they are rarely needed on a daily basis. Chest physiotherapy is not used in the treatment of ACS. 31. The nurse is participating in the care conference for a patient with ACS. What goal should guide the care teams selection of assessments, interventions, and treatments? A) Maximizing cardiac output while minimizing heart rate B) Decreasing energy expenditure of the myocardium C) Balancing myocardial oxygen supply with demand D) Increasing the size of the myocardial muscle C Feedback: Balancing myocardial oxygen supply with demand (e.g., as evidenced by the relief of chest pain) is the top priority in the care of the patient with ACS. Treatment is not aimed directly at minimizing heart rate because some patients experience bradycardia. Increasing the size of the myocardium is never a goal. Reducing the myocardiums energy expenditure is often beneficial, but this must be balanced with productivity. 32. The nurse working on the coronary care unit is caring for a patient with ACS. How can the nurse best meet the patients psychosocial needs? A) Reinforce the fact that treatment will be successful. B) Facilitate a referral to a chaplain or spiritual leader. C) Increase the patients participation in rehabilitation activities. D) Directly address the patients anxieties and fears. D Feedback: Alleviating anxiety and decreasing fear are important nursing functions that reduce the sympathetic stress response. Referrals to spiritual care may or may not be appropriate, and this does not relieve the nurse of responsibility for addressing the patients psychosocial needs. Treatment is not always successful, and false hope should never be fostered. Participation in rehabilitation may alleviate anxiety for some patients, but it may exacerbate it for others. 33. The nurse is caring for a patient who has undergone percutaneous transluminal coronary angioplasty (PTCA). What is the major indicator of success for this procedure? A) Increase in the size of the arterys lumen B) Decrease in arterial blood flow in relation to venous flow C) Increase in the patients resting heart rate D) Increase in the patients level of consciousness (LOC) A Feedback: PTCA is used to open blocked coronary vessels and resolve ischemia. The procedure may result in beneficial changes to the patients LOC or heart rate, but these are not the overarching goals of PTCA. Increased arterial flow is the focus of the procedures. 34. A nurse has taken on the care of a patient who had a coronary artery stent placed yesterday. When reviewing the patients daily medication administration record, the nurse should anticipate administering what drug? A) Ibuprofen B) Clopidogrel C) Dipyridamole D) Acetaminophen B Feedback: Because of the risk of thrombus formation within the stent, the patient receives antiplatelet medications, usually aspirin and clopidogrel. Ibuprofen and acetaminophen are not antiplatelet drugs. Dipyridamole is not the drug of choice following stent placement. 35. A nurse is working with a patient who has been scheduled for a percutaneous coronary intervention (PCI) later in the week. What anticipatory guidance should the nurse provide to the patient? A) He will remain on bed rest for 48 to 72 hours after the procedure. B) He will be given vitamin K infusions to prevent bleeding following PCI. C) A sheath will be placed over the insertion site after the procedure is finished. D) The procedure will likely be repeated in 6 to 8 weeks to ensure success. C Feedback: A sheath is placed over the PCI access site and kept in place until adequate coagulation is achieved. Patients resume activity a few hours after PCI and repeated treatments may or may not be necessary. Anticoagulants, not vitamin K, are administered during PCI. 36. Preoperative education is an important part of the nursing care of patients having coronary artery revascularization. When explaining the pre- and postoperative regimens, the nurse would be sure to include education about which subject? A) Symptoms of hypovolemia B) Symptoms of low blood pressure C) Complications requiring graft removal D) Intubation and mechanical ventilation D Feedback: Most patients remain intubated and on mechanical ventilation for several hours after surgery. It is important that patients realize that this will prevent them from talking, and the nurse should reassure them that the staff will be able to assist them with other means of communication. Teaching would generally not include symptoms of low blood pressure or hypovolemia, as these are not applicable to most patients. Teaching would also generally not include rare complications that would require graft removal. 37. A patient in the cardiac step-down unit has begun bleeding from the percutaneous coronary intervention (PCI) access site in her femoral region. What is the nurses most appropriate action? A) Call for assistance and initiate cardiopulmonary resuscitation. B) Reposition the patients leg in a nondependent position. C) Promptly remove the femoral sheath. D) Call for help and apply pressure to the access site. D Feedback: The femoral sheath produces pressure on the access site. Pressure will temporarily reduce bleeding and allow for subsequent interventions. Removing the sheath would exacerbate bleeding and repositioning would not halt it. CPR is not indicated unless there is evidence of respiratory or cardiac arrest. 38. The nurse providing care for a patient post PTCA knows to monitor the patient closely. For what complications should the nurse monitor the patient? Select all that apply. A) Abrupt closure of the coronary artery B) Venous insufficiency C) Bleeding at the insertion site D) Retroperitoneal bleeding E) Arterial occlusion A, C, D, E Feedback: Complications after the procedure may include abrupt closure of the coronary artery and vascular complications, such as bleeding at the insertion site, retroperitoneal bleeding, hematoma, and arterial occlusion, as well as acute renal failure. Venous insufficiency is not a postprocedure complication of a PTCA. 39. A patient who is postoperative day 1 following a CABG has produced 20 mL of urine in the past 3 hours and the nurse has confirmed the patency of the urinary catheter. What is the nurses most appropriate action? A) Document the patients low urine output and monitor closely for the next several hours. B) Contact the dietitian and suggest the need for increased oral fluid intake. C) Contact the patients physician and suggest assessment of fluid balance and renal function. D) Increase the infusion rate of the patients IV fluid to prompt an increase in renal function. C Feedback: Nursing management includes accurate measurement of urine output. An output of less than 1 mL/kg/h may indicate hypovolemia or renal insufficiency. Prompt referral is necessary. IV fluid replacement may be indicated, but is beyond the independent scope of the dietitian or nurse. 40. A patient is recovering in the hospital from cardiac surgery. The nurse has identified the diagnosis of risk for ineffective airway clearance related to pulmonary secretions. What intervention best addresses this risk? A) Administration of bronchodilators by nebulizer B) Administration of inhaled corticosteroids by metered dose inhaler (MDI) C) Patients consistent performance of deep breathing and coughing exercises D) Patients active participation in the cardiac rehabilitation program C Feedback: Clearance of pulmonary secretions is accomplished by frequent repositioning of the patient, suctioning, and chest physical therapy, as well as educating and encouraging the patient to breathe deeply and cough. Medications are not normally used to achieve this goal. Rehabilitation is important, but will not necessarily aid the mobilization of respiratory secretions. Chapter 28: Management of Patients With Structural, Infectious, and Inflammatory Cardiac Disorders 1. A patient with mitral stenosis exhibits new symptoms of a dysrhythmia. Based on the pathophysiology of this disease process, the nurse would expect the patient to exhibit what heart rhythm? A) Ventricular fibrillation (VF) B) Ventricular tachycardia (VT) C) Atrial fibrillation D) Sinus bradycardia C Feedback: In patients with mitral valve stenosis, the pulse is weak and often irregular because of atrial fibrillation. Bradycardia, VF, and VT are not characteristic of this valvular disorder. 2. A patient who has undergone a valve replacement with a mechanical valve prosthesis is due to be discharged home. During discharge teaching, the nurse should discuss the importance of antibiotic prophylaxis prior to which of the following? A) Exposure to immunocompromised individuals B) Future hospital admissions C) Dental procedures D) Live vaccinations C Feedback: Following mechanical valve replacement, antibiotic prophylaxis is necessary before dental procedures involving manipulation of gingival tissue, the periapical area of the teeth or perforation of the oral mucosa (not including routine anesthetic injections, placement of orthodontic brackets, or loss of deciduous teeth). There are no current recommendations around antibiotic prophylaxis prior to vaccination, future hospital admissions, or exposure to people who are immunosuppressed. 3. A patient with hypertrophic cardiomyopathy (HCM) has been admitted to the medical unit. During the nurses admission interview, the patient states that she takes over-the-counter water pills on a regular basis. How should the nurse best respond to the fact that the patient has been taking diuretics? A) Encourage the patient to drink at least 2 liters of fluid daily. B) Increase the patients oral sodium intake. C) Inform the care provider because diuretics are contraindicated. D) Ensure that the patients fluid balance is monitored vigilantly. C Feedback: Diuretics are contraindicated in patients with HCM, so the primary care provider should be made aware. Adjusting the patients sodium or fluid intake or fluid monitoring does not address this important contraindication. 4. The critical care nurse is caring for a patient who is receiving cyclosporine postoperative heart transplant. The patient asks the nurse to remind him what this medication is for. How should the nurse best respond? A) Azathioprine decreases the risk of thrombus formation. B) Azathioprine ensures adequate cardiac output. C) Azathioprine increases the number of white blood cells. D) Azathioprine minimizes rejection of the transplant. D Feedback: After heart transplant, patients are constantly balancing the risk of rejection with the risk of infection. Most commonly, patients receive cyclosporine or tacrolimus (FK506, Prograf), azathioprine (Imuran), or mycophenolate mofetil (CellCept), and corticosteroids (prednisone) to minimize rejection. Cyclosporine does not prevent thrombus formation, enhance cardiac output, or increase white cell counts. 5. A patient with a history rheumatic heart disease knows that she is at risk for bacterial endocarditis when undergoing invasive procedures. Prior to a scheduled cystoscopy, the nurse should ensure that the patient knows the importance of taking which of the following drugs? A) Enoxaparin (Lovenox) B) Metoprolol (Lopressor) C) Azathioprine (Imuran) D) Amoxicillin (Amoxil) D Feedback: Although rare, bacterial endocarditis may be life-threatening. A key strategy is primary prevention in high-risk patients (i.e., those with rheumatic heart disease, mitral valve prolapse, or prosthetic heart valves). Antibiotic prophylaxis is recommended for high-risk patients immediately before and sometimes after certain procedures. Amoxicillin is the drug of choice. None of the other listed drugs is an antibiotic. 6. A patient with pericarditis has just been admitted to the CCU. The nurse planning the patients care should prioritize what nursing diagnosis? A) Anxiety related to pericarditis B) Acute pain related to pericarditis C) Ineffective tissue perfusion related to pericarditis D) Ineffective breathing pattern related to pericarditis B Feedback: The most characteristic symptom of pericarditis is chest pain, although pain also may be located beneath the clavicle, in the neck, or in the left trapezius (scapula) region. The pain or discomfort usually remains fairly constant, but it may worsen with deep inspiration and when lying down or turning. Anxiety is highly plausible and should be addressed, but chest pain is a nearly certain accompaniment to the disease. Breathing and tissue perfusion are likely to be at risk, but pain is certain, especially in the early stages of treatment. 7. A patient newly admitted to the telemetry unit is experiencing progressive fatigue, hemoptysis, and dyspnea. Diagnostic testing has revealed that these signs and symptoms are attributable to pulmonary venous hypertension. What valvular disorder should the nurse anticipate being diagnosed in this patient? A) Aortic regurgitation B) Mitral stenosis C) Mitral valve prolapse D) Aortic stenosis B Feedback: The first symptom of mitral stenosis is often dyspnea on exertion as a result of pulmonary venous hypertension. Symptoms usually develop after the valve opening is reduced by one-third to one-half its usual size. Patients are likely to show progressive fatigue as a result of low cardiac output. The enlarged left atrium may create pressure on the left bronchial tree, resulting in a dry cough or wheezing. Patients may expectorate blood (i.e., hemoptysis) or experience palpitations, orthopnea, paroxysmal nocturnal dyspnea (PND), and repeated respiratory infections. Pulmonary venous hypertension is not typically caused by aortic regurgitation, mitral valve prolapse, or aortic stenosis. 8. The nurse is caring for a patient with mitral stenosis who is scheduled for a balloon valvuloplasty. The patient tells the nurse that he is unsure why the surgeon did not opt to replace his damaged valve rather than repairing it. What is an advantage of valvuloplasty that the nurse should cite? A) The procedure can be performed on an outpatient basis in a physicians office. B) Repaired valves tend to function longer than replaced valves. C) The procedure is not associated with a risk for infection. D) Lower doses of antirejection drugs are required than with valve replacement. B Feedback: In general, valves that undergo valvuloplasty function longer than prosthetic valve replacements and patients do not require continuous anticoagulation. Valvuloplasty carries a risk of infection, like all surgical procedures, and it is not performed in a physicians office. Antirejection drugs are unnecessary because foreign tissue is not introduced. 9. The nurse is reviewing the echocardiography results of a patient who has just been diagnosed with dilated cardiomyopathy (DCM). What changes in heart structure characterize DCM? A) Dilated ventricles with atrophy of the ventricles B) Dilated ventricles without hypertrophy of the ventricles C) Dilation and hypertrophy of all four heart chambers D) Dilation of the atria and hypertrophy of the ventricles B Feedback: DCM is characterized by significant dilation of the ventricles without significant concomitant hypertrophy and systolic dysfunction. The ventricles do not atrophy in patients with DCM. 10. A patient has been admitted to the medical unit with signs and symptoms suggestive of endocarditis. The physicians choice of antibiotics would be primarily based on what diagnostic test? A) Echocardiography B) Blood cultures C) Cardiac aspiration D) Complete blood count B Feedback: To help determine the causative organisms and the most effective antibiotic treatment for the patient, blood cultures are taken. A CBC can help establish the degree and stage of infection, but not the causative microorganism. Echocardiography cannot indicate the microorganisms causing the infection. Cardiac aspiration is not a diagnostic test. 11. A community health nurse is presenting an educational event and is addressing several health problems, including rheumatic heart disease. What should the nurse describe as the most effective way to prevent rheumatic heart disease? A) Recognizing and promptly treating streptococcal infections B) Prophylactic use of calcium channel blockers in high-risk populations C) Adhering closely to the recommended child immunization schedule D) Smoking cessation A Feedback: Group A streptococcus can cause rheumatic heart fever, resulting in rheumatic endocarditis. Being aware of signs and symptoms of streptococcal infections, identifying them quickly, and treating them promptly, are the best preventative techniques for rheumatic endocarditis. Smoking cessation, immunizations, and calcium channel blockers will not prevent rheumatic heart disease. 12. A patient with mitral valve prolapse is admitted for a scheduled bronchoscopy to investigate recent hemoptysis. The physician has ordered gentamicin to be taken before the procedure. What is the rationale for this? A) To prevent bacterial endocarditis B) To prevent hospital-acquired pneumonia C) To minimize the need for antibiotic use during the procedure D) To decrease the need for surgical asepsis A Feedback: Antibiotic prophylaxis is recommended for high-risk patients immediately before and sometimes after the following invasive procedures, such as bronchoscopy. Gentamicin would not be given to prevent pneumonia, to avoid antibiotic use during the procedure, or to decrease the need for surgical asepsis. 13. The nurse is admitting a patient with complaints of dyspnea on exertion and fatigue. The patients ECG shows dysrhythmias that are sometimes associated with left ventricular hypertrophy. What diagnostic tool would be most helpful in diagnosing cardiomyopathy? A) Cardiac catheterization B) Arterial blood gases C) Echocardiogram D) Exercise stress test C Feedback: The echocardiogram is one of the most helpful diagnostic tools because the structure and function of the ventricles can be observed easily. The ECG is also important, and can demonstrate dysrhythmias and changes consistent with left ventricular hypertrophy. Cardiac catheterization specifically addresses coronary artery function and arterial blood gases evaluate gas exchange and acid balance. Stress testing is not normally used to differentiate cardiomyopathy from other cardiac pathologies. 14. The nurse is preparing a patient for cardiac surgery. During the procedure, the patients heart will be removed and a donor heart implanted at the vena cava and pulmonary veins. What procedure will this patient undergo? A) Orthotopic transplant B) Xenograft C) Heterotropic transplant D) Homograft A Feedback: Orthotopic transplantation is the most common surgical procedure for cardiac transplantation. The recipients heart is removed, and the donor heart is implanted at the vena cava and pulmonary veins. Some surgeons still prefer to remove the recipients heart, leaving a portion of the recipients atria (with the vena cava and pulmonary veins) in place. Homografts, or allografts (i.e., human valves), are obtained from cadaver tissue donations and are used for aortic and pulmonic valve replacement. Xenografts and heterotropic transplantation are not terms used to describe heart transplantation. 15. A patient is undergoing diagnostic testing for mitral stenosis. What statement by the patient during the nurses interview is most suggestive of this valvular disorder? A) I get chest pain from time to time, but it usually resolves when I rest. B) Sometimes when Im resting, I can feel my heart skip a beat. C) Whenever I do any form of exercise I get terribly short of breath. D) My feet and ankles have gotten terribly puffy the last few weeks. C Feedback: The first symptom of mitral stenosis is often breathing difficulty (dyspnea) on exertion as a result of pulmonary venous hypertension. Patients with mitral stenosis are likely to show progressive fatigue as a result of low cardiac output. Palpitations occur in some patients, but dyspnea is a characteristic early symptom. Peripheral edema and chest pain are atypical. 16. The nurse is caring for a patient who is scheduled to undergo mechanical valve replacement. Patient education should include which of the following? A) Use of patient-controlled analgesia B) Long-term anticoagulant therapy C) Steroid therapy D) Use of IV diuretics B Feedback: Mechanical valves necessitate long-term use of required anticoagulants. Diuretics and steroids are not indicated and patient-controlled analgesia may or may be not be used in the immediate postoperative period. 17. The staff educator is presenting a workshop on valvular disorders. When discussing the pathophysiology of aortic regurgitation the educator points out the need to emphasize that aortic regurgitation causes what? A) Cardiac tamponade B) Left ventricular hypertrophy C) Right-sided heart failure D) Ventricular insufficiency B Feedback: Aortic regurgitation eventually causes left ventricular hypertrophy. In aortic regurgitation, blood from the aorta returns to the left ventricle during diastole in addition to the blood normally delivered by the left atrium. The left ventricle dilates, trying to accommodate the increased volume of blood. Aortic regurgitation does not cause cardiac tamponade, right-sided heart failure, or ventricular insufficiency. 18. The nurse is creating a plan of care for a patient with a cardiomyopathy. What priority goal should underlie most of the assessments and interventions that are selected for this patient? A) Absence of complications B) Adherence to the self-care program C) Improved cardiac output D) Increased activity tolerance C Feedback: The priority nursing diagnosis of a patient with cardiomyopathy would include improved or maintained cardiac output. Regardless of the category and cause, cardiomyopathy may lead to severe heart failure, lethal dysrhythmias, and death. The pathophysiology of all cardiomyopathies is a series of progressive events that culminate in impaired cardiac output. Absence of complications, adherence to the self-care program, and increased activity tolerance should be included in the care plan, but they do not have the priority of improved cardiac output. 19. An older adult patient has been diagnosed with aortic regurgitation. What change in blood flow should the nurse expect to see on this patients echocardiogram? A) Blood to flow back from the aorta to the left ventricle B) Obstruction of blood flow from the left ventricle C) Blood to flow back from the left atrium to the left ventricle D) Obstruction of blood from the left atrium to left ventricle A Feedback: Aortic regurgitation occurs when the aortic valve does not completely close, and blood flows back to the left ventricle from the aorta during diastole. Aortic regurgitation does not cause obstruction of blood flow from the left ventricle, blood to flow back from the left atrium to the left ventricle, or obstruction of blood from the left atrium to left ventricle. 20. A patient who has undergone valve replacement surgery is being prepared for discharge home. Because the patient will be discharged with a prescription for warfarin (Coumadin), the nurse should educate the patient about which of the following? A) The need for regularly scheduled testing of the patients International Normalized Ratio (INR) B) The need to learn to sleep in a semi-Fowlers position for the first 6 to 8 weeks to prevent emboli C) The need to avoid foods that contain vitamin K D) The need to take enteric-coated ASA on a daily basis A Feedback: Patients who take warfarin (Coumadin) after valve replacement have individualized target INRs; usually between 2 and 3.5 for mitral valve replacement and 1.8 and 2.2 for aortic valve replacement. Natural sources of vitamin K do not normally need to be avoided and ASA is not indicated. Sleeping upright is unnecessary. 21. A nurse is planning discharge health education for a patient who will soon undergo placement of a mechanical valve prosthesis. What aspect of health education should the nurse prioritize in anticipation of discharge? A) The need for long-term antibiotics B) The need for 7 to 10 days of bed rest C) Strategies for preventing atherosclerosis D) Strategies for infection prevention D Feedback: Patients with a mechanical valve prosthesis (including annuloplasty rings and other prosthetic materials used in valvuloplasty) require education to prevent infective endocarditis. Despite these infections risks, antibiotics are not used long term. Activity management is important, but extended bed rest is unnecessary. Valve replacement does not create a heightened risk for atherosclerosis. 22. A patient with mitral valve stenosis is receiving health education at an outpatient clinic. To minimize the patients symptoms, the nurse should teach the patient to do which of the following? A) Eat a high-protein, low-carbohydrate diet. B) Avoid activities that cause an increased heart rate. C) Avoid large crowds and public events. D) Perform deep breathing and coughing exercises. B Feedback: Patients with mitral stenosis are advised to avoid strenuous activities, competitive sports, and pregnancy, all of which increase heart rate. Infection prevention is important, but avoiding crowds is not usually necessary. Deep breathing and coughing are not likely to prevent exacerbations of symptoms and increased protein intake is not necessary. 23. A patient is admitted to the critical care unit (CCU) with a diagnosis of cardiomyopathy. When reviewing the patients most recent laboratory results, the nurse should prioritize assessment of which of the following? A) Sodium B) AST, ALT, and bilirubin C) White blood cell differential D) BUN A Feedback: Sodium is the major electrolyte involved with cardiomyopathy. Cardiomyopathy often leads to heart failure which develops, in part, from fluid overload. Fluid overload is often associated with elevated sodium levels. Consequently, sodium levels are followed more closely than other important laboratory values, including BUN, leukocytes, and liver function tests. 24. A patient has been admitted with an aortic valve stenosis and has been scheduled for a balloon valvuloplasty in the cardiac catheterization lab later today. During the admission assessment, the patient tells the nurse he has thoracolumbar scoliosis and is concerned about lying down for any extended period of time. What is a priority action for the nurse? A) Arrange for an alternative bed. B) Measure the degree of the curvature. C) Notify the surgeon immediately. D) Note the scoliosis on the intake assessment. C Feedback: Most often used for mitral and aortic valve stenosis, balloon valvuloplasty is contraindicated for patients with left atrial or ventricular thrombus, severe aortic root dilation, significant mitral valve regurgitation, thoracolumbar scoliosis, rotation of the great vessels, and other cardiac conditions that require open heart surgery. Therefore notifying the physician would be the priority over further physical assessment. An alternative bed would be unnecessary and documentation is not a sufficient response. 25. A patient is a candidate for percutaneous balloon valvuloplasty, but is concerned about how this procedure will affect her busy work schedule. What guidance should the nurse provide to the patient? A) Patients generally stay in the hospital for 6 to 8 days. B) Patients are kept in the hospital until they are independent with all aspects of their care. C) Patients need to stay in the hospital until they regain normal heart function for their age. D) Patients usually remain at the hospital for 24 to 48 hours. D Feedback: After undergoing percutaneous balloon valvuloplasty, the patient usually remains in the hospital for 24 to 48 hours. Prediagnosis levels of heart function are not always attainable and the patient does not need to be wholly independent prior to discharge. 26. A patient has been diagnosed with a valvular disorder. The patient tells the nurse that he has read about numerous treatment options, including valvuloplasty. What should the nurse teach the patient about valvuloplasty? A) For some patients, valvuloplasty can be done in a cardiac catheterization laboratory. B) Valvuloplasty is a dangerous procedure, but it has excellent potential if it goes well. C) Valvuloplasty is open heart surgery, but this is very safe these days and normally requires only an overnight hospital stay. D) Its prudent to get a second opinion before deciding to have valvuloplasty. A Feedback: Some valvuloplasty procedures do not require general anesthesia or cardiopulmonary bypass and can be performed in a cardiac catheterization laboratory or hybrid room. Open heart surgery is not required and the procedure does not carry exceptional risks that would designate it as being dangerous. Normally there is no need for the nurse to advocate for a second opinion. The patient has just returned to the floor after balloon valvuloplasty of the aortic valve and the nurse is planning appropriate assessments. The nurse should know that complications following this procedure include what? Select all that apply. A) Emboli B) Mitral valve damage C) Ventricular dysrhythmia D) Atrial-septal defect E) Plaque formation A, B, C Feedback: Possible complications include aortic regurgitation, emboli, ventricular perforation, rupture of the aortic valve annulus, ventricular dysrhythmia, mitral valve damage, and bleeding from the catheter insertion sites. Atrial-septal defect and plaque formation are not complications of a balloon valvuloplasty. 28. The nurse is caring for a patient with right ventricular hypertrophy and consequently decreased right ventricular function. What valvular disorder may have contributed to this patients diagnosis? A) Mitral valve regurgitation B) Aortic stenosis C) Aortic regurgitation D) Mitral valve stenosis D Feedback: Because no valve protects the pulmonary veins from the backward flow of blood from the atrium, the pulmonary circulation becomes congested. As a result, the right ventricle must contract against an abnormally high pulmonary arterial pressure and is subjected to excessive strain. Eventually, the right ventricle fails. None of the other listed valvular disorders has this pathophysiological effect. 29. The cardiac nurse is caring for a patient who has been diagnosed with dilated cardiomyopathy (DCM). Echocardiography is likely to reveal what pathophysiological finding? A) Decreased ejection fraction B) Decreased heart rate C) Ventricular hypertrophy D) Mitral valve regurgitation A Feedback: DCM is distinguished by significant dilation of the ventricles without simultaneous hypertrophy. The ventricles have elevated systolic and diastolic volumes, but a decreased ejection fraction. Bradycardia and mitral valve regurgitation do not typically occur in patients with DCM. 30. A 17-year-old boy is being treated in the ICU after going into cardiac arrest during a football practice. Diagnostic testing reveals cardiomyopathy as the cause of the arrest. What type of cardiomyopathy is particularly common among young people who appear otherwise healthy? A) Dilated cardiomyopathy (DCM). B) Arrhythmogenic right ventricular cardiomyopathy (ARVC) C) Hypertrophic cardiomyopathy (HCM) D) Restrictive or constrictive cardiomyopathy (RCM) C Feedback: With HCM, cardiac arrest (i.e., sudden cardiac death) may be the initial manifestation in young people, including athletes. DCM, ARVC, and RCM are not typically present in younger adults who appear otherwise healthy. 31. The nurse is teaching a patient diagnosed with aortic stenosis appropriate strategies for attempting to relieve the symptom of angina without drugs. What should the nurse teach the patient? A) To eat a small meal before taking nitroglycerin B) To drink a glass of milk before taking nitroglycerin C) To engage in 15 minutes of light exercise before taking nitroglycerin D) To rest and relax before taking nitroglycerin D Feedback: The venous dilation that results from nitroglycerin decreases blood return to the heart, thus decreasing cardiac output and increasing the risk of syncope and decreased coronary artery blood flow. The nurse teaches the patient about the importance of attempting to relieve the symptoms of angina with rest and relaxation before taking nitroglycerin and to anticipate the potential adverse effects. Exercising, eating, and drinking are not recommended prior to using nitroglycerin. 32. A patient has been living with dilated cardiomyopathy for several years but has experienced worsening symptoms despite aggressive medical management. The nurse should anticipate what potential treatment? A) Heart transplantation B) Balloon valvuloplasty C) Cardiac catheterization D) Stent placement A Feedback: When heart failure progresses and medical treatment is no longer effective, surgical intervention, including heart transplantation, is considered. Valvuloplasty, stent placement, and cardiac catheterization will not address the pathophysiology of cardiomyopathy. 33. A patient has undergone a successful heart transplant and has been discharged home with a medication regimen that includes cyclosporine and tacrolimus. In light of this patients medication regimen, what nursing diagnosis should be prioritized? A) Risk for injury B) Risk for infection C) Risk for peripheral neurovascular dysfunction D) Risk for unstable blood glucose B Feedback: Immunosuppressants decrease the bodys ability to resist infections, and a satisfactory balance must be achieved between suppressing rejection and avoiding infection. These drugs do not create a heightened risk of injury, neurovascular dysfunction, or unstable blood glucose levels. 34. The nurse is caring for a patient with acute pericarditis. What nursing management should be instituted to minimize complications? A) The nurse keeps the patient isolated to prevent nosocomial infections. B) The nurse encourages coughing and deep breathing. C) The nurse helps the patient with activities until the pain and fever subside. D) The nurse encourages increased fluid intake until the infection resolves. C Feedback: To minimize complications, the nurse helps the patient with activity restrictions until the pain and fever subside. As the patients condition improves, the nurse encourages gradual increases of activity. Actions to minimize complications of acute pericarditis do not include keeping the patient isolated. Due to pain, coughing and deep breathing are not normally encouraged. An increase in fluid intake is not always necessary. 35. A patient who has recently recovered from a systemic viral infection is undergoing diagnostic testing for myocarditis. Which of the nurses assessment findings is most consistent with myocarditis? A) Sudden changes in level of consciousness (LOC) B) Peripheral edema and pulmonary edema C) Pleuritic chest pain D) Flulike symptoms D Feedback: The most common symptoms of myocarditis are flulike. Chest pain, edema, and changes in LOC are not characteristic of myocarditis. 36. The nurse on the hospitals infection control committee is looking into two cases of hospital-acquired infective endocarditis among a specific classification of patients. What classification of patients would be at greatest risk for hospital-acquired endocarditis? A) Hemodialysis patients B) Patients on immunoglobulins C) Patients who undergo intermittent urinary catheterization D) Children under the age of 12 A Feedback: Hospital-acquired infective endocarditis occurs most often in patients with debilitating disease or indwelling catheters and in patients who are receiving hemodialysis or prolonged IV fluid or antibiotic therapy. Patients taking immunosuppressive medications or corticosteroids are more susceptible to fungal endocarditis. Patients on immunoglobulins, those who need in and out catheterization, and children are not at increased risk for nosocomial infective endocarditis. 37. The nurse is caring for a recent immigrant who has been diagnosed with mitral valve regurgitation. The nurse should know that in developing countries the most common cause of mitral valve regurgitation is what? A) A decrease in gamma globulins B) An insect bite C) Rheumatic heart disease and its sequelae D) Sepsis and its sequelae C Feedback: The most common cause of mitral valve regurgitation in developing countries is rheumatic heart disease and its sequelae. Most individuals who have mitral valve prolapse never have any symptoms, although this is not the case for every patient. What symptoms might a patient have with mitral valve prolapse? Select all that apply. A) Anxiety B) Fatigue C) Shoulder pain D) Tachypnea E) Palpitations A, B, E Feedback: Most people who have mitral valve prolapse never have symptoms. A few have symptoms of fatigue, shortness of breath, lightheadedness, dizziness, syncope, palpitations, chest pain, and anxiety. Hyperpnea and shoulder pain are not characteristic symptoms of mitral valve prolapse. 39. A cardiac surgery patients new onset of signs and symptoms is suggestive of cardiac tamponade. As a member of the interdisciplinary team, what is the nurses most appropriate action? A) Prepare to assist with pericardiocentesis. B) Reposition the patient into a prone position. C) Administer a dose of metoprolol. D) Administer a bolus of normal saline. A Feedback: Cardiac tamponade requires immediate pericardiocentesis. Beta-blockers and fluid boluses will not relieve the pressure on the heart and prone positioning would likely exacerbate symptoms. 40. The nurse is auscultating the breath sounds of a patient with pericarditis. What finding is most consistent with this diagnosis? A) Wheezes B) Friction rub C) Fine crackles D) Coarse crackles B Feedback: A pericardial friction rub is diagnostic of pericarditis. Crackles are associated with pulmonary edema and fluid accumulation, whereas wheezes signal airway constriction; neither of these occurs with pericarditis. Chapter 29: Management of Patients With Complications from Heart Disease 1. The nurse notes that a patient has developed a cough productive for mucoid sputum, is short of breath, has cyanotic hands, and has noisy, moist-sounding, rapid breathing. These symptoms and signs are suggestive of what health problem? A) Pericarditis B) Cardiomyopathy C) Pulmonary edema D) Right ventricular hypertrophy C Feedback: As a result of decreased cerebral oxygenation, the patient with pulmonary edema becomes increasingly restless and anxious. Along with a sudden onset of breathlessness and a sense of suffocation, the patients hands become cold and moist, the nail beds become cyanotic (bluish), and the skin turns ashen (gray). The pulse is weak and rapid, and the neck veins are distended. Incessant coughing may occur, producing increasing quantities of foamy sputum. Pericarditis, ventricular hypertrophy, and cardiomyopathy do not involve wet breath sounds or mucus production. 2. The nurse is assessing an older adult patient with numerous health problems. What assessment datum indicates an increase in the patients risk for heart failure (HF)? A) The patient takes Lasix (furosemide) 20 mg/day. B) The patients potassium level is 4.7 mEq/L. C) The patient is an African American man. D) The patients age is greater than 65. D Feedback: HF is the most common reason for hospitalization of people older than 65 years of age and is the second most common reason for visits to a physicians office. A potassium level of 4.7 mEq/L is within reference range and does not indicate an increased risk for HF. The fact that the patient takes Lasix 20 mg/day does not indicate an increased risk for HF, although this drug is often used in the treatment of HF. The patient being an African American man does not indicate an increased risk for HF. 3. The triage nurse in the ED is assessing a patient with chronic HF who has presented with worsening symptoms. In reviewing the patients medical history, what is a potential primary cause of the patients heart failure? A) Endocarditis B) Pleural effusion C) Atherosclerosis D) Atrial-septal defect C Feedback: Atherosclerosis of the coronary arteries is the primary cause of HF. Pleural effusion, endocarditis, and an atrial-septal defect are not health problems that contribute to the etiology of HF. 4. Which assessment would be most appropriate for a patient who is receiving a loop diuretic for HF? A) Monitor liver function studies B) Monitor for hypotension C) Assess the patients vitamin D intake D) Assess the patient for hyperkalemia B Feedback: Diuretic therapy increases urine output and decreases blood volume, which places the patient at risk of hypotension. Patients are at risk of losing potassium with loop diuretic therapy and need to continue with potassium in their diet; hypokalemia is a consequent risk. Liver function is rarely compromised by diuretic therapy and vitamin D intake is not relevant. 5. The nurse is assessing a patient who is known to have right-sided HF. What assessment finding is most consistent with this patients diagnosis? A) Pulmonary edema B) Distended neck veins C) Dry cough D) Orthopnea B Feedback: Right-sided HF may manifest by distended neck veins, dependent edema, hepatomegaly, weight gain, ascites, anorexia, nausea, nocturia, and weakness. The other answers do not apply. 6. The nurse is caring for an adult patient with HF who is prescribed digoxin. When assessing the patient for adverse effects, the nurse should assess for which of the following signs and symptoms? A) Confusion and bradycardia B) Uncontrolled diuresis and tachycardia C) Numbness and tingling in the extremities D) Chest pain and shortness of breath A Feedback: A key concern associated with digitalis therapy is digitalis toxicity. Symptoms include anorexia, nausea, visual disturbances, confusion, and bradycardia. The other listed signs and symptoms are not characteristic of digitalis toxicity. 7. A nurse in the CCU is caring for a patient with HF who has developed an intracardiac thrombus. This creates a high risk for what sequela? A) Stroke B) Myocardial infarction (MI) C) Hemorrhage D) Peripheral edema A Feedback: Intracardiac thrombi can become lodged in the cerebral vasculature, causing stroke. There is no direct risk of MI, hemorrhage, or peripheral edema. 8. The nurse is caring for a 68-year-old patient the nurse suspects has digoxin toxicity. In addition to physical assessment, the nurse should collect what assessment datum? A) Skin turgor B) Potassium level C) White blood cell count D) Peripheral pulses B Feedback: The serum potassium level is monitored because the effect of digoxin is enhanced in the presence of hypokalemia and digoxin toxicity may occur. Skin turgor, white cell levels, and peripheral pulses are not normally affected in cases of digitalis toxicity. 9. The triage nurse in the ED is performing a rapid assessment of a man with complaints of severe chest pain and shortness of breath. The patient is diaphoretic, pale, and weak. When the patient collapses, what should the nurse do first? A) Check for a carotid pulse. B) Apply supplemental oxygen. C) Give two full breaths. D) Gently shake and shout, Are you OK? D Feedback: Assessing responsiveness is the first step in basic life support. Opening the airway and checking for respirations should occur next. If breathing is absent, two breaths should be given, usually accompanied by supplementary oxygen. Circulation is checked by palpating the carotid artery. 10. A patient presents to the ED complaining of increasing shortness of breath. The nurse assessing the patient notes a history of left-sided HF. The patient is agitated and occasionally coughing up pink-tinged, foamy sputum. The nurse should recognize the signs and symptoms of what health problem? A) Right-sided heart failure B) Acute pulmonary edema C) Pneumonia D) Cardiogenic shock B Feedback: Because of decreased contractility and increased fluid volume and pressure in patients with HF, fluid may be driven from the pulmonary capillary beds into the alveoli, causing pulmonary edema and signs and symptoms described. In right-sided heart failure, the patient exhibits hepatomegaly, jugular vein distention, and peripheral edema. In pneumonia, the patient would have a temperature spike, and sputum that varies in color. Cardiogenic shock would show signs of hypotension and tachycardia. 11. A patient admitted to the medical unit with HF is exhibiting signs and symptoms of pulmonary edema. The nurse is aware that positioning will promote circulation. How should the nurse best position the patient? A) In a high Fowlers position B) On the left side-lying position C) In a flat, supine position D) In the Trendelenburg position A Feedback: Proper positioning can help reduce venous return to the heart. The patient is positioned upright. If the patient is unable to sit with the lower extremities dependent, the patient may be placed in an upright position in bed. The supine position and Trendelenburg positions will not reduce venous return, lower the output of the right ventricle, or decrease lung congestion. Similarly, side-lying does not promote circulation. 12. The nurse has entered a patients room and found the patient unresponsive and not breathing. What is the nurses next appropriate action? A) Palpate the patients carotid pulse. B) Illuminate the patients call light. C) Begin performing chest compressions. D) Activate the Emergency Response System (ERS). D Feedback: After checking for responsiveness and breathing, the nurse should activate the ERS. Assessment of carotid pulse should follow and chest compressions may be indicated. Illuminating the call light is an insufficient response. 13. The nurse is providing discharge education to a patient diagnosed with HF. What should the nurse teach this patient to do to assess her fluid balance in the home setting? A) Monitor her blood pressure daily B) Assess her radial pulses daily C) Monitor her weight daily D) Monitor her bowel movements C Feedback: To assess fluid balance at home, the patient should monitor daily weights at the same time every day. Assessing radial pulses and monitoring the blood pressure may be done, but these measurements do not provide information about fluid balance. Bowel function is not indicative of fluid balance. 14. The nurse is caring for an 84-year-old man who has just returned from the OR after inguinal hernia repair. The OR report indicates that the patient received large volumes of IV fluids during surgery and the nurse recognizes that the patient is at risk for left-sided heart failure. What signs and symptoms would indicate left-sided heart failure? A) Jugular vein distention B) Right upper quadrant pain C) Bibasilar fine crackles D) Dependent edema C Feedback: Bibasilar fine crackles are a sign of alveolar fluid, a sequela of left ventricular fluid, or pressure overload. Jugular vein distention, right upper quadrant pain (hepatomegaly), and dependent edema are caused by right-sided heart failure, usually a chronic condition. 15. A patient with HF is placed on a low-sodium diet. Which statement by the patient indicates that the nurses nutritional teaching plan has been effective? A) I will have a ham and cheese sandwich for lunch. B) I will have a baked potato with broiled chicken for dinner. C) I will have a tossed salad with cheese and croutons for lunch. D) I will have chicken noodle soup with crackers and an apple for lunch. B Feedback: The patients choice of a baked potato with broiled chicken indicates that the teaching plan has been effective. Potatoes and chicken are relatively low in sodium. Ham, cheese, and soup are often high in sodium. 16. The nurses comprehensive assessment of a patient who has HF includes evaluation of the patients hepatojugular reflux. What action should the nurse perform during this assessment? A) Elevate the patients head to 90 degrees. B) Press the right upper abdomen. C) Press above the patients symphysis pubis. D) Lay the patient flat in bed. B Feedback: Hepatojugular reflux, a sign of right-sided heart failure, is assessed with the head of the bed at a 45- degree angle. As the right upper abdomen (the area over the liver) is compressed for 30 to 40 seconds, the nurse observes the internal jugular vein. If the internal jugular vein becomes distended, a patient has positive hepatojugular reflux. 17. The nurse overseeing care in the ICU reviews the shift report on four patients. The nurse recognizes which patient to be at greatest risk for the development of cardiogenic shock? A) The patient admitted with acute renal failure B) The patient admitted following an MI C) The patient admitted with malignant hypertension D) The patient admitted following a stroke B Feedback: Cardiogenic shock may occur following an MI when a large area of the myocardium becomes ischemic, necrotic, and hypokinetic. It also can occur as a result of end-stage heart failure, cardiac tamponade, pulmonary embolism, cardiomyopathy, and dysrhythmias. While patients with acute renal failure are at risk for dysrhythmias and patients experiencing a stroke are at risk for thrombus formation, the patient admitted following an MI is at the greatest risk for development of cardiogenic shock when compared with the other listed diagnoses. 18. When assessing the patient with pericardial effusion, the nurse will assess for pulsus paradoxus. Pulsus paradoxus is characterized by what assessment finding? A) A diastolic blood pressure that is lower during exhalation B) A diastolic blood pressure that is higher during inhalation C) A systolic blood pressure that is higher during exhalation D) A systolic blood pressure that is lower during inhalation D Feedback: Systolic blood pressure that is markedly lower during inhalation is called pulsus paradoxus. The difference in systolic pressure between the point that is heard during exhalation and the point that is heard during inhalation is measured. Pulsus paradoxus exceeding 10 mm Hg is abnormal. 19. The cardiac monitor alarm alerts the critical care nurse that the patient is showing no cardiac rhythm on the monitor. The nurses rapid assessment suggests cardiac arrest. In providing cardiac resuscitation documentation, how will the nurse describe this initial absence of cardiac rhythm? A) Pulseless electrical activity (PEA) B) Ventricular fibrillation C) Ventricular tachycardia D) Asystole D Feedback: Cardiac arrest occurs when the heart ceases to produce an effective pulse and circulate blood. It may be caused by a cardiac electrical event such as ventricular fibrillation, ventricular tachycardia, profound bradycardia, or when there is no heart rhythm at all (asystole). Cardiac arrest may also occur when electrical activity is present, but there is ineffective cardiac contraction or circulating volume, which is PEA. Asystole is the only condition that involves the absolute absence of a heart rhythm. 20. The nurse is reviewing a newly admitted patients electronic health record, which notes a history of orthopnea? What nursing action is most clearly indicated? A) Teach the patient deep breathing and coughing exercises. B) Administer supplemental oxygen at all times. C) Limit the patients activity level. D) Avoid positioning the patient supine. D Feedback: Orthopnea is defined as difficulty breathing while lying flat. This is a possible complication of HF and, consequently, the nurse should avoid positioning the patient supine. Oxygen supplementation may or may not be necessary and activity does not always need to be curtailed. Deep breathing and coughing exercises do not directly address this symptom. 21. The nurse is planning the care of a patient with HF. The nurse should identify what overall goals of this patients care? A) Improve functional status B) Prevent endocarditis. C) Extend survival. D) Limit physical activity. E) Relieve patient symptoms. A, C, E Feedback: The overall goals of management of HF are to relieve the patients symptoms, to improve functional status and quality of life, and to extend survival. Activity limitations should be accommodated, but reducing activity is not a goal. Endocarditis is not a common complication of HF and preventing it is not a major goal of care. 22. A patient with HF has met with his primary care provider and begun treatment with an angiotensin- converting enzyme (ACE) inhibitor. When the patient begins treatment, the nurse should prioritize what assessment? A) Blood pressure B) Level of consciousness (LOC) C) Assessment for nausea D) Oxygen saturation A Feedback: Patients receiving ACE inhibitors are monitored for hypotension, hyperkalemia (increased potassium in the blood), and alterations in renal function. ACE inhibitors do not typically cause alterations in LOC. Oxygen saturation must be monitored in patients with HF, but this is not particular to ACE inhibitor therapy. ACE inhibitors do not normally cause nausea. 23. The nurse is reviewing the medication administration record of a patient diagnosed with systolic HF. What medication should the nurse anticipate administering to this patient? A) A beta-adrenergic blocker B) An antiplatelet aggregator C) A calcium channel blocker D) A nonsteroidal anti-inflammatory drug (NSAID) A Feedback: Several medications are routinely prescribed for systolic HF, including ACE inhibitors, beta-blockers, diuretics, and digitalis. Calcium channel blockers, antiplatelet aggregators, and NSAIDs are not commonly prescribed. 24. The nurse is caring for a patient with systolic HF whose previous adverse reactions preclude the safe use of ACE inhibitors. The nurse should anticipate that the prescriber may choose what combination of drugs? A) Loop diuretic and antiplatelet aggregator B) Loop diuretic and calcium channel blocker C) Combination of hydralazine and isosorbide dinitrate D) Combination of digoxin and normal saline C Feedback: A combination of hydralazine and isosorbide dinitrate may be an alternative for patients who cannot take ACE inhibitors. Antiplatelet aggregators, calcium channel blockers, and normal saline are not typically prescribed. 25. A patient with a diagnosis of HF is started on a beta-blocker. What is the nurses priority role during gradual increases in the patients dose? A) Educating the patient that symptom relief may not occur for several weeks B) Stressing that symptom relief may take up to 4 months to occur C) Making adjustments to each days dose based on the blood pressure trends D) Educating the patient about the potential changes in LOC that may result from the drug A Feedback: An important nursing role during titration is educating the patient about the potential worsening of symptoms during the early phase of treatment and stressing that improvement may take several weeks. Relief does not take 4 months, however. The nurse monitors blood pressure, but changes are not made based on short-term assessment results. Beta-blockers rarely affect LOC. 26. The nurse is performing a physical assessment on a patient suspected of having HF. The presence of what sound would signal the possibility of impending HF? A) An S3 heart sound B) Pleural friction rub C) Faint breath sounds D) A heart murmur A Feedback: The heart is auscultated for an S3 heart sound, a sign that the heart is beginning to fail and that increased blood volume fills the ventricle with each beat. HF does not normally cause a pleural friction rub or murmurs. Changes in breath sounds occur, such as the emergence of crackles or wheezes, but faint breath sounds are less characteristic of HF. 27. An older adult patient with HF is being discharged home on an ACE inhibitor and a loop diuretic. The patients most recent vital signs prior to discharge include oxygen saturation of 93% on room air, heart rate of 81 beats per minute, and blood pressure of 94/59 mm Hg. When planning this patients subsequent care, what nursing diagnosis should be identified? A) Risk for ineffective tissue perfusion related to dysrhythmia B) Risk for fluid volume excess related to medication regimen C) Risk for ineffective breathing pattern related to hypoxia D) Risk for falls related to hypotension D Feedback: The combination of low BP, diuretic use, and ACE inhibitor use constitute a risk for falls. There is no evidence, or heightened risk, of dysrhythmia. The patients medications create a risk for fluid deficit, not fluid excess. Hypoxia is a risk for all patients with HF, but this is not in evidence for this patient at this time. 28. The nurse is performing an initial assessment of a client diagnosed with HF. The nurse also assesses the patients sensorium and LOC. Why is the assessment of the patients sensorium and LOC important in patients with HF? A) HF ultimately affects oxygen transportation to the brain. B) Patients with HF are susceptible to overstimulation of the sympathetic nervous system. C) Decreased LOC causes an exacerbation of the signs and symptoms of HF. D) The most significant adverse effect of medications used for HF treatment is altered LOC. A Feedback: As the volume of blood ejected by the heart decreases, so does the amount of oxygen transported to the brain. Sympathetic stimulation is not a primary concern in patients with HF, although it is a possibility. HF affects LOC but the reverse is not usually true. Medications used to treat HF carry many adverse effects, but the most common and significant effects are cardiovascular. 29. Cardiopulmonary resuscitation has been initiated on a patient who was found unresponsive. When performing chest compressions, the nurse should do which of the following? A) Perform at least 100 chest compressions per minute. B) Pause to allow a colleague to provide a breath every 10 compressions. C) Pause chest compressions to allow for vital signs monitoring every 4 to 5 minutes. D) Perform high-quality chest compressions as rapidly as possible. A Feedback: During CPR, the chest is compressed 2 inches at a rate of at least 100 compressions per minute. This rate is the resuscitators goal; the aim is not to give compressions as rapidly as possible. Compressions are not stopped after 10 compressions to allow for a breath or for full vital signs monitoring. 30. The nurse is providing patient education prior to a patients discharge home after treatment for HF. The nurse gives the patient a home care checklist as part of the discharge teaching. What should be included on this checklist? A) Know how to recognize and prevent orthostatic hypotension. B) Weigh yourself weekly at a consistent time of day. C) Measure everything you eat and drink until otherwise instructed. D) Limit physical activity to only those tasks that are absolutely necessary. A Feedback: Patients with HF should be aware of the risks of orthostatic hypotension. Weight should be measured daily; detailed documentation of all forms of intake is not usually required. Activity should be gradually increased within the parameters of safety and comfort. 31. The nurse is educating an 80-year-old patient diagnosed with HF about his medication regimen. What should the nurse to teach this patient about the use of oral diuretics? A) Avoid drinking fluids for 2 hours after taking the diuretic. B) Take the diuretic in the morning to avoid interfering with sleep. C) Avoid taking the medication within 2 hours consuming dairy products. D) Take the diuretic only on days when experiencing shortness of breath. B Feedback: Oral diuretics should be administered early in the morning so that diuresis does not interfere with the patients nighttime rest. Discussing the timing of medication administration is especially important for elderly patients who may have urinary urgency or incontinence. The nurse would not teach the patient about the timing of fluid intake. Fluid intake does not need to be adjusted and dairy products are not contraindicated. 32. The nurse is addressing exercise and physical activity during discharge education with a patient diagnosed with HF. What should the nurse teach this patient about exercise? A) Do not exercise unsupervised. B) Eventually aim to work up to 30 minutes of exercise each day. C) Slow down if you get dizzy or short of breath. D) Start your exercise program with high-impact activities. B Feedback: Eventually, a total of 30 minutes of physical activity every day should be encouraged. Supervision is not necessarily required and the emergence of symptoms should prompt the patient to stop exercising, not simply to slow the pace. Low-impact activities should be prioritized. 33. The nurse is creating a care plan for a patient diagnosed with HF. When addressing the problem of anxiety, what interventions should the nurse include in the care plan? Select all that apply. A) Facilitate the presence of friends and family whenever possible. B) Teach the patient about the harmful effects of anxiety on cardiac function. C) Provide supplemental oxygen, as needed. D) Provide validation of the patients expressions of anxiety. E) Administer benzodiazepines two to three times daily. A, C, D Feedback: The nurse should empathically validate the patients sensations of anxiety. The presence of friends and family are frequently beneficial and oxygen supplementation promotes comfort. Antianxiety medications may be necessary for some patients, but alternative methods of relief should be prioritized. As well, medications are administered on a PRN basis. Teaching the patient about the potential harms of anxiety is likely to exacerbate, not relieve, the problem. 34. The critical care nurse is caring for a patient who is in cardiogenic shock. What assessments must the nurse perform on this patient? Select all that apply. A) Platelet level B) Fluid status C) Cardiac rhythm D) Action of medications E) Sputum volume B, C, D Feedback: The critical care nurse must carefully assess the patient in cardiogenic shock, observe the cardiac rhythm, monitor hemodynamic parameters, monitor fluid status, and adjust medications and therapies based on the assessment data. Platelet levels and sputum production are not major assessment parameters in a patient who is experiencing cardiogenic shock. 35. A cardiovascular patient with a previous history of pulmonary embolism (PE) is experiencing a sudden onset of dyspnea, rapid breathing, and chest pain. The nurse recognizes the characteristic signs and symptoms of a PE. What is the nurses best action? A) Rapidly assess the patients cardiopulmonary status. B) Arrange for an ECG. C) Increase the height of the patients bed. D) Manage the patients anxiety. A Feedback: Patient management in the event of a PE begins with cardiopulmonary assessment and intervention. This is a priority over ECG monitoring, management of anxiety, or repositioning of the patient, even though each of these actions may be appropriate and necessary. 36. The nurse is caring for a patient who has developed obvious signs of pulmonary edema. What is the priority nursing action? A) Lay the patient flat. B) Notify the family of the patients critical state. C) Stay with the patient. D) Update the physician. C Feedback: Because the patient has an unstable condition, the nurse must remain with the patient. The physician must be updated promptly, but the patient should not be left alone in order for this to happen. Supine positioning is unlikely to relieve dyspnea. The family should be informed, but this is not the priority action. 37. A cardiac patients resistance to left ventricular filling has caused blood to back up into the patients circulatory system. What health problem is likely to result? A) Acute pulmonary edema B) Right-sided HF C) Right ventricular hypertrophy D) Left-sided HF A Feedback: With increased resistance to left ventricular filling, blood backs up into the pulmonary circulation. The patient quickly develops pulmonary edema from the blood volume overload in the lungs. When the blood backs up into the pulmonary circulation, right-sided HF, left-sided HF, and right ventricular hypertrophy do not directly occur. 38. A patient who is at high risk for developing intracardiac thrombi has been placed on long-term anticoagulation. What aspect of the patients health history creates a heightened risk of intracardiac thrombi? A) Atrial fibrillation B) Infective endocarditis C) Recurrent pneumonia D) Recent surgery A Feedback: Intracardiac thrombi are especially common in patients with atrial fibrillation, because the atria do not contract forcefully and blood flows slowly and turbulently, increasing the likelihood of thrombus formation. Endocarditis, pneumonia, and recent surgery do not normally cause an increased risk for intracardiac thrombi formation. 39. Diagnostic imaging reveals that the quantity of fluid in a clients pericardial sac is dangerously increased. The nurse should collaborate with the other members of the care team to prevent the development of what complication? A) Pulmonary edema B) Pericardiocentesis C) Cardiac tamponade D) Pericarditis C Feedback: An increase in pericardial fluid raises the pressure within the pericardial sac and compresses the heart, eventually causing cardiac tamponade. Pericardiocentesis is the treatment for this complication. Pericarditis and pulmonary edema do not result from this pathophysiological process. 40. The nurse is caring for a patient with severe left ventricular dysfunction who has been identified as being at risk for sudden cardiac death. What medical intervention can be performed that may extend the survival of the patient? A) Insertion of an implantable cardioverter defibrillator B) Insertion of an implantable pacemaker C) Administration of a calcium channel blocker D) Administration of a beta-blocker A Feedback: In patients with severe left ventricular dysfunction and the possibility of life-threatening dysrhythmias, placement of an implantable cardioverter defibrillator (ICD) can prevent sudden cardiac death and extend survival. A pacemaker, a calcium channel blocker, and a beta-blocker are not medical interventions that may extend the survival of the patient with left ventricular dysfunction. Chapter 30: Assessment and Management of Patients With Vascular Disorders and Problems of Peripheral Circulation 1. The nurse is taking a health history of a new patient. The patient reports experiencing pain in his left lower leg and foot when walking. This pain is relieved with rest. The nurse notes that the left lower leg is slightly edematous and is hairless. When planning this patients subsequent care, the nurse should most likely address what health problem? A) Coronary artery disease (CAD) B) Intermittent claudication C) Arterial embolus D) Raynauds disease B Feedback: A muscular, cramp-type pain in the extremities consistently reproduced with the same degree of exercise or activity and relieved by rest is experienced by patients with peripheral arterial insufficiency. Referred to as intermittent claudication, this pain is caused by the inability of the arterial system to provide adequate blood flow to the tissues in the face of increased demands for nutrients and oxygen during exercise. The nurse would not suspect the patient has CAD, arterial embolus, or Raynauds disease; none of these health problems produce this cluster of signs and symptoms. 2. While assessing a patient the nurse notes that the patients ankle-brachial index (ABI) of the right leg is 0.40. How should the nurse best respond to this assessment finding? A) Assess the patients use of over-the-counter dietary supplements. B) Implement interventions relevant to arterial narrowing. C) Encourage the patient to increase intake of foods high in vitamin K. D) Adjust the patients activity level to accommodate decreased coronary output. B Feedback: ABI is used to assess the degree of stenosis of peripheral arteries. An ABI of less than 1.0 indicates possible claudication of the peripheral arteries. It does not indicate inadequate coronary output. There is no direct indication for changes in vitamin K intake and OTC medications are not likely causative. 3. The nurse is providing care for a patient who has just been diagnosed with peripheral arterial occlusive disease (PAD). What assessment finding is most consistent with this diagnosis? A) Numbness and tingling in the distal extremities B) Unequal peripheral pulses between extremities C) Visible clubbing of the fingers and toes D) Reddened extremities with muscle atrophy B Feedback: PAD assessment may manifest as unequal pulses between extremities, with the affected leg cooler and paler than the unaffected leg. Intermittent claudication is far more common than sensations of numbness and tingling. Clubbing and muscle atrophy are not associated with PAD. 4. The nurse is admitting a 32-year-old woman to the presurgical unit. The nurse learns during the admission assessment that the patient takes oral contraceptives. Consequently, the nurses postoperative plan of care should include what intervention? A) Early ambulation and leg exercises B) Cessation of the oral contraceptives until 3 weeks postoperative C) Doppler ultrasound of peripheral circulation twice daily D) Dependent positioning of the patients extremities when at rest A Feedback: Oral contraceptive use increases blood coagulability; with bed rest, the patient may be at increased risk of developing deep vein thrombosis. Leg exercises and early ambulation are among the interventions that address this risk. Assessment of peripheral circulation is important, but Doppler ultrasound may not be necessary to obtain these data. Dependent positioning increases the risk of venous thromboembolism (VTE). Contraceptives are not normally discontinued to address the risk of VTE in the short term. 5. A nurse is creating an education plan for a patient with venous insufficiency. What measure should the nurse include in the plan? A) Avoiding tight-fitting socks. B) Limit activity whenever possible. C) Sleep with legs in a dependent position. D) Avoid the use of pressure stockings. A Feedback: Measures taken to prevent complications include avoiding tight-fitting socks and panty girdles; maintaining activities, such as walking, sleeping with legs elevated, and using pressure stockings. Not included in the teaching plan for venous insufficiency would be reducing activity, sleeping with legs dependent, and avoiding pressure stockings. Each of these actions exacerbates venous insufficiency. 6. The nurse is caring for a patient with a large venous leg ulcer. What intervention should the nurse implement to promote healing and prevent infection? A) Provide a high-calorie, high-protein diet. B) Apply a clean occlusive dressing once daily and whenever soiled. C) Irrigate the wound with hydrogen peroxide once daily. D) Apply an antibiotic ointment on the surrounding skin with each dressing change. A Feedback: Wound healing is highly dependent on adequate nutrition. The diet should be sufficiently high in calories and protein. Antibiotic ointments are not normally used on the skin surrounding a leg ulcer and occlusive dressings can exacerbate impaired blood flow. Hydrogen peroxide is not normally used because it can damage granulation tissue. 7. The nurse is caring for a patient who returned from the tropics a few weeks ago and who sought care with signs and symptoms of lymphedema. The nurses plan of care should prioritize what nursing diagnosis? A) Risk for infection related to lymphedema B) Disturbed body image related to lymphedema C) Ineffective health maintenance related to lymphedema D) Risk for deficient fluid volume related to lymphedema A Feedback: Lymphedema, which is caused by accumulation of lymph in the tissues, constitutes a significant risk for infection. The patients body image is likely to be disturbed, and the nurse should address this, but infection is a more significant threat to the patients physiological well-being. Lymphedema is unrelated to ineffective health maintenance and deficient fluid volume is not a significant risk. 8. An occupational health nurse is providing an educational event and has been asked by an administrative worker about the risk of varicose veins. What should the nurse suggest as a proactive preventative measure for varicose veins? A) Sit with crossed legs for a few minutes each hour to promote relaxation. B) Walk for several minutes every hour to promote circulation. C) Elevate the legs when tired. D) Wear snug-fitting ankle socks to decrease edema. B Feedback: A proactive approach to preventing varicose veins would be to walk for several minutes every hour to promote circulation. Sitting with crossed legs may promote relaxation, but it is contraindicated for patients with, or at risk for, varicose veins. Elevating the legs only helps blood passively return to the heart and does not help maintain the competency of the valves in the veins. Wearing tight ankle socks is contraindicated for patients with, or at risk for, varicose veins; socks that are below the muscles of the calf do not promote venous return, the socks simply capture the blood and promote venous stasis. 9. A patient comes to the walk-in clinic with complaints of pain in his foot following stepping on a roofing nail 4 days ago. The patient has a visible red streak running up his foot and ankle. What health problem should the nurse suspect? A) Cellulitis B) Local inflammation C) Elephantiasis D) Lymphangitis D Feedback: Lymphangitis is an acute inflammation of the lymphatic channels. It arises most commonly from a focus of infection in an extremity. Usually, the infectious organism is hemolytic streptococcus. The characteristic red streaks that extend up the arm or the leg from an infected wound outline the course of the lymphatic vessels as they drain. Cellulitis is caused by bacteria, which cause a generalized edema in the subcutaneous tissues surrounding the affected area. Local inflammation would not present with red streaks in the lymphatic channels. Elephantiasis is transmitted by mosquitoes that carry parasitic worm larvae; the parasites obstruct the lymphatic channels and results in gross enlargement of the limbs. 10. The triage nurse in the ED is assessing a patient who has presented with complaint of pain and swelling in her right lower leg. The patients pain became much worse last night and appeared along with fever, chills, and sweating. The patient states, I hit my leg on the car door 4 or 5 days ago and it has been sore ever since. The patient has a history of chronic venous insufficiency. What intervention should the nurse anticipate for this patient? A) Platelet transfusion to treat thrombocytopenia B) Warfarin to treat arterial insufficiency C) Antibiotics to treat cellulitis D) Heparin IV to treat VTE C Feedback: Cellulitis is the most common infectious cause of limb swelling. The signs and symptoms include acute onset of swelling, localized redness, and pain; it is frequently associated with systemic signs of fever, chills, and sweating. The patient may be able to identify a trauma that accounts for the source of infection. Thrombocytopenia is a loss or decrease in platelets and increases a patients risk of bleeding; this problem would not cause these symptoms. Arterial insufficiency would present with ongoing pain related to activity. This patient does not have signs and symptoms of VTE. 11. A nurse in a long-term care facility is caring for an 83-year-old woman who has a history of HF and peripheral arterial disease (PAD). At present the patient is unable to stand or ambulate. The nurse should implement measures to prevent what complication? A) Aoritis B) Deep vein thrombosis C) Thoracic aortic aneurysm D) Raynauds disease B Feedback: Although the exact cause of venous thrombosis remains unclear, three factors, known as Virchows triad, are believed to play a significant role in its development: stasis of blood (venous stasis), vessel wall injury, and altered blood coagulation. In this womans case, she has venous stasis from immobility, vessel wall injury from PAD, and altered blood coagulation from HF. The cause of aoritis is unknown, but it has no direct connection to HF, PAD, or mobility issues. The greatest risk factors for thoracic aortic aneurysm are atherosclerosis and hypertension; there is no direct connection to HF, PAD, or mobility issues. Raynauds disease is a disorder that involves spasms of blood vessels and, again, no direct connection to HF, PAD, or mobility issues. 12. A nurse is admitting a 45-year-old man to the medical unit who has a history of PAD. While providing his health history, the patient reveals that he smokes about two packs of cigarettes a day, has a history of alcohol abuse, and does not exercise. What would be the priority health education for this patient? A) The lack of exercise, which is the main cause of PAD. B) The likelihood that heavy alcohol intake is a significant risk factor for PAD. C) Cigarettes contain nicotine, which is a powerful vasoconstrictor and may cause or aggravate PAD. D) Alcohol suppresses the immune system, creates high glucose levels, and may cause PAD. C Feedback: Tobacco is powerful vasoconstrictor; its use with PAD is highly detrimental, and patients are strongly advised to stop using tobacco. Sedentary lifestyle is also a risk factor, but smoking is likely a more significant risk factor that the nurse should address. Alcohol use is less likely to cause PAD, although it carries numerous health risks. 13. A nurse has written a plan of care for a man diagnosed with peripheral arterial insufficiency. One of the nursing diagnoses in the care plan is altered peripheral tissue perfusion related to compromised circulation. What is the most appropriate intervention for this diagnosis? A) Elevate his legs and arms above his heart when resting. B) Encourage the patient to engage in a moderate amount of exercise. C) Encourage extended periods of sitting or standing. D) Discourage walking in order to limit pain. B Feedback: The nursing diagnosis of altered peripheral tissue perfusion related to compromised circulation requires interventions that focus on improving circulation. Encouraging the patient to engage in a moderate amount of exercise serves to improve circulation. Elevating his legs and arms above his heart when resting would be passive and fails to promote circulation. Encouraging long periods of sitting or standing would further compromise circulation. The nurse should encourage, not discourage, walking to increase circulation and decrease pain. 14. The nurse is caring for a 72-year-old patient who is in cardiac rehabilitation following heart surgery. The patient has been walking on a regular basis for about a week and walks for 15 minutes 3 times a day. The patient states that he is having a cramp-like pain in the legs every time he walks and that the pain gets better when I rest. The patients care plan should address what problem? A) Decreased mobility related to VTE B) Acute pain related to intermittent claudication C) Decreased mobility related to venous insufficiency D) Acute pain related to vasculitis B Feedback: Intermittent claudication presents as a muscular, cramp-type pain in the extremities consistently reproduced with the same degree of exercise or activity and relieved by rest. Patients with peripheral arterial insufficiency often complain of intermittent claudication due to a lack of oxygen to muscle tissue. Venous insufficiency presents as a disorder of venous blood reflux and does not present with cramp-type pain with exercise. Vasculitis is an inflammation of the blood vessels and presents with weakness, fever, and fatigue, but does not present with cramp-type pain with exercise. The pain associated with VTE does not have this clinical presentation. 15. A nurse in the rehabilitation unit is caring for an older adult patient who is in cardiac rehabilitation following an MI. The nurses plan of care calls for the patient to walk for 10 minutes 3 times a day. The patient questions the relationship between walking and heart function. How should the nurse best reply? A) The arteries in your legs constrict when you walk and allow the blood to move faster and with more pressure on the tissue. B) Walking increases your heart rate and blood pressure. Therefore your heart is under less stress. C) Walking helps your heart adjust to your new arteries and helps build your self-esteem. D) When you walk, the muscles in your legs contract and pump the blood in your veins back toward your heart, which allows more blood to return to your heart. D Feedback: Veins, unlike arteries, are equipped with valves that allow blood to move against the force of gravity. The legs have one-way bicuspid valves that prevent blood from seeping backward as it moves forward by the muscles in our legs pressing on the veins as we walk and increasing venous return. Leg arteries do constrict when walking, which allows the blood to move faster and with more pressure on the tissue, but the greater concern is increasing the flow of venous blood to the heart. Walking increases, not decreases, the heart pumping ability, which increases heart rate and blood pressure and the hearts ability to manage stress. Walking does help the heart adjust to new arteries and may enhance self-esteem, but the patient had an MIthere are no new arteries. 16. The nurse is caring for a patient who is admitted to the medical unit for the treatment of a venous ulcer in the area of her lateral malleolus that has been unresponsive to treatment. What is the nurse most likely to find during an assessment of this patients wound? A) Hemorrhage B) Heavy exudate C) Deep wound bed D) Pale-colored wound bed B Feedback: Venous ulcerations in the area of the medial or lateral malleolus (gaiter area) are typically large, superficial, and highly exudative. Venous hypertension causes extravasation of blood, which discolors the area of the wound bed. Bleeding is not normally present. 17. The nurse is preparing to administer warfarin (Coumadin) to a client with deep vein thrombophlebitis (DVT). Which laboratory value would most clearly indicate that the patients warfarin is at therapeutic levels? A) Partial thromboplastin time (PTT) within normal reference range B) Prothrombin time (PT) eight to ten times the control C) International normalized ratio (INR) between 2 and 3 D) Hematocrit of 32% C Feedback: The INR is most often used to determine if warfarin is at a therapeutic level; an INR of 2 to 3 is considered therapeutic. Warfarin is also considered to be at therapeutic levels when the clients PT is 1.5 to 2 times the control. Higher values indicate increased risk of bleeding and hemorrhage, whereas lower values indicate increased risk of blood clot formation. Heparin, not warfarin, prolongs PTT. Hematocrit does not provide information on the effectiveness of warfarin; however, a falling hematocrit in a client taking warfarin may be a sign of hemorrhage. 18. The clinic nurse is caring for a 57-year-old client who reports experiencing leg pain whenever she walks several blocks. The patient has type 1 diabetes and has smoked a pack of cigarettes every day for the past 40 years. The physician diagnoses intermittent claudication. The nurse should provide what instruction about long-term care to the client? A) Be sure to practice meticulous foot care. B) Consider cutting down on your smoking. C) Reduce your activity level to accommodate your limitations. D) Try to make sure you eat enough protein. A Feedback: The patient with peripheral vascular disease or diabetes should receive education or reinforcement about skin and foot care. Intermittent claudication and other chronic peripheral vascular diseases reduce oxygenation to the feet, making them susceptible to injury and poor healing; therefore, meticulous foot care is essential. The patient should stop smokingnot just cut downbecause nicotine is a vasoconstrictor. Daily walking benefits the patient with intermittent claudication. Increased protein intake will not alleviate the patients symptoms. 19. A patient who has undergone a femoral to popliteal bypass graft surgery returns to the surgical unit. Which assessments should the nurse perform during the first postoperative day? A) Assess pulse of affected extremity every 15 minutes at first. B) Palpate the affected leg for pain during every assessment. C) Assess the patient for signs and symptoms of compartment syndrome every 2 hours. D) Perform Doppler evaluation once daily. A Feedback: The primary objective in the postoperative period is to maintain adequate circulation through the arterial repair. Pulses, Doppler assessment, color and temperature, capillary refill, and sensory and motor function of the affected extremity are checked and compared with those of the other extremity; these values are recorded initially every 15 minutes and then at progressively longer intervals if the patients status remains stable. Doppler evaluations should be performed every 2 hours. Pain is regularly assessed, but palpation is not the preferred method of performing this assessment. Compartment syndrome results from the placement of a cast, not from vascular surgery. 20. You are caring for a patient who is diagnosed with Raynauds phenomenon. The nurse should plan interventions to address what nursing diagnosis? A) Chronic pain B) Ineffective tissue perfusion C) Impaired skin integrity D) Risk for injury B Feedback: Raynauds phenomenon is a form of intermittent arteriolar vasoconstriction resulting in inadequate tissue perfusion. This results in coldness, pain, and pallor of the fingertips or toes. Pain is typically intermittent and acute, not chronic, and skin integrity is rarely at risk. In most cases, the patient is not at a high risk for injury. 21. A patient presents to the clinic complaining of the inability to grasp objects with her right hand. The patients right arm is cool and has a difference in blood pressure of more than 20 mm Hg compared with her left arm. The nurse should expect that the primary care provider may diagnose the woman with what health problem? A) Lymphedema B) Raynauds phenomenon C) Upper extremity arterial occlusive disease D) Upper extremity VTE C Feedback: The patient with upper extremity arterial occlusive disease typically complains of arm fatigue and pain with exercise (forearm claudication) and inability to hold or grasp objects (e.g., combing hair, placing objects on shelves above the head) and, occasionally, difficulty driving. Assessment findings include coolness and pallor of the affected extremity, decreased capillary refill, and a difference in arm blood pressures of more than 20 mm Hg. These symptoms are not closely associated with Raynauds or lymphedema. The upper extremities are rare sites for VTE. 22. A nurse working in a long-term care facility is performing the admission assessment of a newly admitted, 85-year-old resident. During inspection of the residents feet, the nurse notes that she appears to have early evidence of gangrene on one of her great toes. The nurse knows that gangrene in the elderly is often the first sign of what? A) Chronic venous insufficiency B) Raynauds phenomenon C) VTE D) PAD D Feedback: In elderly people, symptoms of PAD may be more pronounced than in younger people. In elderly patients who are inactive, gangrene may be the first sign of disease. Venous insufficiency does not normally manifest with gangrene. Similarly, VTE and Raynauds phenomenon do not cause the ischemia that underlies gangrene. 23. The prevention of VTE is an important part of the nursing care of high-risk patients. When providing patient teaching for these high-risk patients, the nurse should advise lifestyle changes, including which of the following? Select all that apply. A) High-protein diet B) Weight loss C) Regular exercise D) Smoking cessation E) Calcium and vitamin D supplementation B, C, D Feedback: Patients at risk for VTE should be advised to make lifestyle changes, as appropriate, which may include weight loss, smoking cessation, and regular exercise. Increased protein intake and supplementation with vitamin D and calcium do not address the main risk factors for VTE. 24. The nurse is caring for an acutely ill patient who is on anticoagulant therapy. The patient has a comorbidity of renal insufficiency. How will this patients renal status affect heparin therapy? A) Heparin is contraindicated in the treatment of this patient. B) Heparin may be administered subcutaneously, but not IV. C) Lower doses of heparin are required for this patient. D) Coumadin will be substituted for heparin. C Feedback: If renal insufficiency exists, lower doses of heparin are required. Coumadin cannot be safely and effectively used as a substitute and there is no contraindication for IV administration. 25. The nurse is assessing a woman who is pregnant at 27 weeks gestation. The patient is concerned about the recent emergence of varicose veins on the backs of her calves. What is the nurses best response? A) Facilitate a referral to a vascular surgeon. B) Assess the patients ankle-brachial index (ABI) and perform Doppler ultrasound testing. C) Encourage the patient to increase her activity level. D) Teach the patient that circulatory changes during pregnancy frequently cause varicose veins. D Feedback: Pregnancy may cause varicosities because of hormonal effects related to decreased venous outflow, increased pressure by the gravid uterus, and increased blood volume. In most cases, no intervention or referral is necessary. This finding is not an indication for ABI assessment and increased activity will not likely resolve the problem. 26. Graduated compression stockings have been prescribed to treat a patients venous insufficiency. What education should the nurse prioritize when introducing this intervention to the patient? A) The need to take anticoagulants concurrent with using compression stockings B) The need to wear the stockings on a one day on, one day off schedule C) The importance of wearing the stockings around the clock to ensure maximum benefit D) The importance of ensuring the stockings are applied evenly with no pressure points D Feedback: Any type of stocking can inadvertently become a tourniquet if applied incorrectly (i.e., rolled tightly at the top). In such instances, the stockings produce rather than prevent stasis. For ambulatory patients, graduated compression stockings are removed at night and reapplied before the legs are lowered from the bed to the floor in the morning. They are used daily, not on alternating days. Anticoagulants are not always indicated in patients who are using compression stockings. 27. The nurse caring for a patient with a leg ulcer has finished assessing the patient and is developing a problem list prior to writing a plan of care. What major nursing diagnosis might the care plan include? A) Risk for disuse syndrome B) Ineffective health maintenance C) Sedentary lifestyle D) Imbalanced nutrition: less than body requirements D Feedback: Major nursing diagnoses for the patient with leg ulcers may include imbalanced nutrition: less than body requirements, related to increased need for nutrients that promote wound healing. Risk for disuse syndrome is a state in which an individual is at risk for deterioration of body systems owing to prescribed or unavoidable musculoskeletal inactivity. A leg ulcer will affect activity, but rarely to this degree. Leg ulcers are not necessarily a consequence of ineffective health maintenance or sedentary lifestyle. 28. How should the nurse best position a patient who has leg ulcers that are venous in origin? A) Keep the patients legs flat and straight. B) Keep the patients knees bent to 45-degree angle and supported with pillows. C) Elevate the patients lower extremities. D) Dangle the patients legs over the side of the bed. C Feedback: Positioning of the legs depends on whether the ulcer is of arterial or venous origin. With venous insufficiency, dependent edema can be avoided by elevating the lower extremities. Dangling the patients legs and applying pillows may further compromise venous return. 29. A patient with advanced venous insufficiency is confined following orthopedic surgery. How can the nurse best prevent skin breakdown in the patients lower extremities? A) Ensure that the patients heels are protected and supported. B) Closely monitor the patients serum albumin and prealbumin levels. C) Perform gentle massage of the patients lower legs, as tolerated. D) Perform passive range-of-motion exercises once per shift. A Feedback: If the patient is on bed rest, it is important to relieve pressure on the heels to prevent pressure ulcerations, since the heels are among the most vulnerable body regions. Monitoring blood work does not directly prevent skin breakdown, even though albumin is related to wound healing. Massage is not normally indicated and may exacerbate skin breakdown. Passive range- of-motion exercises do not directly reduce the risk of skin breakdown. 30. The nurse has performed a thorough nursing assessment of the care of a patient with chronic leg ulcers. The nurses assessment should include which of the following components? Select all that apply. A) Location and type of pain B) Apical heart rate C) Bilateral comparison of peripheral pulses D) Comparison of temperature in the patients legs E) Identification of mobility limitations A, C, D, E Feedback: A careful nursing history and assessment are important. The extent and type of pain are carefully assessed, as are the appearance and temperature of the skin of both legs. The quality of all peripheral pulses is assessed, and the pulses in both legs are compared. Any limitation of mobility and activity that results from vascular insufficiency is identified. Not likely is there any direct indication for assessment of apical heart rate, although peripheral pulses must be assessed. 31. A nurse on a medical unit is caring for a patient who has been diagnosed with lymphangitis. When reviewing this patients medication administration record, the nurse should anticipate which of the following? A) Coumadin (warfarin) B) Lasix (furosemide) C) An antibiotic D) An antiplatelet aggregator C Feedback: Lymphangitis is an acute inflammation of the lymphatic channels caused by an infectious process. Antibiotics are always a component of treatment. Diuretics are of nominal use. Anticoagulants and antiplatelet aggregators are not indicated in this form of infection. 32. A postsurgical patient has illuminated her call light to inform the nurse of a sudden onset of lower leg pain. On inspection, the nurse observes that the patients left leg is visibly swollen and reddened. What is the nurses most appropriate action? A) Administer a PRN dose of subcutaneous heparin. B) Inform the physician that the patient has signs and symptoms of VTE. C) Mobilize the patient promptly to dislodge any thrombi in the patients lower leg. D) Massage the patients lower leg to temporarily restore venous return. B Feedback: VTE requires prompt medical follow-up. Heparin will not dissolve an established clot. Massaging the patients leg and mobilizing the patient would be contraindicated because they would dislodge the clot, possibly resulting in a pulmonary embolism. 33. A nurse is closely monitoring a patient who has recently been diagnosed with an abdominal aortic aneurysm. What assessment finding would signal an impending rupture of the patients aneurysm? A) Sudden increase in blood pressure and a decrease in heart rate B) Cessation of pulsating in an aneurysm that has previously been pulsating visibly C) Sudden onset of severe back or abdominal pain D) New onset of hemoptysis C Feedback: Signs of impending rupture include severe back or abdominal pain, which may be persistent or intermittent. Impending rupture is not typically signaled by increased blood pressure, bradycardia, cessation of pulsing, or hemoptysis. 34. A nurse is reviewing the physiological factors that affect a patients cardiovascular health and tissue oxygenation. What is the systemic arteriovenous oxygen difference? A) The average amount of oxygen removed by each organ in the body B) The amount of oxygen removed from the blood by the heart C) The amount of oxygen returning to the lungs via the pulmonary artery D) The amount of oxygen in aortic blood minus the amount of oxygen in the vena caval blood D Feedback: The average amount of oxygen removed collectively by all of the body tissues is about 25%. This means that the blood in the vena cava contains about 25% less oxygen than aortic blood. This is known as the systemic arteriovenous oxygen difference. The other answers do not apply. 35. The nurse is evaluating a patients diagnosis of arterial insufficiency with reference to the adequacy of the patients blood flow. On what physiological variables does adequate blood flow depend? Select all that apply. A) Efficiency of heart as a pump B) Adequacy of circulating blood volume C) Ratio of platelets to red blood cells D) Size of red blood cells E) Patency and responsiveness of the blood vessels A, B, E Feedback: Adequate blood flow depends on the efficiency of the heart as a pump, the patency and responsiveness of the blood vessels, and the adequacy of circulating blood volume. Adequacy of blood flow does not primarily depend on the size of red cells or their ratio to the number of platelets. 36. A nurse is assessing a new patient who is diagnosed with PAD. The nurse cannot feel the pulse in the patients left foot. How should the nurse proceed with assessment? A) Have the primary care provider order a CT. B) Apply a tourniquet for 3 to 5 minutes and then reassess. C) Elevate the extremity and attempt to palpate the pulses. D) Use Doppler ultrasound to identify the pulses. D Feedback: When pulses cannot be reliably palpated, a hand-held continuous wave (CW) Doppler ultrasound device may be used to hear (insonate) the blood flow in vessels. CT is not normally warranted and the application of a tourniquet poses health risks and will not aid assessment. Elevating the extremity would make palpation more difficult. 37. A medical nurse has admitted four patients over the course of a 12-hour shift. For which patient would assessment of ankle-brachial index (ABI) be most clearly warranted? A) A patient who has peripheral edema secondary to chronic heart failure B) An older adult patient who has a diagnosis of unstable angina C) A patient with poorly controlled type 1 diabetes who is a smoker D) A patient who has community-acquired pneumonia and a history of COPD C Feedback: Nurses should perform a baseline ABI on any patient with decreased pulses or any patient 50 years of age or older with a history of diabetes or smoking. The other answers do not apply. 38. An older adult patient has been treated for a venous ulcer and a plan is in place to prevent the occurrence of future ulcers. What should the nurse include in this plan? A) Use of supplementary oxygen to aid tissue oxygenation B) Daily use of normal saline compresses on the lower limbs C) Daily administration of prophylactic antibiotics D) A high-protein diet that is rich in vitamins D Feedback: A diet that is high in protein, vitamins C and A, iron, and zinc is encouraged to promote healing and prevent future ulcers. Prophylactic antibiotics and saline compresses are not used to prevent ulcers. Oxygen supplementation does not prevent ulcer formation. 39. A 79-year-old man is admitted to the medical unit with digital gangrene. The man states that his problems first began when he stubbed his toe going to the bathroom in the dark. In addition to this trauma, the nurse should suspect that the patient has a history of what health problem? A) Raynauds phenomenon B) CAD C) Arterial insufficiency D) Varicose veins C Feedback: Arterial insufficiency may result in gangrene of the toe (digital gangrene), which usually is caused by trauma. The toe is stubbed and then turns black. Raynauds, CAD and varicose veins are not the usual causes of digital gangrene in the elderly. 40. When assessing venous disease in a patients lower extremities, the nurse knows that what test will most likely be ordered? A) Duplex ultrasonography B) Echocardiography C) Positron emission tomography (PET) D) Radiography A Feedback: Duplex ultrasound may be used to determine the level and extent of venous disease as well as its chronicity. Radiographs (x-rays), PET scanning, and echocardiography are never used for this purpose as they do not allow visualization of blood flow. Chapter 31: Assessment and Management of Patients With Hypertension 1. An older adult is newly diagnosed with primary hypertension and has just been started on a beta-blocker. The nurses health education should include which of the following? A) Increasing fluids to avoid extracellular volume depletion from the diuretic effect of the beta- blocker B) Maintaining a diet high in dairy to increase protein necessary to prevent organ damage C) Use of strategies to prevent falls stemming from postural hypotension D) Limiting exercise to avoid injury that can be caused by increased intracranial pressure C Feedback: Elderly people have impaired cardiovascular reflexes and are more sensitive to postural hypotension. The nurse teaches patients to change positions slowly when moving from lying or sitting positions to a standing position, and counsels elderly patients to use supportive devices as necessary to prevent falls that could result from dizziness. Lifestyle changes, such as regular physical activity/exercise, and a diet rich in fruits, vegetables, and low-fat dairy products, is strongly recommended. Increasing fluids in elderly patients may be contraindicated due to cardiovascular disease. Increased intracranial pressure is not a risk and activity should not normally be limited. 2. A patient with primary hypertension comes to the clinic complaining of a gradual onset of blurry vision and decreased visual acuity over the past several weeks. The nurse is aware that these symptoms could be indicative of what? A) Retinal blood vessel damage B) Glaucoma C) Cranial nerve damage D) Hypertensive emergency A Feedback: Blurred vision, spots in front of the eyes, and diminished visual acuity can mean retinal blood vessel damage indicative of damage elsewhere in the vascular system as a result of hypertension. Glaucoma and cranial nerve damage do not normally cause these symptoms. A hypertensive emergency would have a more rapid onset. 3. A nurse is performing blood pressure screenings at a local health fair. While obtaining subjective assessment data from a patient with hypertension, the nurse learns that the patient has a family history of hypertension and she herself has high cholesterol and lipid levels. The patient says she smokes one pack of cigarettes daily and drinks about a pack of beer every day. The nurse notes what nonmodifiable risk factor for hypertension? A) Hyperlipidemia B) Excessive alcohol intake C) A family history of hypertension D) Closer adherence to medical regimen C Feedback: Unlike cholesterol levels, alcohol intake and adherence to treatment, family history is not modifiable. 4. The staff educator is teaching ED nurses about hypertensive crisis. The nurse educator should explain that hypertensive urgency differs from hypertensive emergency in what way? A) The BP is always higher in a hypertensive emergency. B) Vigilant hemodynamic monitoring is required during treatment of hypertensive emergencies. C) Hypertensive urgency is treated with rest and benzodiazepines to lower BP. D) Hypertensive emergencies are associated with evidence of target organ damage. D Feedback: Hypertensive emergencies are acute, life-threatening BP elevations that require prompt treatment in an intensive care setting because of the serious target organ damage that may occur. Blood pressures are extremely elevated in both urgency and emergencies, but there is no evidence of target organ damage in hypertensive urgency. Extremely close hemodynamic monitoring of the patients BP is required in both situations. The medications of choice in hypertensive emergencies are those with an immediate effect, such as IV vasodilators. Oral doses of fast-acting agents, such as beta-adrenergic blocking agents, angiotensin-converting enzyme inhibitors, or alpha-agonists, are recommended for the treatment of hypertensive urgencies. 5. A group of student nurses are practicing taking blood pressure. A 56-year-old male student has a blood pressure reading of 146/96 mm Hg. Upon hearing the reading, he exclaims, My pressure has never been this high. Do you think my doctor will prescribe medication to reduce it? Which of the following responses by the nursing instructor would be best? A) Yes. Hypertension is prevalent among men; it is fortunate we caught this during your routine examination. B) We will need to reevaluate your blood pressure because your age places you at high risk for hypertension. C) A single elevated blood pressure does not confirm hypertension. You will need to have your blood pressure reassessed several times before a diagnosis can be made. D) You have no need to worry. Your pressure is probably elevated because you are being tested. C Feedback: Hypertension is confirmed by two or more readings with systolic pressure of at least 140 mm Hg and diastolic pressure of at least 90 mm Hg. An age of 56 does not constitute a risk factor in and of itself. The nurse should not tell the student that there is no need to worry. A 40-year-old male newly diagnosed with hypertension is discussing risk factors with the nurse. The nurse talks about lifestyle changes with the patient and advises that the patient should avoid tobacco use. What is the primary rationale behind that advice to the patient? A) Quitting smoking will cause the patients hypertension to resolve. B) Tobacco use increases the patients concurrent risk of heart disease. C) Tobacco use is associated with a sedentary lifestyle. D) Tobacco use causes ventricular hypertrophy. B Feedback: Smoking increases the risk for heart disease, for which a patient with hypertension is already at an increased risk. Quitting will not necessarily cause hypertension to resolve and smoking does not directly cause ventricular hypertrophy. The association with a sedentary lifestyle is true, but this is not the main rationale for the nurses advice; the association with heart disease is more salient. 7. A patient has been prescribed antihypertensives. After assessment and analysis, the nurse has identified a nursing diagnosis of risk for ineffective health maintenance related to nonadherence to therapeutic regimen. When planning this patients care, what desired outcome should the nurse identify? A) Patient takes medication as prescribed and reports any adverse effects. B) Patients BP remains consistently below 140/90 mm Hg. C) Patient denies signs and symptoms of hypertensive urgency. D) Patient is able to describe modifiable risk factors for hypertension. A Feedback: The most appropriate expected outcome for a patient who is given the nursing diagnosis of risk for ineffective health maintenance is that he or she takes the medication as prescribed. The other listed goals are valid aspects of care, but none directly relates to the patients role in his or her treatment regimen. 8. The nurse is providing care for a patient with a new diagnosis of hypertension. How can the nurse best promote the patients adherence to the prescribed therapeutic regimen? A) Screen the patient for visual disturbances regularly. B) Have the patient participate in monitoring his or her own BP. C) Emphasize the dire health outcomes associated with inadequate BP control. D) Encourage the patient to lose weight and exercise regularly. B Feedback: Adherence to the therapeutic regimen increases when patients actively participate in self-care, including self-monitoring of BP and diet. Dire warnings may motivate some patients, but for many patients this is not an appropriate or effective strategy. Screening for vision changes and promoting healthy lifestyle are appropriate nursing actions, but do not necessarily promote adherence to a therapeutic regimen. 9. A patient newly diagnosed with hypertension asks the nurse what happens when uncontrolled hypertension is prolonged. The nurse explains that a patient with prolonged, uncontrolled hypertension is at risk for developing what health problem? A) Renal failure B) Right ventricular hypertrophy C) Glaucoma D) Anemia A Feedback: When uncontrolled hypertension is prolonged, it can result in renal failure, myocardial infarction, stroke, impaired vision, left ventricular hypertrophy, and cardiac failure. Glaucoma and anemia are not directly associated with hypertension. 10. A patient with primary hypertension complains of dizziness with ambulation. The patient is currently on an alpha-adrenergic blocker and the nurse assesses characteristic signs and symptoms of postural hypotension. When teaching this patient about risks associated with postural hypotension, what should the nurse emphasize? A) Rising slowly from a lying or sitting position B) Increasing fluids to maintain BP C) Stopping medication if dizziness persists D) Taking medication first thing in the morning A Feedback: Patients who experience postural hypotension should be taught to rise slowly from a lying or sitting position and use a cane or walker if necessary for safety. It is not necessary to teach these patients about increasing fluids or taking medication in the morning (this would increase the effects of dizziness). Patient should not be taught to stop the medication if dizziness persists because this is unsafe and beyond the nurses scope of practice. 11. The nurse is planning the care of a patient who has been diagnosed with hypertension, but who otherwise enjoys good health. When assessing the response to an antihypertensive drug regimen, what blood pressure would be the goal of treatment? A) 156/96 mm Hg or lower B) 140/90 mm Hg or lower C) Average of 2 BP readings of 150/80 mm Hg D) 120/80 mm Hg or lower B Feedback: The goal of antihypertensive drug therapy is a BP of 140/90 mm Hg or lower. A pressure of 130/80 mm Hg is the goal for patients with diabetes or chronic kidney disease. 12. A patient in a hypertensive emergency is admitted to the ICU. The nurse anticipates that the patient will be treated with IV vasodilators, and that the primary goal of treatment is what? A) Lower the BP to reduce onset of neurologic symptoms, such as headache and vision changes. B) Decrease the BP to a normal level based on the patients age. C) Decrease the mean arterial pressure between 20% and 25% in the first hour of treatment. D) Reduce the BP to 120/75 mm Hg as quickly as possible. C Feedback: Initially, the treatment goal in hypertensive emergencies is to reduce the mean arterial pressure by 25% in the first hour of treatment, with further reduction over the next 24 hours. Lowering the BP too fast may cause hypotension in a patient whose body has adjusted to hypertension and could cause a stroke, MI, or visual changes. Neurologic symptoms should be addressed, but this is not the primary focus of treatment planning. 13. The nursing lab instructor is teaching student nurses how to take blood pressure. To ensure accurate measurement, the lab instructor would teach the students to avoid which of the following actions? A) Measuring the BP after the patient has been seated quietly for more than 5 minutes B) Taking the BP at least 10 minutes after nicotine or coffee ingestion C) Using a cuff with a bladder that encircles at least 80% of the limb D) Using a bare forearm supported at heart level on a firm surface B Feedback: Blood pressures should be taken with the patient seated with arm bare, supported, and at heart level. The patient should not have smoked tobacco or taken caffeine in the 30 minutes preceding the measurement. The patient should rest quietly for 5 minutes before the reading is taken. The cuff bladder should encircle at least 80% of the limb being measured and have a width of at least 40% of limb circumference. Using a cuff that is too large results in a lower BP and a cuff that is too small will give a higher BP measurement. 14. A nurse is teaching an adult female patient about the risk factors for hypertension. What should the nurse explain as risk factors for primary hypertension? A) Obesity and high intake of sodium and saturated fat B) Diabetes and use of oral contraceptives C) Metabolic syndrome and smoking D) Renal disease and coarctation of the aorta A Feedback: Obesity, stress, high intake of sodium or saturated fat, and family history are all risk factors for primary hypertension. Diabetes and oral contraceptives are risk factors for secondary hypertension. Metabolic syndrome, renal disease, and coarctation of the aorta are causes of secondary hypertension. 15. The nurse is caring for an older adult with a diagnosis of hypertension who is being treated with a diuretic and beta-blocker. Which of the following should the nurse integrate into the management of this clients hypertension? A) Ensure that the patient receives a larger initial dose of antihypertensive medication due to impaired absorption. B) Pay close attention to hydration status because of increased sensitivity to extracellular volume depletion. C) Recognize that an older adult is less likely to adhere to his or her medication regimen than a younger patient. D) Carefully assess for weight loss because of impaired kidney function resulting from normal aging. B Feedback: Elderly people have impaired cardiovascular reflexes and thus are more sensitive to extracellular volume depletion caused by diuretics. The nurse needs to assess hydration status, low BP, and postural hypotension carefully. Older adults may have impaired absorption, but they do not need a higher initial dose of an antihypertensive than a younger person. Adherence to treatment is not necessarily linked to age. Kidney function and absorption decline with age; less, rather than more antihypertensive medication is prescribed. Weight gain is not necessarily indicative of kidney function decline. 16. A patient with secondary hypertension has come into the clinic for a routine check-up. The nurse is aware that the difference between primary hypertension and secondary hypertension is which of the following? A) Secondary hypertension has a specific cause. B) Secondary hypertension has a more gradual onset than primary hypertension. C) Secondary hypertension does not cause target organ damage. D) Secondary hypertension does not normally respond to antihypertensive drug therapy. A Feedback: Secondary hypertension has a specific identified cause. A cause could include narrowing of the renal arteries, renal parenchymal disease, hyperaldosteronism, certain medications, pregnancy, and coarctation of the aorta. Secondary hypertension does respond to antihypertensive drug therapy and can cause target organ damage if left untreated. 17. The nurse is assessing a patient new to the clinic. Records brought to the clinic with the patient show the patient has hypertension and that her current BP readings approximate the readings from when she was first diagnosed. What contributing factor should the nurse first explore in an effort to identify the cause of the clients inadequate BP control? A) Progressive target organ damage B) Possibility of medication interactions C) Lack of adherence to prescribed drug therapy D) Possible heavy alcohol use or use of recreational drugs C Feedback: Deviation from the therapeutic program is a significant problem for people with hypertension and other chronic conditions requiring lifetime management. An estimated 50% of patients discontinue their medications within 1 year of beginning to take them. Consequently, this is a more likely problem than substance use, organ damage, or adverse drug interactions. 18. A patient has come to the clinic for a follow-up assessment that will include a BP reading. To ensure an accurate reading, the nurse should confirm that the patient has done which of the following? A) Tried to rest quietly for 5 minutes before the reading is taken B) Refrained from smoking for at least 8 hours C) Drunk adequate fluids during the day prior D) Avoided drinking coffee for 12 hours before the visit A Feedback: Prior to the nurse assessing the patients BP, the patient should try to rest quietly for 5 minutes. The forearm should be positioned at heart level. Caffeine products and cigarette smoking should be avoided for at least 30 minutes prior to the visit. Recent fluid intake is not normally relevant. 19. The nurse is providing care for a patient with a diagnosis of hypertension. The nurse should consequently assess the patient for signs and symptoms of which other health problem? A) Migraines B) Atrial-septal defect C) Atherosclerosis D) Thrombocytopenia C Feedback: Hypertension is both a sign and a risk factor for atherosclerotic heart disease. It is not associated with structural cardiac defects, low platelet levels, or migraines. 20. The nurse is developing a nursing care plan for a patient who is being treated for hypertension. What is a measurable patient outcome that the nurse should include? A) Patient will reduce Na+ intake to no more than 2.4 g daily. B) Patient will have a stable BUN and serum creatinine levels. C) Patient will abstain from fat intake and reduce calorie intake. D) Patient will maintain a normal body weight. A Feedback: Dietary sodium intake of no more than 2.4 g sodium is recommended as a dietary lifestyle modification to prevent and manage hypertension. Giving a specific amount of allowable sodium intake makes this a measurable goal. None of the other listed goals is quantifiable and measurable. 21. A patient with newly diagnosed hypertension has come to the clinic for a follow-up visit. The patient asks the nurse why she has to come in so often. What would be the nurses best response? A) We do this so you dont suffer a stroke. B) We do this to determine how your blood pressure changes throughout the day. C) We do this to see how often you should change your medication dose. D) We do this to make sure your health is stable. Well then monitor it at routinely scheduled intervals. D Feedback: When hypertension is initially detected, nursing assessment involves carefully monitoring the BP at frequent intervals and then at routinely scheduled intervals. The reference to stroke is frightening and does not capture the overall rationale for the monitoring regimen. Changes throughout the day are not a clinical priority for most patients. The patient must not change his or her medication doses unilaterally. 22. The hospital nurse cares for many patients who have hypertension. What nursing diagnosis is most common among patients who are being treated for this health problem? A) Deficient knowledge regarding the lifestyle modifications for management of hypertension B) Noncompliance with therapeutic regimen related to adverse effects of prescribed therapy C) Deficient knowledge regarding BP monitoring D) Noncompliance with treatment regimen related to medication costs B Feedback: Deviation from the therapeutic program is a significant problem for people with hypertension and other chronic conditions requiring lifetime management. For many patients, this is related to adverse effects of medications. Medication cost is relevant for many patients, but adverse effects are thought to be a more significant barrier. Many patients are aware of necessary lifestyle modification, but do not adhere to them. Most patients are aware of the need to monitor their BP. 23. The nurse is teaching a patient about some of the health consequences of uncontrolled hypertension. What health problems should the nurse describe? Select all that apply. A) Transient ischemic attacks B) Cerebrovascular accident C) Retinal hemorrhage D) Venous insufficiency E) Right ventricular hypertrophy A, B, C Feedback: Potential complications of hypertension include the following: left ventricular hypertrophy; MI; heart failure; transient ischemic attacks (TIAs); cerebrovascular accident; renal insufficiency and failure; and retinal hemorrhage. Venous insufficiency and right ventricular hypertrophy are not potential complications of uncontrolled hypertension. 24. The nurse is collaborating with the dietitian and a patient with hypertension to plan dietary modifications. These modifications should include which of the following? A) Reduced intake of protein and carbohydrates B) Increased intake of calcium and vitamin D C) Reduced intake of fat and sodium D) Increased intake of potassium, vitamin B12 and vitamin D C Feedback: Lifestyle modifications usually include restricting sodium and fat intake, increasing intake of fruits and vegetables, and implementing regular physical activity. There is no need to increase calcium, potassium, and vitamin intake. Calorie restriction may be required for some patients, but a specific reduction in protein and carbohydrates is not normally indicated. 25. The critical care nurse is caring for a patient just admitted in a hypertensive emergency. The nurse should anticipate the administration of what medication? A) Warfarin (Coumadin) B) Furosemide (Lasix) C) Sodium nitroprusside (Nitropress) D) Ramipril (Altace) C Feedback: The medications of choice in hypertensive emergencies are those that have an immediate effect. IV vasodilators, including sodium nitroprusside (Nitropress), nicardipine hydrochloride (Cardene), clevidipine (Cleviprex), fenoldopam mesylate (Corlopam), enalaprilat, and nitroglycerin, have immediate actions that are short lived (minutes to 4 hours), and they are therefore used for initial treatment. Ramipril is administered orally and would not meet the patients immediate need for BP management. Diuretics, such as Lasix, are not used as initial treatments and there is no indication for anticoagulants such as Coumadin. 26. A patient in hypertensive emergency is being cared for in the ICU. The patient has become hypovolemic secondary to natriuresis. What is the nurses most appropriate action? A) Add sodium to the patients IV fluid, as ordered. B) Administer a vasoconstrictor, as ordered. C) Promptly cease antihypertensive therapy. D) Administer normal saline IV, as ordered. D Feedback: If there is volume depletion secondary to natriuresis caused by the elevated BP, then volume replacement with normal saline can prevent large, sudden drops in BP when antihypertensive medications are administered. Sodium administration, cessation of antihypertensive therapy, and administration of vasoconstrictors are not normally indicated. 27. During an adult patients last two office visits, the nurse obtained BP readings of 122/84 mm Hg and 130/88 mm Hg, respectively. How would this patients BP be categorized? A) Normal B) Prehypertensive C) Stage 1 hypertensive D) Stage 2 hypertensive B Feedback: Prehypertension is defined systolic BP of 120 to 139 mm Hg or diastolic BP of 80 to 89 mm Hg. 28. A patient comes to the walk-in clinic complaining of frequent headaches. While assessing the patients vital signs, the nurse notes the BP is 161/101 mm Hg. According to JNC 7, how would this patients BP be defined if a similar reading were obtained at a subsequent office visit? A) High normal B) Normal C) Stage 1 hypertensive D) Stage 2 hypertensive D Feedback: JNC 7 defines stage 2 hypertension as a reading 160/100 mm Hg. 29. A patient has been diagnosed as being prehypertensive. What should the nurse encourage this patient to do to aid in preventing a progression to a hypertensive state? A) Avoid excessive potassium intake. B) Exercise on a regular basis. C) Eat less protein and more vegetables. D) Limit morning activity. B Feedback: To prevent or delay progression to hypertension and reduce risk, JNC 7 urged health care providers to encourage people with blood pressures in the prehypertension category to begin lifestyle modifications, such as nutritional changes and exercise. There is no need for patients to limit their activity in the morning or to avoid potassium and protein intake. 30. The nurse is screening a number of adults for hypertension. What range of blood pressure is considered normal? A) Less than 140/90 mm Hg B) Less than 130/90 mm Hg C) Less than 129/89 mm Hg D) Less than 120/80 mm Hg D Feedback: JNC 7 defines a blood pressure of less than 120/80 mm Hg as normal, 120 to 129/80 to 89 mm Hg as prehypertension, and 140/90 mm Hg or higher as hypertension. 31. A community health nurse teaching a group of adults about preventing and treating hypertension. The nurse should encourage these participants to collaborate with their primary care providers and regularly monitor which of the following? A) Heart rate B) Sodium levels C) Potassium levels D) Blood lipid levels D Feedback: Hypertension often accompanies other risk factors for atherosclerotic heart disease, such as dyslipidemia (abnormal blood fat levels), obesity, diabetes, metabolic syndrome, and a sedentary lifestyle. Individuals with hypertension need to monitor their sodium intake, but hypernatremia is not a risk factor for hypertension. In many patients, heart rate does not correlate closely with BP. Potassium levels do not normally relate to BP. 32. A community health nurse is planning an educational campaign addressing hypertension. The nurse should anticipate that the incidence and prevalence of hypertension are likely to be highest among members of what ethnic group? A) Pacific Islanders B) African Americans C) Asian-Americans D) Hispanics D Feedback: The prevalence of uncontrolled hypertension varies by ethnicity, with Hispanics and African Americans having the highest prevalence at approximately 63% and 57%, respectively. 33. The home health nurse is caring for a patient who has a comorbidity of hypertension. What assessment question most directly addresses the possibility of worsening hypertension? A) Are you eating less salt in your diet? B) How is your energy level these days? C) Do you ever get chest pain when you exercise? D) Do you ever see spots in front of your eyes? D Feedback: To identify complications or worsening hypertension, the patient is questioned about blurred vision, spots in front of the eyes, and diminished visual acuity. The heart, nervous system, and kidneys are also carefully assessed, but angina pain and decreased energy are not normally suggestive of worsening hypertension. Sodium limitation is a beneficial lifestyle modification, but nonadherence to this is not necessarily a sign of worsening symptoms. 34. A student nurse is taking care of an elderly patient with hypertension during a clinical experience. The instructor asks the student about the relationships between BP and age. What would be the best answer by the student? A) Because of reduced smooth muscle tone in blood vessels, blood pressure tends to go down with age, not up. B) Decreases in the strength of arteries and the presence of venous insufficiency cause hypertension in the elderly. C) Structural and functional changes in the cardiovascular system that occur with age contribute to increases in blood pressure. D) The neurologic system of older adults is less efficient at monitoring and regulating blood pressure. C Feedback: Structural and functional changes in the heart and blood vessels contribute to increases in BP that occur with aging. Venous insufficiency does not cause hypertension, however. Increased BP is not primarily a result of neurologic changes. 35. A 55-year-old patient comes to the clinic for a routine check-up. The patients BP is 159/100 mm Hg and the physician diagnoses hypertension after referring to previous readings. The patient asks why it is important to treat hypertension. What would be the nurses best response? A) Hypertension can cause you to develop dangerous blood clots in your legs that can migrate to your lungs. B) Hypertension puts you at increased risk of type 1 diabetes and cancer in your age group. C) Hypertension is the leading cause of death in people your age. D) Hypertension greatly increases your risk of stroke and heart disease. D Feedback: Hypertension, particularly elevated systolic BP, increases the risk of death, stroke, and heart failure in people older than 50 years. Hypertension is not a direct precursor to pulmonary emboli, and it does not put older adults at increased risk of type 1 diabetes or cancer. It is not the leading cause of death in people 55 years of age. 36. The nurse is reviewing the medication administration record of a patient who takes a variety of medications for the treatment of hypertension. What potential therapeutic benefits of antihypertensives should the nurse identify? Select all that apply. A) Increased venous return B) Decreased peripheral resistance C) Decreased blood volume D) Decreased strength and rate of myocardial contractions E) Decreased blood viscosity B, C, D Feedback: The medications used for treating hypertension decrease peripheral resistance, blood volume, or the strength and rate of myocardial contraction. Antihypertensive medications do not increase venous return or decrease blood viscosity. 37. A newly diagnosed patient with hypertension is prescribed Diuril, a thiazide diuretic. What patient education should the nurse provide to this patient? A) Eat a banana every day because Diuril causes moderate hyperkalemia. B) Take over-the-counter potassium pills because Diuril causes your kidneys to lose potassium. C) Diuril can cause low blood pressure and dizziness, especially when you get up suddenly. D) Diuril increases sodium levels in your blood, so cut down on your salt. C Feedback: Thiazide diuretics can cause postural hypotension, which may be potentiated by alcohol, barbiturates, opioids, or hot weather. Diuril does not cause either moderate hyperkalemia or severe hypokalemia and it does not result in hypernatremia. 38. A patient in hypertensive urgency is admitted to the hospital. The nurse should be aware of what goal of treatment for a patient in hypertensive urgency? A) Normalizing BP within 2 hours B) Obtaining a BP of less than 110/70 mm Hg within 36 hours C) Obtaining a BP of less than 120/80 mm Hg within 36 hours D) Normalizing BP within 24 to 48 hours D Feedback: In cases of hypertensive urgency, oral agents can be administered with the goal of normalizing BP within 24 to 48 hours. For patients with this health problem, a BP of 120/80 mm Hg may be unrealistic. 39. A patients medication regimen for the treatment of hypertension includes hydrochlorothiazide. Following administration of this medication, the nurse should anticipate what effect? A) Drowsiness or lethargy B) Increased urine output C) Decreased heart rate D) Mild agitation B Feedback: Thiazide diuretics lower BP by reducing circulating blood volume; this results in a short-term increase in urine output. These drugs do not cause bradycardia, agitation, or drowsiness. 40. A patients recently elevated BP has prompted the primary care provider to prescribe furosemide (Lasix). The nurse should closely monitor which of the following? A) The clients oxygen saturation level B) The patients red blood cells, hematocrit, and hemoglobin C) The patients level of consciousness D) The patients potassium level D Feedback: Loop diuretics can cause potassium depletion. They do not normally affect level of consciousness, erythrocytes, or oxygen saturation. Chapter 32: Assessment of Hematologic Function and Treatment Modalities 1. A patient with a hematologic disorder asks the nurse how the body forms blood cells. The nurse should describe a process that takes place where? A) In the spleen B) In the kidneys C) In the bone marrow D) In the liver C Feedback: Bone marrow is the primary site for hematopoiesis. The liver and spleen may be involved during embryonic development or when marrow is destroyed. The kidneys release erythropoietin, which stimulates the marrow to increase production of red blood cells (RBCs). However, blood cells are not primarily formed in the spleen, kidneys, or liver. 2. A man suffers a leg wound which causes minor blood loss. As a result of bleeding, the process of primary hemostasis is activated. What occurs in primary hemostasis? A) Severed blood vessels constrict. B) Thromboplastin is released. C) Prothrombin is converted to thrombin. D) Fibrin is lysed. A Feedback: Primary hemostasis involves the severed vessel constricting and platelets collecting at the injury site. Secondary hemostasis occurs when thromboplastin is released, prothrombin converts to thrombin, and fibrin is lysed. 3. A patient has come to the OB/GYN clinic due to recent heavy menstrual flow. Because of the patients consequent increase in RBC production, the nurse knows that the patient may need to increase her daily intake of what substance? A) Vitamin E B) Vitamin D C) Iron D) Magnesium C Feedback: To replace blood loss, the rate of red cell production increases. Iron is incorporated into hemoglobin. Vitamins E and D and magnesium do not need to be increased when RBC production is increased. 4. The nurse is planning the care of a patient with a nutritional deficit and a diagnosis of megaloblastic anemia. The nurse should recognize that this patients health problem is due to what? A) Production of inadequate quantities of RBCs B) Premature release of immature RBCs C) Injury to the RBCs in circulation D) Abnormalities in the structure and function RBCs D Feedback: Vitamin B12 and folic acid deficiencies are characterized by the production of abnormally large erythrocytes called megaloblasts. Because these cells are abnormal, many are sequestered (trapped) while still in the bone marrow, and their rate of release is decreased. Some of these cells actually die in the marrow before they can be released into the circulation. This results in megaloblastic anemia. This pathologic process does not involve inadequate production, premature release, or injury to existing RBCs. 5. A nurse is caring for a patient who undergoing preliminary testing for a hematologic disorder. What sign or symptom most likely suggests a potential hematologic disorder? A) Sudden change in level of consciousness (LOC) B) Recurrent infections C) Anaphylaxis D) Severe fatigue D Feedback: The most common indicator of hematologic disease is extreme fatigue. This is more common than changes in LOC, infections, or analphylaxis. 6. The nurse caring for a patient receiving a transfusion notes that 15 minutes after the infusion of packed red blood cells (PRBCs) has begun, the patient is having difficulty breathing and complains of severe chest tightness. What is the most appropriate initial action for the nurse to take? A) Notify the patients physician. B) Stop the transfusion immediately. C) Remove the patients IV access. D) Assess the patients chest sounds and vital signs. B Feedback: Vascular collapse, bronchospasm, laryngeal edema, shock, fever, chills, and jugular vein distension are severe reactions. The nurse should discontinue the transfusion immediately, monitor the patients vital signs, and notify the physician. The blood container and tubing should be sent to the blood bank. A blood and urine specimen may be needed if a transfusion reaction or a bacterial infection is suspected. The patients IV access should not be removed. 7. The nurse is describing the role of plasminogen in the clotting cascade. Where in the body is plasminogen present? A) Myocardial muscle tissue B) All body fluids C) Cerebral tissue D) Venous and arterial vessel walls B Feedback: Plasminogen, which is present in all body fluids, circulates with fibrinogen. Plasminogen is found in body fluids, not tissue. 8. The nurse is caring for a patient who has developed scar tissue in many of the areas that normally produce blood cells. What organs can become active in blood cell production by the process of extramedullary hematopoiesis? A) Spleen and kidneys B) Kidneys and pancreas C) Pancreas and liver D) Liver and spleen D Feedback: In adults with disease that causes marrow destruction, fibrosis, or scarring, the liver and spleen can also resume production of blood cells by a process known as extramedullary hematopoiesis. The kidneys and pancreas do not produce blood cells for the body. 9. Through the process of hematopoiesis, stem cells differentiate into either myeloid or lymphoid stem cells. Into what do myeloid stem cells further differentiate? Select all that apply. A) Leukocytes B) Natural killer cells C) Cytokines D) Platelets E) Erythrocytes A, D, E Feedback: Myeloid stem cells differentiate into three broad cell types: erythrocytes, leukocytes, and platelets. Natural killer cells and cytokines do not originate as myeloid stem cells. 10. A patients wound has begun to heal and the blood clot which formed is no longer necessary. When a blood clot is no longer needed, the fibrinogen and fibrin will be digested by which of the following? A) Plasminogen B) Thrombin C) Prothrombin D) Plasmin D Feedback: The substance plasminogen is required to lyse (break down) the fibrin. Plasminogen, which is present in all body fluids, circulates with fibrinogen and is therefore incorporated into the fibrin clot as it forms. When the clot is no longer needed (e.g., after an injured blood vessel has healed), the plasminogen is activated to form plasmin. Plasmin digests the fibrinogen and fibrin. Prothrombin is converted to thrombin, which in turn catalyzes the conversion of fibrinogen to fibrin so a clot can form. 11. A patient undergoing a hip replacement has autologous blood on standby if a transfusion is needed. What is the primary advantage of autologous transfusions? A) Safe transfusion for patients with a history of transfusion reactions B) Prevention of viral infections from another persons blood C) Avoidance of complications in patients with alloantibodies D) Prevention of alloimmunization B Feedback: The primary advantage of autologous transfusions is the prevention of viral infections from another persons blood. Other secondary advantages include safe transfusion for patients with a history of transfusion reactions, prevention of alloimmunization, and avoidance of complications in patients with alloantibodies. 12. A patient has been diagnosed with a lymphoid stem cell defect. This patient has the potential for a problem involving which of the following? A) Plasma cells B) Neutrophils C) Red blood cells D) Platelets A Feedback: A defect in a myeloid stem cell can cause problems with erythrocyte, leukocyte, and platelet production. In contrast, a defect in the lymphoid stem cell can cause problems with T or B lymphocytes, plasma cells (a more differentiated form of B lymphocyte), or natural killer (NK) cells. 13. The nurse is describing normal RBC physiology to a patient who has a diagnosis of anemia. The nurse should explain that the RBCs consist primarily of which of the following? A) Plasminogen B) Hemoglobin C) Hematocrit D) Fibrin B Feedback: Mature erythrocytes consist primarily of hemoglobin, which contains iron and makes up 95% of the cell mass. RBCs are not made of fibrin or plasminogen. Hematocrit is a measure of RBC volume in whole blood. 14. The nurse educating a patient with anemia is describing the process of RBC production. When the patients kidneys sense a low level of oxygen in circulating blood, what physiologic response is initiated? A) Increased stem cell synthesis B) Decreased respiratory rate C) Arterial vasoconstriction D) Increased production of erythropoietin D Feedback: If the kidney detects low levels of oxygen, as occurs when fewer red cells are available to bind oxygen (i.e., anemia), erythropoietin levels increase. The body does not compensate with vasoconstriction, decreased respiration, or increased stem cell activity. 15. An older adult client is exhibiting many of the characteristic signs and symptoms of iron deficiency. In addition to a complete blood count, what diagnostic assessment should the nurse anticipate? A) Stool for occult blood B) Bone marrow biopsy C) Lumbar puncture D) Urinalysis A Feedback: Iron deficiency in the adult generally indicates blood loss (e.g., from bleeding in the GI tract or heavy menstrual flow). Bleeding in the GI tract can be preliminarily identified by testing stool for the presence of blood. A bone marrow biopsy would not be undertaken for the sole purpose of investigating an iron deficiency. Lumbar puncture and urinalysis would not be clinically relevant. 16. A patient is being treated for the effects of a longstanding vitamin B12 deficiency. What aspect of the patients health history would most likely predispose her to this deficiency? A) The patient has irregular menstrual periods. B) The patient is a vegan. C) The patient donated blood 60 days ago. D) The patient frequently smokes marijuana. B Feedback: Because vitamin B12 is found only in foods of animal origin, strict vegetarians may ingest little vitamin B12. Irregular menstrual periods, marijuana use, and blood donation would not precipitate a vitamin B12 deficiency. 17. The nurses review of a patients most recent blood work reveals a significant increase in the number of band cells. The nurses subsequent assessment should focus on which of the following? A) Respiratory function B) Evidence of decreased tissue perfusion C) Signs and symptoms of infection D) Recent changes in activity tolerance C Feedback: Ordinarily, band cells account for only a small percentage of circulating granulocytes, although their percentage can increase greatly under conditions in which neutrophil production increases, such as infection. This finding is not suggestive of problems with oxygenation and subsequent activity intolerance. 18. A nurse is educating a patient about the role of B lymphocytes. The nurses description will include which of the following physiologic processes? A) Stem cell differentiation B) Cytokine production C) Phagocytosis D) Antibody production D Feedback: B lymphocytes are capable of differentiating into plasma cells. Plasma cells, in turn, produce antibodies. Cytokines are produced by NK cells. Stem cell differentiation greatly precedes B lymphocyte production. 19. A patients most recent blood work reveals low levels of albumin. This assessment finding should suggest the possibility of what nursing diagnosis? A) Risk for imbalanced fluid volume related to low albumin B) Risk for infection related to low albumin C) Ineffective tissue perfusion related to low albumin D) Impaired skin integrity related to low albumin A Feedback: Albumin is particularly important for the maintenance of fluid balance within the vascular system. Deficiencies nearly always manifest as fluid imbalances. Tissue oxygenation and skin integrity are not normally affected. Low albumin does not constitute a risk for infection. 20. An individual has accidentally cut his hand, immediately initiating the process of hemostasis. Following vasoconstriction, what event in the process of hemostasis will take place? A) Fibrin will be activated at the bleeding site. B) Platelets will aggregate at the injury site. C) Thromboplastin will form a clot. D) Prothrombin will be converted to thrombin. B Feedback: Following vasoconstriction, circulating platelets aggregate at the site and adhere to the vessel and to one another, forming an unstable hemostatic plug. Events involved in the clotting cascade take place subsequent to this initial platelet action. 21. The nurse is providing care for an older adult who has a hematologic disorder. What age-related change in hematologic function should the nurse integrate into care planning? A) Bone marrow in older adults produces a smaller proportion of healthy, functional blood cells. B) Older adults are less able to increase blood cell production when demand suddenly increases. C) Stem cells in older adults eventually lose their ability to differentiate. D) The ratio of plasma to erythrocytes and lymphocytes increases with age. B Feedback: Due to a variety of factors, when an older person needs more blood cells, the bone marrow may not be able to increase production of these cells adequately. Stem cell activity continues throughout the lifespan, although at a somewhat decreased rate. The proportion of functional cells does not greatly decrease and the relative volume of plasma does not change significantly. 22. A clients health history reveals daily consumption of two to three bottles of wine. The nurse should plan assessments and interventions in light of the patients increased risk for what hematologic disorder? A) Leukemia B) Anemia C) Thrombocytopenia D) Lymphoma B Feedback: Heavy alcohol use is associated with numerous health problems, including anemia. Leukemia and lymphoma are not associated with alcohol use; RBC levels are typically affected more than platelet levels. 23. A patients diagnosis of atrial fibrillation has prompted the primary care provider to prescribe warfarin (Coumadin), an anticoagulant. When assessing the therapeutic response to this medication, what is the nurses most appropriate action? A) Assess for signs of myelosuppression. B) Review the patients platelet level. C) Assess the patients capillary refill time. D) Review the patients international normalized ratio (INR). D Feedback: The INR and aPTT serve as useful screening tools for evaluating a patients clotting ability and to monitor the therapeutic effectiveness of anticoagulant medications. The patients platelet level is not normally used as a short-term indicator of anticoagulation effectiveness. Assessing the patient for signs of myelosuppression and capillary refill time does not address the effectiveness of anticoagulants. 24. A patient has been scheduled for a bone marrow biopsy and admits to the nurse that she is worried about the pain involved with the procedure. What patient education is most accurate? A) Youll be given painkillers before the test, so there wont likely be any pain? B) Youll feel some pain when the needle enters your skin, but none when the needle enters the bone because of the absence of nerves in bone. C) Most people feel some brief, sharp pain when the needle enters the bone. D) Ill be there with you, and Ill try to help you keep your mind off the pain. C Feedback: Patients typically feel a pressure sensation as the needle is advanced into position. The actual aspiration always causes sharp, but brief pain, resulting from the suction exerted as the marrow is aspirated into the syringe; the patient should be warned about this. Stating, Ill try to help you keep your mind off the pain may increase the patients fears of pain, because this does not help the patient know what to expect. 25. A patient is scheduled for a splenectomy. During discharge education, what teaching point should the nurse prioritize? A) The importance of adhering to prescribed immunosuppressant therapy B) The need to report any signs or symptoms of infection promptly C) The need to ensure adequate folic acid, iron, and vitamin B12 intake D) The importance of limiting activity postoperatively to prevent hemorrhage B Feedback: After splenectomy, the patient is instructed to seek prompt medical attention if even relatively minor symptoms of infection occur. Often, patients with high platelet counts have even higher counts after splenectomy, which can predispose them to serious thrombotic or hemorrhagic problems. However, this increase is usually transient and therefore often does not warrant additional treatment. Dietary modifications are not normally necessary and immunosuppressants would be strongly contraindicated. 26. The nurses brief review of a patients electronic health record indicates that the patient regularly undergoes therapeutic phlebotomy. Which of the following rationales for this procedure is most plausible? A) The patient may chronically produce excess red blood cells. B) The patient may frequently experience a low relative plasma volume. C) The patient may have impaired stem cell function. D) The patient may previously have undergone bone marrow biopsy. A Feedback: Persistently elevated hematocrit is an indication for therapeutic phlebotomy. It is not used to address excess or deficient plasma volume and is not related to stem cell function. Bone marrow biopsy is not an indication for therapeutic phlebotomy. 27. A nurse has participated in organizing a blood donation drive at a local community center. Which of the following individuals would most likely be disallowed from donating blood? A) A man who is 81 years of age B) A woman whose blood pressure is 88/51 mm Hg C) A man who donated blood 4 months ago D) A woman who has type 1 diabetes B Feedback: For potential blood donors, systolic arterial BP should be 90 to 180 mm Hg, and the diastolic pressure should be 50 to 100 mm Hg. There is no absolute upper age limit. Donation 4 months ago does not preclude safe repeat donation and diabetes is not a contraindication. 28. A nurse at a blood donation clinic has completed the collection of blood from a woman. The woman states that she feels lightheaded and she appears visibly pale. What is the nurses most appropriate action? A) Help her into a sitting position with her head lowered below her knees. B) Administer supplementary oxygen by nasal prongs. C) Obtain a full set of vital signs. D) Inform a physician or other primary care provider. A Feedback: A donor who appears pale or complains of faintness should immediately lie down or sit with the head lowered below the knees. He or she should be observed for another 30 minutes. There is no immediate need for a physicians care. Supplementary oxygen may be beneficial, but may take too much time to facilitate before a syncopal episode. Repositioning must precede assessment of vital signs. 29. A patients low hemoglobin level has necessitated transfusion of PRBCs. Prior to administration, what action should the nurse perform? A) Have the patient identify his or her blood type in writing. B) Ensure that the patient has granted verbal consent for transfusion. C) Assess the patients vital signs to establish baselines. D) Facilitate insertion of a central venous catheter. C Feedback: Prior to a transfusion, the nurse must take the patients temperature, pulse, respiration, and BP to establish a baseline. Written consent is required and the patients blood type is determined by type and cross match, not by the patients self-declaration. Peripheral venous access is sufficient for blood transfusion. 30. A patient on the medical unit is receiving a unit of PRBCs. Difficult IV access has necessitated a slow infusion rate and the nurse notes that the infusion began 4 hours ago. What is the nurses most appropriate action? A) Apply an icepack to the blood that remains to be infused. B) Discontinue the remainder of the PRBC transfusion and inform the physician. C) Disconnect the bag of PRBCs, cool for 30 minutes and then administer. D) Administer the remaining PRBCs by the IV direct (IV push) route. B Feedback: Because of the risk of infection, a PRBC transfusion should not exceed 4 hours. Remaining blood should not be transfused, even if it is cooled. Blood is not administered by the IV direct route. 31. Two units of PRBCs have been ordered for a patient who has experienced a GI bleed. The patient is highly reluctant to receive a transfusion, stating, Im terrified of getting AIDS from a blood transfusion. How can the nurse best address the patients concerns? A) All the donated blood in the United States is treated with antiretroviral medications before it is used. B) That did happen in some high-profile cases in the twentieth century, but it is no longer a possibility. C) HIV was eradicated from the US blood supply in the early 2000s. D) The chances of contracting AIDS from a blood transfusion in the United States are exceedingly low. D Feedback: The patient can be reassured about the very low possibility of contracting HIV from the transfusion. However, it is not an absolute impossibility. Antiretroviral medications are not introduced into donated blood. The blood supply is constantly dynamic, due to the brief life of donated blood. 32. A patient is being treated in the ICU after a medical error resulted in an acute hemolytic transfusion reaction. What was the etiology of this patients adverse reaction? A) Antibodies to donor leukocytes remained in the blood. B) The donor blood was incompatible with that of the patient. C) The patient had a sensitivity reaction to a plasma protein in the blood. D) The blood was infused too quickly and overwhelmed the patients circulatory system. B Feedback: An acute hemolytic reaction occurs when the donor blood is incompatible with that of the recipient. In the case of a febrile nonhemolytic reaction, antibodies to donor leukocytes remain in the unit of blood or blood component. An allergic reaction is a sensitivity reaction to a plasma protein within the blood component. Hypervolemia does not cause an acute hemolytic reaction. 33. An interdisciplinary team has been commissioned to create policies and procedures aimed at preventing acute hemolytic transfusion reactions. What action has the greatest potential to reduce the risk of this transfusion reaction? A) Ensure that blood components are never infused at a rate greater than 125 mL/hr. B) Administer prophylactic antihistamines prior to all blood transfusions. C) Establish baseline vital signs for all patients receiving transfusions. D) Be vigilant in identifying the patient and the blood component. D Feedback: The most common causes of acute hemolytic reaction are errors in blood component labeling and patient identification that result in the administration of an ABO-incompatible transfusion. Actions to address these causes are necessary in all health care settings. Prophylactic antihistamines are not normally administered, and would not prevent acute hemolytic reactions. Similarly, baseline vital signs and slow administration will not prevent this reaction. 34. A patient is receiving a blood transfusion and complains of a new onset of slight dyspnea. The nurses rapid assessment reveals bilateral lung crackles and elevated BP. What is the nurses most appropriate action? A) Slow the infusion rate and monitor the patient closely. B) Discontinue the transfusion and begin resuscitation. C) Pause the transfusion and administer a 250 mL bolus of normal saline. D) Discontinue the transfusion and administer a beta-blocker, as ordered. A Feedback: The patient is showing early signs of hypervolemia; the nurse should slow the infusion rate and assess the patient closely for any signs of exacerbation. At this stage, discontinuing the transfusion is not necessary. A bolus would worsen the patients fluid overload. 35. A patient lives with a diagnosis of sickle cell anemia and receives frequent blood transfusions. The nurse should recognize the patients consequent risk of what complication of treatment? A) Hypovolemia B) Vitamin B12 deficiency C) Thrombocytopenia D) Iron overload D Feedback: Patients with chronic transfusion requirements can quickly acquire more iron than they can use, leading to iron overload. These individuals are not at risk for hypovolemia and there is no consequent risk for low platelet or vitamin B12 levels. 36. A patient is receiving the first of two ordered units of PRBCs. Shortly after the initiation of the transfusion, the patient complains of chills and experiences a sharp increase in temperature. What is the nurses priority action? A) Position the patient in high Fowlers. B) Discontinue the transfusion. C) Auscultate the patients lungs. D) Obtain a blood specimen from the patient. B Feedback: Stopping the transfusion is the first step in any suspected transfusion reaction. This must precede other assessments and interventions, including repositioning, chest auscultation, and collecting specimens. 37. Fresh-frozen plasma (FFP) has been ordered for a hospital patient. Prior to administration of this blood product, the nurse should prioritize what patient education? A) Infection risks associated with FFP administration B) Physiologic functions of plasma C) Signs and symptoms of a transfusion reaction D) Strategies for managing transfusion-associated anxiety C Feedback: Patients should be educated about signs and symptoms of transfusion reactions prior to administration of any blood product. In most cases, this is priority over education relating to infection. Anxiety may be an issue for some patients, but transfusion reactions are a possibility for all patients. Teaching about the functions of plasma is not likely a high priority. 38. The nurse is preparing to administer a unit of platelets to an adult patient. When administering this blood product, which of the following actions should the nurse perform? A) Administer the platelets as rapidly as the patient can tolerate. B) Establish IV access as soon as the platelets arrive from the blood bank. C) Ensure that the patient has a patent central venous catheter. D) Aspirate 10 to 15 mL of blood from the patients IV immediately following the transfusion. A Feedback: The nurse should infuse each unit of platelets as fast as patient can tolerate to diminish platelet clumping during administration. IV access should be established prior to obtaining the platelets from the blood bank. A central line is appropriate for administration, but peripheral IV access (22-gauge or larger) is sufficient. There is no need to aspirate after the transfusion. 39. Which of the following circumstances would most clearly warrant autologous blood donation? A) The patient has type-O blood. B) The patient has sickle cell disease or a thalassemia. C) The patient has elective surgery pending. D) The patient has hepatitis C. C Feedback: Autologous blood donation is useful for many elective surgeries where the potential need for transfusion is high. Type-O blood, hepatitis, sickle cell disease, and thalassemia are not clear indications for autologous donation. 40. A patients electronic health record states that the patient receives regular transfusions of factor IX. The nurse would be justified in suspecting that this patient has what diagnosis? A) Leukemia B) Hemophilia C) Hypoproliferative anemia D) Hodgkins lymphoma B Feedback: Administration of clotting factors is used to treat diseases where these factors are absent or insufficient; hemophilia is among the most common of these diseases. Factor IX is not used in the treatment of leukemia, lymphoma, or anemia. Chapter 33: Management of Patients With Nonmalignant Hematologic Disorders 1. A nurse is caring for a patient who has sickle cell anemia and the nurses assessment reveals the possibility of substance abuse. What is the nurses most appropriate action? A) Encourage the patient to rely on complementary and alternative therapies. B) Encourage the patient to seek care from a single provider for pain relief. C) Teach the patient to accept chronic pain as an inevitable aspect of the disease. D) Limit the reporting of emergency department visits to the primary health care provider. B Feedback: The patient should be encouraged to use a single primary health care provider to address health care concerns. Emergency department visits should be reported to the primary health care provider to achieve optimal management of the disease. It would inappropriate to teach the patient to simply accept his or her pain. Complementary therapies are usually insufficient to fully address pain in sickle cell disease. 2. A patient newly diagnosed with thrombocytopenia is admitted to the medical unit. After the admission assessment, the patient asks the nurse to explain the disease. What should the nurse explain to this patient? A) There could be an attack on the platelets by antibodies. B) There could be decreased production of platelets. C) There could be impaired communication between platelets. D) There could be an autoimmune process causing platelet malfunction. B Feedback: Thrombocytopenia can result from a decreased platelet production, increased platelet destruction, or increased consumption of platelets. Impaired platelet communication, antibodies, and autoimmune processes are not typical pathologies. 3. A critical care nurse is caring for a patient with autoimmune hemolytic anemia. The patient is not responding to conservative treatments, and his condition is now becoming life threatening. The nurse is aware that a treatment option in this case may include what? A) Hepatectomy B) Vitamin K administration C) Platelet transfusion D) Splenectomy D Feedback: A splenectomy may be the course of treatment if autoimmune hemolytic anemia does not respond to conservative treatment. Vitamin K administration is treatment for vitamin K deficiency and does not resolve anemia. Platelet transfusion may be the course of treatment for some bleeding disorders. Hepatectomy would not help the patient. 4. A nurse is providing education to a patient with iron deficiency anemia who has been prescribed iron supplements. What should the nurse include in health education? A) Take the iron with dairy products to enhance absorption. B) Increase the intake of vitamin E to enhance absorption. C) Iron will cause the stools to darken in color. D) Limit foods high in fiber due to the risk for diarrhea. C Feedback: The nurse will inform the patient that iron will cause the stools to become dark in color. Iron should be taken on an empty stomach, as its absorption is affected by food, especially dairy products. Patients should be instructed to increase their intake of vitamin C to enhance iron absorption. Foods high in fiber should be consumed to minimize problems with constipation, a common side effect associated with iron therapy. 5. The nurse is assessing a new patient with complaints of overwhelming fatigue and a sore tongue that is visibly smooth and beefy red. This patient is demonstrating signs and symptoms associated with what form of what hematologic disorder? A) Sickle cell anemia B) Hemophilia C) Megaloblastic anemia D) Thrombocytopenia C Feedback: A red, smooth, sore tongue is a symptom associated with megaloblastic anemia. Sickle cell disease, hemophilia, and thrombocytopenia do not have symptoms involving the tongue. 6. A patient with renal failure has decreased erythropoietin production. Upon analysis of the patients complete blood count, the nurse will expect which of the following results? A) An increased hemoglobin and decreased hematocrit B) A decreased hemoglobin and hematocrit C) A decreased mean corpuscular volume (MCV) and red cell distribution width (RDW) D) An increased MCV and RDW B Feedback: The decreased production of erythropoietin will result in a decreased hemoglobin and hematocrit. The patient will have normal MCV and RDW because the erythrocytes are normal in appearance. 7. A patient comes to the clinic complaining of fatigue and the health interview is suggestive of pica. Laboratory findings reveal a low serum iron level and a low ferritin level. With what would the nurse suspect that the patient will be diagnosed? A) Iron deficiency anemia B) Pernicious anemia C) Sickle cell anemia D) Hemolytic anemia A Feedback: A low serum iron level, a low ferritin level, and symptoms of pica are associated with iron deficiency anemia. TIBC may also be elevated. None of the other anemias are associated with pica. 8. A patient comes into the clinic complaining of fatigue. Blood work shows an increased bilirubin concentration and an increased reticulocyte count. What would the nurse suspect the patient has? A) A hypoproliferative anemia B) A leukemia C) Thrombocytopenia D) A hemolytic anemia D Feedback: In hemolytic anemias, premature destruction of erythrocytes results in the liberation of hemoglobin from the erythrocytes into the plasma; the released hemoglobin is converted in large part to bilirubin, and therefore the bilirubin concentration rises. The increased erythrocyte destruction leads to tissue hypoxia, which in turn stimulates erythropoietin production. This increased production is reflected in an increased reticulocyte count as the bone marrow responds to the loss of erythrocytes. Hypoproliferative anemias, leukemia, and thrombocytopenia lack this pathology and presentation. 9. A nurse is caring for a patient with severe anemia. The patient is tachycardic and complains of dizziness and exertional dyspnea. The nurse knows that in an effort to deliver more blood to hypoxic tissue, the workload on the heart is increased. What signs and symptoms might develop if this patient goes into heart failure? A) Peripheral edema B) Nausea and vomiting C) Migraine D) Fever A Feedback: Cardiac status should be carefully assessed in patients with anemia. When the hemoglobin level is low, the heart attempts to compensate by pumping faster and harder in an effort to deliver more blood to hypoxic tissue. This increased cardiac workload can result in such symptoms as tachycardia, palpitations, dyspnea, dizziness, orthopnea, and exertional dyspnea. Heart failure may eventually develop, as evidenced by an enlarged heart (cardiomegaly) and liver (hepatomegaly), and by peripheral edema. Nausea, migraine, and fever are not associated with heart failure. 10. A patient is admitted to the hospital with pernicious anemia. The nurse should prepare to administer which of the following medications? A) Folic acid B) Vitamin B12 C) Lactulose D) Magnesium sulfate B Feedback: Pernicious anemia is characterized by vitamin B12 deficiency. Magnesium sulfate, lactulose, and folic acid do not address the pathology of this type of anemia. 11. A patients blood work reveals a platelet level of 17,000/mm3. When inspecting the patients integumentary system, what finding would be most consistent with this platelet level? A) Dermatitis B) Petechiae C) Urticaria D) Alopecia B Feedback: When the platelet count drops to less than 20,000/mm3, petechiae can appear. Low platelet levels do not normally result in dermatitis, urticaria (hives), or alopecia (hair loss). 12. A nurse is admitting a patient with immune thrombocytopenic purpura to the unit. In completing the admission assessment, the nurse must be alert for what medications that potentially alter platelet function? Select all that apply. A) Antihypertensives B) Penicillins C) Sulfa-containing medications D) Aspirin-based drugs E) NSAIDs C, D, E Feedback: The nurse must be alert for sulfa-containing medications and others that alter platelet function (e.g., aspirin-based or other NSAIDs). Antihypertensive drugs and the penicillins do not alter platelet function. 13. A patient, 25 years of age, comes to the emergency department complaining of excessive bleeding from a cut sustained when cleaning a knife. Blood work shows a prolonged PT but a vitamin K deficiency is ruled out. When assessing the patient, areas of ecchymosis are noted on other areas of the body. Which of the following is the most plausible cause of the patients signs and symptoms? A) Lymphoma B) Leukemia C) Hemophilia D) Hepatic dysfunction D Feedback: Prolongation of the PT, unless it is caused by vitamin K deficiency, may indicate severe hepatic dysfunction. The majority of hemophiliacs are diagnosed as children. The scenario does not describe signs or symptoms of lymphoma or leukemia. 14. A patient with a history of cirrhosis is admitted to the ICU with a diagnosis of bleeding esophageal varices; an attempt to stop the bleeding has been only partially successful. What would the critical care nurse expect the care team to order for this patient? A) Packed red blood cells (PRBCs) B) Vitamin K C) Oral anticoagulants D) Heparin infusion A Feedback: Patients with liver dysfunction may have life-threatening hemorrhage from peptic ulcers or esophageal varices. In these cases, replacement with fresh frozen plasma, PRBCs, and platelets is usually required. Vitamin K may be ordered once the bleeding is stopped, but that is not what is needed to stop the bleeding of the varices. Anticoagulants would exacerbate the patients bleeding. 15. The nurse on the pediatric unit is caring for a 10-year-old boy with a diagnosis of hemophilia. The nurse knows that a priority nursing diagnosis for a patient with hemophilia is what? A) Hypothermia B) Diarrhea C) Ineffective coping D) Imbalanced nutrition: Less than body requirements C Feedback: Most patients with hemophilia are diagnosed as children. They often require assistance in coping with the condition because it is chronic, places restrictions on their lives, and is an inherited disorder that can be passed to future generations. Children with hemophilia are not at risk of hypothermia, diarrhea, or imbalanced nutrition. 16. A group of nurses are learning about the high incidence and prevalence of anemia among different populations. Which of the following individuals is most likely to have anemia? A) A 50-year-old African-American woman who is going through menopause B) An 81-year-old woman who has chronic heart failure C) A 48-year-old man who travels extensively and has a high-stress job D) A 13-year-old girl who has just experienced menarche B Feedback: The incidence and prevalence of anemia are exceptionally high among older adults, and the risk of anemia is compounded by the presence of heart disease. None of the other listed individuals exhibits high-risk factors for anemia, though exceptionally heavy menstrual flow can result in anemia. 17. An adult patient has been diagnosed with iron-deficiency anemia. What nursing diagnosis is most likely to apply to this patients health status? A) Risk for deficient fluid volume related to impaired erythropoiesis B) Risk for infection related to tissue hypoxia C) Acute pain related to uncontrolled hemolysis D) Fatigue related to decreased oxygen-carrying capacity D Feedback: Fatigue is the major assessment finding common to all forms of anemia. Anemia does not normally result in acute pain or fluid deficit. The patient may have an increased risk of infection due to impaired immune function, but fatigue is more likely. 18. A patient has been living with a diagnosis of anemia for several years and has experienced recent declines in her hemoglobin levels despite active treatment. What assessment finding would signal complications of anemia? A) Venous ulcers and visual disturbances B) Fever and signs of hyperkalemia C) Epistaxis and gastroesophageal reflux D) Ascites and peripheral edema D Feedback: A significant complication of anemia is heart failure from chronic diminished blood volume and the hearts compensatory effort to increase cardiac output. Patients with anemia should be assessed for signs and symptoms of heart failure, including ascites and peripheral edema. None of the other listed signs and symptoms is characteristic of heart failure. 19. A woman who is in her third trimester of pregnancy has been experiencing an exacerbation of iron- deficiency anemia in recent weeks. When providing the patient with nutritional guidelines and meal suggestions, what foods would be most likely to increase the womans iron stores? A) Salmon accompanied by whole milk B) Mixed vegetables and brown rice C) Beef liver accompanied by orange juice D) Yogurt, almonds, and whole grain oats C Feedback: Food sources high in iron include organ meats, other meats, beans (e.g., black, pinto, and garbanzo), leafy green vegetables, raisins, and molasses. Taking iron-rich foods with a source of vitamin C (e.g., orange juice) enhances the absorption of iron. All of the listed foods are nutritious, but liver and orange juice are most likely to be of benefit. 20. A patient with poorly controlled diabetes has developed end-stage renal failure and consequent anemia. When reviewing this patients treatment plan, the nurse should anticipate the use of what drug? A) Magnesium sulfate B) Epoetin alfa C) Low-molecular weight heparin D) Vitamin K B Feedback: The availability of recombinant erythropoietin (epoetin alfa [Epogen, Procrit], darbepoetin alfa [Aranesp]) has dramatically altered the management of anemia in end-stage renal disease. Heparin, vitamin K, and magnesium are not indicated in the treatment of renal failure or the consequent anemia. 21. A nurse is planning the care of a patient with a diagnosis of sickle cell disease who has been admitted for the treatment of an acute vaso-occlusive crisis. What nursing diagnosis should the nurse prioritize in the patients plan of care? A) Risk for disuse syndrome related to ineffective peripheral circulation B) Functional urinary incontinence related to urethral occlusion C) Ineffective tissue perfusion related to thrombosis D) Ineffective thermoregulation related to hypothalamic dysfunction C Feedback: There are multiple potential complications of sickle cell disease and sickle cell crises. Central among these, however, is the risk of thrombosis and consequent lack of tissue perfusion. Sickle cell crises are not normally accompanied by impaired thermoregulation or genitourinary complications. Risk for disuse syndrome is not associated with the effects of acute vaso-occlusive crisis. 22. A patient is being treated on the medical unit for a sickle cell crisis. The nurses most recent assessment reveals an oral temperature of 100.5F and a new onset of fine crackles on lung auscultation. What is the nurses most appropriate action? A) Apply supplementary oxygen by nasal cannula. B) Administer bronchodilators by nebulizer. C) Liaise with the respiratory therapist and consider high-flow oxygen. D) Inform the primary care provider that the patient may have an infection. D Feedback: Patients with sickle cell disease are highly susceptible to infection,thus any early signs of infection should be reported promptly. There is no evidence of respiratory distress, so oxygen therapy and bronchodilators are not indicated. 23. The medical nurse is aware that patients with sickle cell anemia benefit from understanding what situations can precipitate a sickle cell crisis. When teaching a patient with sickle cell anemia about strategies to prevent crises, what measures should the nurse recommend? A) Using prophylactic antibiotics and performing meticulous hygiene B) Maximizing physical activity and taking OTC iron supplements C) Limiting psychosocial stress and eating a high-protein diet D) Avoiding cold temperatures and ensuring sufficient hydration D Feedback: Keeping warm and providing adequate hydration can be effective in diminishing the occurrence and severity of attacks. Hygiene, antibiotics, and high protein intake do not prevent crises. Maximizing activity may exacerbate pain and be unrealistic. 24. A patient with a documented history of glucose-6-phosphate dehydrogenase deficiency has presented to the emergency department with signs and symptoms including pallor, jaundice, and malaise. Which of the nurses assessment questions relates most directly to this patients hematologic disorder? A) When did you last have a blood transfusion? B) What medications have taken recently? C) Have you been under significant stress lately? D) Have you suffered any recent injuries? B Feedback: Exacerbations of glucose-6-phosphate dehydrogenase deficiency are nearly always precipitated by medications. Blood transfusions, stress, and injury are less common triggers. 25. A patients electronic health record notes that he has previously undergone treatment for secondary polycythemia. How should this aspect of the patients history guide the nurses subsequent assessment? A) The nurse should assess for recent blood donation. B) The nurse should assess for evidence of lung disease. C) The nurse should assess for a history of venous thromboembolism. D) The nurse should assess the patient for impaired renal function. B Feedback: Any reduction in oxygenation, such as lung disease, can cause secondary polycythemia. Blood donation does not precipitate this problem and impaired renal function typically causes anemia, not polycythemia. A history of VTE is not a likely contributor. 26. A patients absolute neutrophil count (ANC) is 440/mm3. But the nurses assessment reveals no apparent signs or symptoms of infection. What action should the nurse prioritize when providing care for this patient? A) Meticulous hand hygiene B) Timely administration of antibiotics C) Provision of a nutrient-dense diet D) Maintaining a sterile care environment A Feedback: Providing care for a patient with neutropenia requires that the nurse adhere closely to standard precautions and infection control procedures. Hand hygiene is central to such efforts. Prophylactic antibiotics are rarely used and it is not possible to provide a sterile environment for care. Nutrition is highly beneficial, but hand hygiene is the central aspect of care. 27. A nurse is providing discharge education to a patient who has recently been diagnosed with a bleeding disorder. What topic should the nurse prioritize when teaching this patient? A) Avoiding buses, subways, and other crowded, public sites B) Avoiding activities that carry a risk for injury C) Keeping immunizations current D) Avoiding foods high in vitamin K B Feedback: Patients with bleeding disorders need to understand the importance of avoiding activities that increase the risk of bleeding, such as contact sports. Immunizations involve injections and may be contraindicated for some patients. Patients with bleeding disorders do not need to normally avoid crowds. Foods high in vitamin K may beneficial, not detrimental. 28. A nurse is a long-term care facility is admitting a new resident who has a bleeding disorder. When planning this residents care, the nurse should include which of the following? A) Housing the resident in a private room B) Implementing a passive ROM program to compensate for activity limitation C) Implementing of a plan for fall prevention D) Providing the patient with a high-fiber diet C Feedback: To prevent bleeding episodes, the nurse should ensure that an older adult with a bleeding disorder does not suffer a fall. Activity limitation is not necessarily required, however. A private room is not necessary and there is no reason to increase fiber intake. 29. The results of a patients most recent blood work and physical assessment are suggestive of immune thrombocytopenic purpura (ITP). This patient should undergo testing for which of the following potential causes? Select all that apply. A) Hepatitis B) Acute renal failure C) HIV D) Malignant melanoma E) Cholecystitis A, C Feedback: Viral illnesses have the potential to cause ITP. Renal failure, malignancies, and gall bladder inflammation are not typical causes of ITP. 30. A patient with a recent diagnosis of ITP has asked the nurse why the care team has not chosen to administer platelets, stating, I have low platelets, so why not give me a transfusion of exactly what Im missing? How should the nurse best respond? A) Transfused platelets usually arent beneficial because theyre rapidly destroyed in the body. B) A platelet transfusion often blunts your bodys own production of platelets even further. C) Finding a matching donor for a platelet transfusion is exceedingly difficult. D) A very small percentage of the platelets in a transfusion are actually functional. A Feedback: Despite extremely low platelet counts, platelet transfusions are usually avoided. Transfusions tend to be ineffective not because the platelets are nonfunctional but because the patients antiplatelet antibodies bind with the transfused platelets, causing them to be destroyed. Matching the patients blood type is not usually necessary for a platelet transfusion. Platelet transfusions do not exacerbate low platelet production. A client with several chronic health problems has been newly diagnosed with a qualitative platelet defect. What component of the patients previous medication regimen may have contributed to the development of this disorder? A) Calcium carbonate B) Vitamin B12 C) Aspirin D) Vitamin D C Feedback: Aspirin may induce a platelet disorder. Even small amounts of aspirin reduce normal platelet aggregation, and the prolonged bleeding time lasts for several days after aspirin ingestion. Calcium, vitamin D, and vitamin B12 do not have the potential to induce a platelet defect. 32. A young man with a diagnosis of hemophilia A has been brought to emergency department after suffering a workplace accident resulting in bleeding. Rapid assessment has revealed the source of the patients bleeding and established that his vital signs are stable. What should be the nurses next action? A) Position the patient in a prone position to minimize bleeding. B) Establish IV access for the administration of vitamin K. C) Prepare for the administration of factor VIII. D) Administer a normal saline bolus to increase circulatory volume. C Feedback: Injuries in patients with hemophilia necessitate prompt administration of clotting factors. Vitamin K is not a treatment modality and a prone position will not be appropriate for all types and locations of wounds. A normal saline bolus is not indicated. 33. A nurse is planning the care of a patient who has a diagnosis of hemophilia A. When addressing the nursing diagnosis of Acute Pain Related to Joint Hemorrhage, what principle should guide the nurses choice of interventions? A) Gabapentin (Neurontin) is effective because of the neuropathic nature of the patients pain. B) Opioids partially inhibit the patients synthesis of clotting factors. C) Opioids may cause vasodilation and exacerbate bleeding. D) NSAIDs are contraindicated due to the risk for bleeding. D Feedback: NSAIDs may be contraindicated in patients with hemophilia due to the associated risk of bleeding. Opioids do not have a similar effect and they do not inhibit platelet synthesis. The pain associated with hemophilia is not neuropathic. 34. A night nurse is reviewing the next days medication administration record (MAR) of a patient who has hemophilia. The nurse notes that the MAR specifies both oral and subcutaneous options for the administration of a PRN antiemetic. What is the nurses best action? A) Ensure that the day nurse knows not to give the antiemetic. B) Contact the prescriber to have the subcutaneous option discontinued. C) Reassess the patients need for antiemetics. D) Remove the subcutaneous route from the patients MAR. B Feedback: Injections must be avoided in patients with hemophilia. Consequently, the nurse should ensure that the prescriber makes the necessary change. The nurse cannot independently make a change to a patients MAR in most cases. Facilitating the necessary change is preferable to deferring to the day nurse. 35. A patient with Von Willebrand disease (vWD) has experienced recent changes in bowel function that suggest the need for a screening colonoscopy. What intervention should be performed in anticipation of this procedure? A) The patient should not undergo the normal bowel cleansing protocol prior to the procedure. B) The patient should receive a unit of fresh-frozen plasma 48 hours before the procedure. C) The patient should be admitted to the surgical unit on the day before the procedure. D) The patient should be given necessary clotting factors before the procedure. D Feedback: A goal of treating vWD is to replace the deficient protein (e.g., vWF or factor VIII) prior to an invasive procedure to prevent subsequent bleeding. Bowel cleansing is not contraindicated and FFP does not reduce the patients risk of bleeding. There may or may not be a need for preprocedure hospital admission. 36. A patients low prothrombin time (PT) was attributed to a vitamin K deficiency and the patients PT normalized after administration of vitamin K. When performing discharge education in an effort to prevent recurrence, what should the nurse emphasize? A) The need for adequate nutrition B) The need to avoid NSAIDs C) The need for constant access to factor concentrate D) The need for meticulous hygiene A Feedback: Vitamin K deficiency is often the result of a nutritional deficit. NSAIDs do not influence vitamin K synthesis and clotting factors are not necessary to treat or prevent a vitamin K deficiency. Hygiene is not related to the onset or prevention of vitamin K deficiency. 37. A patient with a history of atrial fibrillation has contacted the clinic saying that she has accidentally overdosed on her prescribed warfarin (Coumadin). The nurse should recognize the possible need for what antidote? A) IVIG B) Factor X C) Vitamin K D) Factor VIII C Feedback: Vitamin K is administered as an antidote for warfarin toxicity. 38. An intensive care nurse is aware of the need to identify patients who may be at risk of developing disseminated intravascular coagulation (DIC). Which of the following ICU patients most likely faces the highest risk of DIC? A) A patient with extensive burns B) A patient who has a diagnosis of acute respiratory distress syndrome C) A patient who suffered multiple trauma in a workplace accident D) A patient who is being treated for septic shock D Feedback: Sepsis is a common cause of DIC. A wide variety of acute illnesses can precipitate DIC, but sepsis is specifically identified as a cause. 39. A patient is being treated for DIC and the nurse has prioritized the nursing diagnosis of Risk for Deficient Fluid Volume Related to Bleeding. How can the nurse best determine if goals of care relating to this diagnosis are being met? A) Assess for edema. B) Assess skin integrity frequently. C) Assess the patients level of consciousness frequently. D) Closely monitor intake and output. D Feedback: The patient with DIC is at a high risk of deficient fluid volume. The nurse can best gauge the effectiveness of care by closely monitoring the patients intake and output. Each of the other assessments is a necessary element of care, but none addresses fluid balance as directly as close monitoring of intake and output. 40. A patient with a pulmonary embolism is being treated with a heparin infusion. What diagnostic finding suggests to the nurse that treatment is effective? A) The patients PT is within reference ranges. B) Arterial blood sampling tests positive for the presence of factor XIII. C) The patients platelet level is below 100,000/mm3. D) The patients activated partial thromboplastin time (aPTT) is 1.5 to 2.5 times the control value. D Feedback: The therapeutic effect of heparin is monitored by serial measurements of the aPTT; the dose is adjusted to maintain the range at 1.5 to 2.5 times the laboratory control. Heparin dosing is not determined on the basis of platelet levels, the presence or absence of clotting factors, or PT levels. Chapter 34: Management of Patients With Hematologic Neoplasms 1. An oncology nurse is providing health education for a patient who has recently been diagnosed with leukemia. What should the nurse explain about commonalities between all of the different subtypes of leukemia? A) The different leukemias all involve unregulated proliferation of white blood cells. B) The different leukemias all have unregulated proliferation of red blood cells and decreased bone marrow function. C) The different leukemias all result in a decrease in the production of white blood cells. D) The different leukemias all involve the development of cancer in the lymphatic system. A Feedback: Leukemia commonly involves unregulated proliferation of white blood cells. Decreased production of red blood cells is associated with anemias. Decreased production of white blood cells is associated with leukopenia. The leukemias are not characterized by their involvement with the lymphatic system. 2. A nurse is caring for a patient who has a diagnosis of acute leukemia. What assessment most directly addresses the most common cause of death among patients with leukemia? A) Monitoring for infection B) Monitoring nutritional status C) Monitor electrolyte levels D) Monitoring liver function A Feedback: In patients with acute leukemia, death typically occurs from infection or bleeding. Compromised nutrition, electrolyte imbalances, and impaired liver function are all plausible, but none is among the most common causes of death in this patient population. 3. An oncology nurse is caring for a patient with multiple myeloma who is experiencing bone destruction. When reviewing the patients most recent blood tests, the nurse should anticipate what imbalance? A) Hypercalcemia B) Hyperproteinemia C) Elevated serum viscosity D) Elevated RBC count A Feedback: Hypercalcemia may result when bone destruction occurs due to the disease process. Elevated serum viscosity occurs because plasma cells excrete excess immunoglobulin. RBC count will be decreased. Hyperproteinemia would not be present. 4. A nurse is planning the care of a patient who has been admitted to the medical unit with a diagnosis of multiple myeloma. In the patients care plan, the nurse has identified a diagnosis of Risk for Injury. What pathophysiologic effect of multiple myeloma most contributes to this risk? A) Labyrinthitis B) Left ventricular hypertrophy C) Decreased bone density D) Hypercoagulation C Feedback: Clients with multiple myeloma are at risk for pathologic bone fractures secondary to diffuse osteoporosis and osteolytic lesions. Labyrinthitis is uncharacteristic, and patients do not normally experience hypercoagulation or cardiac hypertrophy. 5. A patient with advanced leukemia is responding poorly to treatment. The nurse finds the patient tearful and trying to express his feelings, but he is clearly having difficulty. What is the nurses most appropriate action? A) Tell him that you will give him privacy and leave the room. B) Offer to call pastoral care. C) Ask if he would like you to sit with him while he collects his thoughts. D) Tell him that you can understand how hes feeling. C Feedback: Providing emotional support and discussing the uncertain future are crucial. Leaving is incorrect because leaving the patient doesnt show acceptance of his feelings. Offering to call pastoral care may be helpful for some patients but should be done after the nurse has spent time with the patient. Telling the patient that you understand how hes feeling is inappropriate because it doesnt help him express his feelings. 6. A nursing student is caring for a patient with acute myeloid leukemia who is preparing to undergo induction therapy. In preparing a plan of care for this patient, the student should assign the highest priority to which nursing diagnoses? A) Activity Intolerance B) Risk for Infection C) Acute Confusion D) Risk for Spiritual Distress B Feedback: Induction therapy places the patient at risk for infection, thus this is the priority nursing diagnosis. During the time of induction therapy, the patient is very ill, with bacterial, fungal, and occasional viral infections; bleeding and severe mucositis, which causes diarrhea; and marked decline in the ability to maintain adequate nutrition. Supportive care consists of administering blood products and promptly treating infections. Immobility, confusion, and spiritual distress are possible, but infection is the patients most acute physiologic threat. 7. A 77-year-old male is admitted to a unit with a suspected diagnosis of acute myeloid leukemia (AML). When planning this patients care, the nurse should be aware of what epidemiologic fact? A) Early diagnosis is associated with good outcomes. B) Five-year survival for older adults is approximately 50%. C) Five-year survival for patients over 75 years old is less than 2%. D) Survival rates are wholly dependent on the patients pre-illness level of health. C Feedback: The 5-year survival rate for patients with AML who are 50 years of age or younger is 43%; it drops to 19% for those between 50 and 64 years, and drops to1.6% for those older than 75 years. Early diagnosis is beneficial, but is nonetheless not associated with good outcomes or high survival rates. Preillness health is significant, but not the most important variable. 8. A 35-year-old male is admitted to the hospital complaining of severe headaches, vomiting, and testicular pain. His blood work shows reduced numbers of platelets, leukocytes, and erythrocytes, with a high proportion of immature cells. The nurse caring for this patient suspects a diagnosis of what? A) AML B) CML C) MDS D) ALL D Feedback: In acute lymphocytic leukemia (ALL), manifestations of leukemic cell infiltration into other organs are more common than with other forms of leukemia, and include pain from an enlarged liver or spleen, as well as bone pain. The central nervous system is frequently a site for leukemic cells; thus, patients may exhibit headache and vomiting because of meningeal involvement. Other extranodal sites include the testes and breasts. This particular presentation is not closely associated with acute myeloid leukemia (AML), chronic myeloid leukemia (CML), or myelodysplastic syndromes (MDS). 9. A patient with leukemia has developed stomatitis and is experiencing a nutritional deficit. An oral anesthetic has consequently been prescribed. What health education should the nurse provide to the patient? A) Chew with care to avoid inadvertently biting the tongue. B) Use the oral anesthetic 1 hour prior to meal time. C) Brush teeth before and after eating. D) Swallow slowly and deliberately. A Feedback: If oral anesthetics are used, the patient must be warned to chew with extreme care to avoid inadvertently biting the tongue or buccal mucosa. An oral anesthetic would be metabolized by the time the patient eats if it is used 1 hour prior to meals. There is no specific need to warn the patient about brushing teeth or swallowing slowly because an oral anesthetic has been used. 10. A patient diagnosed with acute myelogenous leukemia has just been admitted to the oncology unit. When writing this patients care plan, what potential complication should the nurse address? A) Pancreatitis B) Hemorrhage C) Arteritis D) Liver dysfunction B Feedback: Pancreatitis, arteritis, and liver dysfunction are generally not complications of leukemia. However, the patient faces a high risk of hemorrhage. 11. An emergency department nurse is triaging a 77-year-old man who presents with uncharacteristic fatigue as well as back and rib pain. The patient denies any recent injuries. The nurse should recognize the need for this patient to be assessed for what health problem? A) Hodgkin disease B) Non-Hodgkin lymphoma C) Multiple myeloma D) Acute thrombocythemia C Feedback: Back pain, which is often a presenting symptom in multiple myeloma, should be closely investigated in older patients. The lymphomas and bleeding disorders do not typically present with the primary symptom of back pain or rib pain. 12. A home health nurse is caring for a patient with multiple myeloma. Which of the following interventions should the nurse prioritize when addressing the patients severe bone pain? A) Implementing distraction techniques B) Educating the patient about the effective use of hot and cold packs C) Teaching the patient to use NSAIDs effectively D) Helping the patient manage the opioid analgesic regimen D Feedback: For severe pain resulting from multiple myeloma, opioids are likely necessary. NSAIDs would likely be ineffective and are associated with significant adverse effects. Hot and cold packs as well as distraction would be insufficient for severe pain. 13. A nurse is caring for a patient with Hodgkin lymphoma at the oncology clinic. The nurse should be aware of what main goal of care? A) Cure of the disease B) Enhancing quality of life C) Controlling symptoms D) Palliation A Feedback: The goal in the treatment of Hodgkin lymphoma is cure. Palliation is thus not normally necessary. Quality of life and symptom control are vital, but the overarching goal is the cure the disease. 14. A patient with non-Hodgkins lymphoma is receiving information from the oncology nurse. The patient asks the nurse why she should stop drinking and smoking and stay out of the sun. What would be the nurses best response? A) Everyone should do these things because theyre health promotion activities that apply to everyone. B) You dont want to develop a second cancer, do you? C) You need to do this just to be on the safe side. D) Its important to reduce other factors that increase the risk of second cancers. D Feedback: The nurse should encourage patients to reduce other factors that increase the risk of developing second cancers, such as use of tobacco and alcohol and exposure to environmental carcinogens and excessive sunlight. The other options do not answer the patients question, and also make light of the patients question. 15. An adult patient has presented to the health clinic with a complaint of a firm, painless cervical lymph node. The patient denies any recent infectious diseases. What is the nurses most appropriate response to the patients complaint? A) Call 911. B) Promptly refer the patient for medical assessment. C) Facilitate a radiograph of the patients neck and have the results forwarded to the patients primary care provider. D) Encourage the patient to track the size of the lymph node and seek care in 1 week. B Feedback: Hodgkin lymphoma usually begins as an enlargement of one or more lymph nodes on one side of the neck. The individual nodes are painless and firm but not hard. Prompt medical assessment is necessary if a patient has this presentation. However, there is no acute need to call 911. Delaying care for 1 week could have serious consequences and x-rays are not among the common diagnostic tests. 16. A nurse practitioner is assessing a patient who has a fever, malaise, and a white blood cell count that is elevated. Which of the following principles should guide the nurses management of the patients care? A) There is a need for the patient to be assessed for lymphoma. B) Infection is the most likely cause of the patients change in health status. C) The patient is exhibiting signs and symptoms of leukemia. D) The patient should undergo diagnostic testing for multiple myeloma. B Feedback: Leukocytosis is most often the result of infection. It is only considered pathologic (and suggestive of leukemia) if it is persistent and extreme. Multiple myeloma and lymphoma are not likely causes of this constellation of symptoms. 17. Diagnostic testing has resulted in a diagnosis of acute myeloid leukemia (AML) in an adult patient who is otherwise healthy. The patient and the care team have collaborated and the patient will soon begin induction therapy. The nurse should prepare the patient for which of the following? A) Daily treatment with targeted therapy medications B) Radiation therapy on a daily basis C) Hematopoietic stem cell transplantation D) An aggressive course of chemotherapy D Feedback: Attempts are made to achieve remission of AML by the aggressive administration of chemotherapy, called induction therapy, which usually requires hospitalization for several weeks. Induction therapy is not synonymous with radiation, stem cell transplantation, or targeted therapies. 18. A patient with a diagnosis of acute myeloid leukemia (AML) is being treated with induction therapy on the oncology unit. What nursing action should be prioritized in the patients care plan? A) Protective isolation and vigilant use of standard precautions B) Provision of a high-calorie, low-texture diet and appropriate oral hygiene C) Including the family in planning the patients activities of daily living D) Monitoring and treating the patients pain A Feedback: Induction therapy causes neutropenia and a severe risk of infection. This risk must be addressed directly in order to ensure the patients survival. For this reason, infection control would be prioritized over nutritional interventions, family care, and pain, even though each of these are important aspects of nursing care. 19. A nurse is caring for a patient who has been diagnosed with leukemia. The nurses most recent assessment reveals the presence of ecchymoseson the patients sacral area and petechiae in her forearms. In addition to informing the patients primary care provider, the nurse should perform what action? A) Initiate measures to prevent venous thromboembolism (VTE). B) Check the patients most recent platelet level. C) Place the patient on protective isolation. D) Ambulate the patient to promote circulatory function. B Feedback: The patients signs are suggestive of thrombocytopenia, thus the nurse should check the patients most recent platelet level. VTE is not a risk and this does not constitute a need for isolation. Ambulation and activity may be contraindicated due to the risk of bleeding. 20. A 60-year-old patient with chronic myeloid leukemia will be treated in the home setting and the nurse is preparing appropriate health education. What topic should the nurse emphasize? A) The importance of adhering to the prescribed drug regimen B) The need to ensure that vaccinations are up to date C) The importance of daily physical activity D) The need to avoid shellfish and raw foods A Feedback: Nurses need to understand that the effectiveness of the drugs used to treat CML is based on the ability of the patient to adhere to the medication regimen as prescribed. Adherence is often incomplete, thus this must be a focus of health education. Vaccinations normally would not be administered during treatment and daily physical activity may be impossible for the patient. Dietary restrictions are not normally necessary. 21. An older adult patient is undergoing diagnostic testing for chronic lymphocytic leukemia (CLL). What assessment finding is certain to be present if the patient has CLL? A) Increased numbers of blast cells B) Increased lymphocyte levels C) Intractable bone pain D) Thrombocytopenia with no evidence of bleeding B Feedback: An increased lymphocyte count (lymphocytosis) is always present in patients with CLL. Each of the other listed symptoms may or may not be present, and none is definitive for CLL. 22. A patient has been found to have an indolent neoplasm. The nurse should recognize what implication of this condition? A) The patient faces a significant risk of malignancy. B) The patient has a myeloid form of leukemia. C) The patient has a lymphocytic form of leukemia. D) The patient has a major risk factor for hemophilia. A Feedback: Indolent neoplasms have the potential to develop into a neoplasm, but this is not always the case. The patient does not necessary have, or go on to develop, leukemia. Indolent neoplasms are unrelated to the pathophysiology of hemophilia. 23. A nurse is caring for a patient who is being treated for leukemia in the hospital. The patient was able to maintain her nutritional status for the first few weeks following her diagnosis but is now exhibiting early signs and symptoms of malnutrition. In collaboration with the dietitian, the nurse should implement what intervention? A) Arrange for total parenteral nutrition (TPN). B) Facilitate placement of a percutaneous endoscopic gastrostomy (PEG) tube. C) Provide the patient with several small, soft-textured meals each day. D) Assign responsibility for the patients nutrition to the patients friends and family. C Feedback: For patients experiencing difficulties with oral intake, the provision of small, easily chewed meals may be beneficial. This option would be trialed before resorting to tube feeding or TPN. The family should be encouraged to participate in care, but should not be assigned full responsibility. 24. A patient who is undergoing consolidation therapy for the treatment of leukemia has been experiencing debilitating fatigue. How can the nurse best meet this patients needs for physical activity? A) Teach the patient about the risks of immobility and the benefits of exercise. B) Assist the patient to a chair during awake times, as tolerated. C) Collaborate with the physical therapist to arrange for stair exercises. D) Teach the patient to perform deep breathing and coughing exercises. B Feedback: Sitting is a chair is preferable to bed rest, even if a patient is experiencing severe fatigue. A patient who has debilitating fatigue would not likely be able to perform stair exercises. Teaching about mobility may be necessary, but education must be followed by interventions that actually involve mobility. Deep breathing and coughing reduce the risk of respiratory complications but are not substitutes for physical mobility in preventing deconditioning. 25. An oncology nurse recognizes a patients risk for fluid imbalance while the patient is undergoing treatment for leukemia. What relevant assessments should the nurse include in the patients plan of care? Select all that apply. A) Monitoring the patients electrolyte levels B) Monitoring the patients hepatic function C) Measuring the patients weight on a daily basis D) Measuring and recording the patients intake and output E) Auscultating the patients lungs frequently A, C, D, E Feedback: Assessments that relate to fluid balance include monitoring the patients electrolytes, auscultating the patients chest for adventitious sounds, weighing the patient daily, and closely monitoring intake and output. Liver function is not directly relevant to the patients fluid status in most cases. 26. After receiving a diagnosis of acute lymphocytic leukemia, a patient is visibly distraught, stating, I have no idea where to go from here. How should the nurse prepare to meet this patients psychosocial needs? A) Assess the patients previous experience with the health care system. B) Reassure the patient that treatment will be challenging but successful. C) Assess the patients specific needs for education and support. D) Identify the patients plan of medical care. C Feedback: In order to meets the patients needs, the nurse must first identify the specific nature of these needs. According to the nursing process, assessment must precede interventions. The plan of medical care is important, but not central to the provision of support. The patients previous health care is not a primary consideration, and the nurse cannot assure the patient of successful treatment. 27. A patient has completed the full course of treatment for acute lymphocytic leukemia and has failed to respond appreciably. When preparing for the patients subsequent care, the nurse should perform what action? A) Arrange a meeting between the patients family and the hospital chaplain. B) Assess the factors underlying the patients failure to adhere to the treatment regimen. C) Encourage the patient to vigorously pursue complementary and alternative medicine (CAM). D) Identify the patients specific wishes around end-of-life care. D Feedback: Should the patient not respond to therapy, it is important to identify and respect the patients choices about treatment, including measures to prolong life and other end-of-life measures. The patient may or may not be open to pursuing CAM. Unsuccessful treatment is not necessarily the result of failure to adhere to the treatment plan. Assessment should precede meetings with a chaplain, which may or may not be beneficial to the patient and congruent with the familys belief system. 28. Following an extensive diagnostic workup, an older adult patient has been diagnosed with a secondary myelodysplastic syndrome (MDS). What assessment question most directly addresses the potential etiology of this patients health problem? A) Were you ever exposed to toxic chemicals in any of the jobs that you held? B) When you were younger, did you tend to have recurrent infections of any kind? C) Have your parents or siblings had any disease like this? D) Would you say that youve had a lot of sun exposure in your lifetime? A Feedback: Secondary MDS can occur at any age and results from prior toxic exposure to chemicals, including chemotherapeutic medications. Family history, sun exposure, and previous infections are unrelated to the pathophysiology of secondary MDS. 29. A patient with a myelodysplastic syndrome is being treated on the medical unit. What assessment finding should prompt the nurse to contact the patients primary care provider? A) The patient is experiencing a frontal lobe headache. B) The patient has an episode of urinary incontinence. C) The patient has an oral temperature of 37.5C (99.5F). D) The patients SpO2 is 91% on room air. C Feedback: Because the patient with MDS is at a high risk for infection, any early signs of infection must be reported promptly. The nurse should address each of the listed assessment findings, but none is as direct a threat to the patients immediate health as an infection. 30. A nurse is preparing health education for a patient who has received a diagnosis of myelodysplastic syndrome (MDS). Which of the following topics should the nurse prioritize? A) Techniques for energy conservation and activity management B) Emergency management of bleeding episodes C) Technique for the administration of bronchodilators by metered-dose inhaler D) Techniques for self-palpation of the lymph nodes B Feedback: Because of patients risks of hemorrhage, patients with MDS should be taught techniques for managing emergent bleeding episodes. Bronchodilators are not indicated for the treatment of MDS and lymphedema is not normally associated with the disease. Energy conservation techniques are likely to be useful, but management of hemorrhage is a priority because of the potential consequences. 31. A clinic patient is being treated for polycythemia vera and the nurse is providing health education. What practice should the nurse recommend in order to prevent the complications of this health problem? A) Avoiding natural sources of vitamin K B) Avoiding altitudes of 1500 feet (457 meters) C) Performing active range of motion exercises daily D) Avoiding tight and restrictive clothing on the legs D Feedback: Because of the risk of DVT, patients with polycythemia vera should avoid tight and restrictive clothing. There is no need to avoid foods with vitamin K or to avoid higher altitudes. Activity levels should be maintained, but there is no specific need for ROM exercises. 32. A clinic nurse is working with a patient who has a long-standing diagnosis of polycythemia vera. How can the nurse best gauge the course of the patients disease? A) Document the color of the patients palms and face during each visit. B) Follow the patients erythrocyte sedimentation rate over time. C) Document the patients response to erythropoietin injections. D) Follow the trends of the patients hematocrit. D Feedback: The course of polycythemia vera can be best ascertained by monitoring the patients hematocrit, which should remain below 45%. Erythropoietin injections would exacerbate the condition. Skin tone should be observed, but is a subjective assessment finding. The patients ESR is not relevant to the course of the disease. 33. A nurse is planning the care of a patient who has been diagnosed with essential thrombocythemia (ET). What nursing diagnosis should the nurse prioritize when choosing interventions? A) Risk for Ineffective Tissue Perfusion B) Risk for Imbalanced Fluid Volume C) Risk for Ineffective Breathing Pattern D) Risk for Ineffective Thermoregulation A Feedback: Patients with ET are at risk for hypercoagulation and consequent ineffective tissue perfusion. Fluid volume, breathing, and thermoregulation are not normally affected. 34. A nurse at a long-term care facility is amending the care plan of a resident who has just been diagnosed with essential thrombocythemia (ET). The nurse should anticipate the administration of what medication? A) Dalteparin B) Allopurinol C) Hydroxyurea D) Hydrochlorothiazide C Feedback: Hydroxyurea is effective in lowering the platelet count for patients with ET. Dalteparin, allopurinol, and HCTZ do not have this therapeutic effect. 35. A nurse is writing the care plan of a patient who has been diagnosed with myelofibrosis. What nursing diagnoses should the nurse address? Select all that apply. Disturbed Body Image A) B) Impaired Mobility C) Imbalanced Nutrition: Less than Body Requirements D) Acute Confusion E) Risk for Infection A, B, C, E Feedback: The profound splenomegaly that accompanies myelofibrosis can impact the patients body image and mobility. As well, nutritional deficits are common and the patient is at risk for infection. Cognitive effects are less common. 36. An adult patients abnormal complete blood count (CBC) and physical assessment have prompted the primary care provider to order a diagnostic workup for Hodgkin lymphoma. The presence of what assessment finding is considered diagnostic of the disease? A) Schwann cells B) Reed-Sternberg cells C) Lewy bodies D) Loops of Henle B Feedback: The malignant cell of Hodgkin lymphoma is the Reed-Sternberg cell, a gigantic tumor cell that is morphologically unique and thought to be of immature lymphoid origin. It is the pathologic hallmark and essential diagnostic criterion. Schwann cells exist in the peripheral nervous system and Lewy bodies are markers of Parkinson disease. Loops of Henle exist in nephrons. 37. A young adult patient has received the news that her treatment for Hodgkin lymphoma has been deemed successful and that no further treatment is necessary at this time. The care team should ensure that the patient receives regular health assessments in the future due to the risk of what complication? A) Iron-deficiency anemia B) Hemophilia C) Hematologic cancers D) Genitourinary cancers C Feedback: Survivors of Hodgkin lymphoma have a high risk of second cancers, with hematologic cancers being the most common. There is no consequent risk of anemia or hemophilia, and hematologic cancers are much more common than GU cancers. 38. The clinical nurse educator is presenting health promotion education to a patient who will be treated for non-Hodgkin lymphoma on an outpatient basis. The nurse should recommend which of the following actions? A) Avoiding direct sun exposure in excess of 15 minutes daily B) Avoiding grapefruit juice and fresh grapefruit C) Avoiding highly crowded public places D) Using an electric shaver rather than a razor C Feedback: The risk of infection is significant for these patients, not only from treatment-related myelosuppression but also from the defective immune response that results from the disease itself. Limiting infection exposure is thus necessary. The need to avoid grapefruit is dependent on the patients medication regimen. Sun exposure and the use of razors are not necessarily contraindicated. 39. A patient has a diagnosis of multiple myeloma and the nurse is preparing health education in preparation for discharge from the hospital. What action should the nurse promote? A) Daily performance of weight-bearing exercise to prevent muscle atrophy B) Close monitoring of urine output and kidney function C) Daily administration of warfarin (Coumadin) as ordered D) Safe use of supplementary oxygen in the home setting B Feedback: Renal function must be monitored closely in the patient with multiple myeloma. Excessive weight- bearing can cause pathologic fractures. There is no direct indication for anticoagulation or supplementary oxygen. 40. A nurse is caring for patient whose diagnosis of multiple myeloma is being treated with bortezomib. The nurse should assess for what adverse effect of this treatment? A) Stomatitis B) Nephropathy C) Cognitive changes D) Peripheral neuropathy D Feedback: A significant toxicity associated with the use of bortezomib for multiple myeloma is peripheral neuropathy. Stomatitis, cognitive changes, and nephropathy are not noted to be adverse effects of this medication. Chapter 35: Assessment of Immune Function 1. A woman has been diagnosed with breast cancer and is being treated aggressively with a chemotherapeutic regimen. As a result of this regimen, she has an inability to fight infection due to the fact that her bone marrow is unable to produce a sufficient amount of what? A) Lymphocytes B) Cytoblasts C) Antibodies D) Capillaries A Feedback: The white blood cells involved in immunity (including lymphocytes) are produced in the bone marrow. Cytoblasts are the protoplasm of the cell outside the nucleus. Antibodies are produced by lymphocytes, but not in the bone marrow. Capillaries are small blood vessels 2. During a mumps outbreak at a local school, a patient, who is a school teacher, is exposed. She has previously been immunized for mumps. What type of immunity does she possess? A) Acquired immunity B) Natural immunity C) Phagocytic immunity D) Humoral immunity A Feedback: Acquired immunity usually develops as a result of prior exposure to an antigen, often through immunization. When the body is attacked by bacteria, viruses, or other pathogens, it has three means of defense. The first line of defense, the phagocytic immune response, involves the WBCs that have the ability to ingest foreign particles. A second protective response is the humoral immune response, which begins when the B lymphocytes transform themselves into plasma cells that manufacture antibodies. The natural immune response system is rapid, nonspecific immunity present at birth. 3. A gardener sustained a deep laceration while working and requires sutures. The patient is asked about the date of her last tetanus shot, which is over 10 years ago. Based on this information, the patient will receive a tetanus immunization. The tetanus injection will allow for the release of what? A) Antibodies B) Antigens C) Cytokines D) Phagocytes A Feedback: Immunizations activate the humoral immune response, culminating in antibody production. Antigens are the substances that induce the production of antibodies. Immunizations do not prompt cytokine or phagocyte production. 4. An infection control nurse is presenting an inservice reviewing the immune response. The nurse describes the clumping effect that occurs when an antibody acts like a cross-link between two antigens. What process is the nurse explaining? A) Agglutination B) Cellular immune response C) Humoral response D) Phagocytic immune response A Feedback: Agglutination refers to the clumping effect occurring when an antibody acts as a cross-link between two antigens. This takes place within the context of the humoral immune response, but is not synonymous with it. Cellular immune response, the immune systems third line of defense, involves the attack of pathogens by T-cells. The phagocytic immune response, or immune response, is the systems first line of defense, involving white blood cells that have the ability to ingest foreign particles. 5. A nurse has administered a childs scheduled vaccination for rubella. This vaccination will cause the child to develop which of the following? A) Natural immunity B) Active acquired immunity C) Cellular immunity D) Mild hypersensitivity B Feedback: Active acquired immunity usually develops as a result of vaccination or contracting a disease. Natural immunity is present at birth and provides a nonspecific response to any foreign invader. Immunizations do not activate the process of cellular immunity. Hypersensitivity is not an expected outcome of immunization. 6. A patient with a history of dermatitis takes corticosteroids on a regular basis. The nurse should assess the patient for which of the following complications of therapy? A) Immunosuppression B) Agranulocytosis C) Anemia D) Thrombocytopenia A Feedback: Corticosteroids such as prednisone can cause immunosuppression. Corticosteroids do not typically cause agranulocytosis, anemia, or low platelet counts. 7. A nurse is planning the assessment of a patient who is exhibiting signs and symptoms of an autoimmune disorder. The nurse should be aware that the incidence and prevalence of autoimmune diseases is known to be higher among what group? A) Young adults B) Native Americans C) Women D) Hispanics C Feedback: Many autoimmune diseases have a higher incidence in females than in males, a phenomenon believed to be correlated with sex hormones. 8. A 16-year-old has been brought to the emergency department by his parents after falling through the glass of a patio door, suffering a laceration. The nurse caring for this patient knows that the site of the injury will have an invasion of what? A) Interferons B) Phagocytic cells C) Apoptosis D) Cytokines B Feedback: Monocytes migrate to injury sites and function as phagocytic cells, engulfing, ingesting, and destroying greater numbers and quantities of foreign bodies or toxins than granulocytes. This occurs in response to the foreign bodies that have invaded the laceration from the dirt on the broken glass. Interferon, one type of biologic response modifier, is a nonspecific viricidal protein that is naturally produced by the body and is capable of activating other components of the immune system. Apoptosis, or programmed cell death, is the bodys way of destroying worn out cells such as blood or skin cells or cells that need to be renewed. Cytokines are the various proteins that mediate the immune response. These do not migrate to injury sites. A man was scratched by an old tool and developed a virulent staphylococcus infection. In the course of the mans immune response, circulating lymphocytes containing the antigenic message returned to the nearest lymph node. During what stage of the immune response did this occur? A) Recognition stage B) Proliferation stage C) Response stage D) Effector stage B Feedback: The recognition stage of antigens as foreign by the immune system is the initiating event in any immune response. The body must first recognize invaders as foreign before it can react to them. In the proliferation stage, the circulating lymphocyte containing the antigenic message returns to the nearest lymph node. Once in the node, the sensitized lymphocyte stimulates some of the resident dormant T and B lymphocytes to enlarge, divide, and proliferate. In the response stage, the differentiated lymphocytes function either in a humoral or a cellular capacity. In the effector stage, either the antibody of the humoral response or the cytotoxic (killer) T cell of the cellular response reaches and connects with the antigen on the surface of the foreign invader. 10. A patient with cystic fibrosis has received a double lung transplant and is now experiencing signs of rejection. What is the immune response that predominates in this situation? A) Humoral B) Nonspecific C) Cellular D) Mitigated C Feedback: Most immune responses to antigens involve both humoral and cellular responses, although only one predominates. During transplantation rejection, the cellular response predominates over the humoral response. Neither a mitigated nor nonspecific cell response is noted in this situation. 11. A patient is being treated for bacterial pneumonia. In the first stages of illness, the patients dyspnea was accompanied by a high fever. Currently, the patient claims to be feeling better and is afebrile. The patient is most likely in which stage of the immune response? A) Recognition stage B) Proliferation stage C) Response stage D) Effector stage D Feedback: The immune response culminates with the effector stage, during which offending microorganisms are killed by the various actions of the immune system. The patients improvement in health status is likely the result of this final stage in the immune response. 12. The nurse is providing care for a patient who has multiple sclerosis. The nurse recognizes the autoimmune etiology of this disease and the potential benefits of what treatment? A) Stem cell transplantation B) Serial immunizations C) Immunosuppression D) Genetic engineering A Feedback: Clinical trials using stem cells are under way in patients with a variety of disorders having an autoimmune component, including multiple sclerosis. Immunizations and genetic engineering are not used to treat multiple sclerosis. Immunosuppression would exacerbate symptoms of MS. 13. A patients injury has initiated an immune response that involves inflammation. What are the first cells to arrive at a site of inflammation? A) Eosinophils B) Red blood cells C) Lymphocytes D) Neutrophils D Feedback: Neutrophils are the first cells to arrive at the site where inflammation occurs. Eosinophils increase in number during allergic reactions and stress responses, but are not always present during inflammation. RBCs do not migrate during an immune response. Lymphocytes become active but do not migrate to the site of inflammation. A nurse is planning a patients care and is relating it to normal immune response. During what stage of the immune response should the nurse know that antibodies or cytotoxic T cells combine and destroy the invading microbes? A) Recognition stage B) Proliferation stage C) Response stage D) Effector stage D Feedback: In the effector stage, either the antibody of the humoral response or the cytotoxic (killer) T cell of the cellular response reaches and couples with the antigen on the surface of the foreign invader. The coupling initiates a series of events that in most instances results in total destruction of the invading microbes or the complete neutralization of the toxin. This does not take place during the three preceding stages. 15. The nurse should recognize a patients risk for impaired immune function if the patient has undergone surgical removal of which of the following? A) Thyroid gland B) Spleen C) Kidney D) Pancreas B Feedback: A history of surgical removal of the spleen, lymph nodes, or thymus may place the patient at risk for impaired immune function. Removal of the thyroid, kidney, or pancreas would not directly lead to impairment of the immune system. 16. A nurse is admitting a patient who exhibits signs and symptoms of a nutritional deficit. Inadequate intake of what nutrient increases a patients susceptibility to infection? A) Vitamin B12 B) Unsaturated fats C) Proteins D) Complex carbohydrates C Feedback: Depletion of protein reserves results in atrophy of lymphoid tissues, depression of antibody response, reduction in the number of circulating T cells, and impaired phagocytic function. As a result, the patient has an increased susceptibility to infection. Low intake of fat and vitamin B12affects health, but is not noted to directly create a risk for infection. Low intake of complex carbohydrates is not noted to constitute a direct risk factor for infection. 17. A nurse has admitted a patient who has been diagnosed with urosepsis. What immune response predominates in sepsis? A) Mitigated B) Nonspecific C) Cellular D) Humoral D Feedback: Most immune responses to antigens involve both humoral and cellular responses, although only one predominates. For example, during transplantation rejection, the cellular response predominates, whereas in the bacterial pneumonias and sepsis, the humoral response plays the dominant role. Neither mitigated nor nonspecific cell response is noted in this situation. 18. A patient is admitted with cellulitis and experiences a consequent increase in white blood cell count. The nurse is aware that during the immune response, pathogens are engulfed by white blood cells that ingest foreign particles. What is this process known as? A) Apoptosis B) Phagocytosis C) Antibody response D) Cellular immune response B Feedback: During the first mechanism of defense, white blood cells, which have the ability to ingest foreign particles, move to the point of attack, where they engulf and destroy the invading agents. This is known as phagocytosis. The action described is not apoptosis (programmed cell death) or an antibody response. Phagocytosis occurs in the context of the cellular immune response, but it does not constitute the entire cellular response. 19. A nurse is reviewing a patients medication administration record in an effort to identify drugs that may contribute to the patients recent immunosuppression. What drug is most likely to have this effect? A) An antibiotic B) A nonsteroidal anti-inflammatory drug (NSAID) C) An antineoplastic D) An antiretroviral C Feedback: Chemotherapy affects bone marrow function, destroying cells that contribute to an effective immune response and resulting in immunosuppression. Antibiotics in large doses cause bone marrow suppression, but antineoplastic drugs have the most pronounced immunosuppressive effect. NSAIDs and antiretrovirals do not normally have this effect. 20. A patient requires ongoing treatment and infection-control precautions because of an inherited deficit in immune function. The nurse should recognize that this patient most likely has what type of immune disorder? A) A primary immune deficiency B) A gammopathy C) An autoimmune disorder D) A rheumatic disorder C Feedback: Primary immune deficiency results from improper development of immune cells or tissues. These disorders are usually congenital or inherited. Autoimmune disorders are less likely to have a genetic component, though some have a genetic component. Overproduction of immunoglobulins is the hallmark of gammopathies. Rheumatic disorders do not normally involve impaired immune function. 21. A neonate exhibited some preliminary signs of infection, but the infants condition resolved spontaneously prior to discharge home from the hospital. This infants recovery was most likely due to what type of immunity? A) Cytokine immunity B) Specific immunity C) Active acquired immunity D) Nonspecific immunity D Feedback: Natural immunity, or nonspecific immunity, is present at birth. Active acquired or specific immunity develops after birth. Cytokines are proteins that mediate the immune response; they are not a type of immunity. 22. A gerontologic nurse is caring for an older adult patient who has a diagnosis of pneumonia. What age- related change increases older adults susceptibility to respiratory infections? A) Atrophy of the thymus B) Bronchial stenosis C) Impaired ciliary action D) Decreased diaphragmatic muscle tone C Feedback: As a consequence of impaired ciliary action due to exposure to smoke and environmental toxins, older adults are vulnerable to lung infections. This vulnerability is not the result of thymus atrophy, stenosis of the bronchi, or loss of diaphragmatic muscle tone. 23. A nurse is explaining the process by which the body removes cells from circulation after they have performed their physiologic function. The nurse is describing what process? A) The cellular immune response B) Apoptosis C) Phagocytosis D) Opsonization B Feedback: Apoptosis, or programmed cell death, is the bodys way of destroying worn out cells such as blood or skin cells or cells that need to be renewed. Opsonization is the coating of antigenantibody molecules with a sticky substance to facilitate phagocytosis. The body does not use phagocytosis or the cellular immune response to remove cells from circulation. 24. A patient is responding to a microbial invasion and the patients differentiated lymphocytes have begun to function in either a humoral or a cellular capacity. During what stage of the immune response does this occur? A) The recognition stage B) The effector stage C) The response stage D) The proliferation stage C Feedback: In the response stage, the differentiated lymphocytes function in either a humoral or a cellular capacity. In the effector stage, either the antibody of the humoral response or the cytotoxic (killer) T cell of the cellular response reaches and connects with the antigen on the surface of the foreign invader. In the recognition stage, the recognition of antigens as foreign, or non-self, by the immune system is the initiating event in any immune response. During the proliferation stage the circulating lymphocytes containing the antigenic message return to the nearest lymph node. 25. A nurse is reviewing the immune system before planning an immunocompromised patients care. How should the nurse characterize the humoral immune response? A) Specialized cells recognize and ingest cells that are recognized as foreign. B) T lymphocytes are assisted by cytokines to fight infection. C) Lymphocytesare stimulated to become cells that attack microbes directly. D) Antibodies are made by B lymphocytes in response to a specific antigen. D Feedback: The humoral response is characterized by the production of antibodies by B lymphocytes in response to a specific antigen. Phagocytosis and direct attack on microbes occur in the context of the cellular immune response. 26. A patient is undergoing testing to determine the overall function of her immune system. What test can be performed to evaluate the functioning of the patients cellular immune system? A) Immunoglobulin testing B) Delayed hypersensitivity skin test C) Specific antibody response D) Total serum globulin assessment B Feedback: Cellular (cell-mediated) immunity tests include the delayed hypersensitivity skin test, since this immune response is specifically dependent on the cellular immune response. Each of the other listed tests assesses functioning of the humoral immune system. 27. Diagnostic testing has revealed a deficiency in the function of a patients complement system. This patient is likely to have an impaired ability to do which of the following? A) Protecting the body against viral infection B) Marking the parameters of the immune response C) Bridging natural and acquired immunity D) Collecting immune complexes during inflammation C Feedback: Complement has three major physiologic functions: defending the body against bacterial infection, bridging natural and acquired immunity, and disposing of immune complexes and the byproducts associated with inflammation. Complement does not mark the parameters of the immune response; complement does not collect immune complexes during inflammation. 28. A patients current immune response involves the direct destruction of foreign microorganisms. This aspect of the immune response may be performed by what cells? A) Suppressor T cells B) Memory T cells C) Cytotoxic T cells D) Complement T cells C Feedback: Cytotoxic T cells (also called CD8 + cells) participate in the destruction of foreign organisms. Memory T cells and suppressor T cells do not perform this role in the immune response. The complement system does not exist as a type of T cell. 29. A nurse is explaining how the humoral and cellular immune responses should be seen as interacting parts of the broader immune system rather than as independent and unrelated processes. What aspect of immune function best demonstrates this? A) The movement of B cells in and out of lymph nodes B) The interactions that occur between T cells and B cells C) The differentiation between different types of T cells D) The universal role of the complement system B Feedback: T cells interact closely with B cells, indicating that humoral and cellular immune responses are not separate, unrelated processes, but rather branches of the immune response that interact. Movement of B cells does not clearly show the presence of a unified immune system. The differentiation between types of T cells and the role of the complement system do not directly suggest a single immune system. 30. A nurse is caring for a patient who has had a severe antigen/antibody reaction. The nurse knows that the portion of the antigen that is involved in binding with the antibody is called what? A) Antibody lock B) Antigenic sequence C) Antigenic determinant D) Antibody channel C Feedback: The portion of the antigen involved in binding with the antibody is referred to as the antigenic determinant. This portion is not known as an antibody lock, antigenic sequence, or antibody channel. 31. A patient is being treated for cancer and the nurse has identified the nursing diagnosis of Risk for Infection Due to Protein Losses. Protein losses inhibit immune response in which of the following ways? A) Causing apoptosis of cytokines B) Increasing interferon production C) Causing CD4+ cells to mutate D) Depressing antibody response D Feedback: Depletion of protein reserves results in atrophy of lymphoid tissues, depression of antibody response, reduction in the number of circulating T cells, and impaired phagocytic function. This specific nutritional deficit does not cause T-cell mutation, an increase in the production of interferons, or apoptosis of cytokines. 32. A patient is vigilant in her efforts to take good care of herself but is frustrated by her recent history of upper respiratory infections and influenza. What aspect of the patients lifestyle may have a negative effect on immune response? A) The patient works out at the gym twice daily. B) The patient does not eat red meats. C) The patient takes over-the-counter dietary supplements. D) The patient sleeps approximately 6 hours each night. A Feedback: Rigorous exercise or competitive exerciseusually considered a positive lifestyle factorcan be a physiologic stressor and cause negative effects on immune response. The patients habits around diet and sleep do not present obvious threats to immune function. 33. The nurse is assessing a clients risk for impaired immune function. What assessment finding should the nurse identify as a risk factor for decreased immunity? A) The patient takes a beta blocker for the treatment of hypertension. B) The patient is under significant psychosocial stress. C) The patient had a pulmonary embolism 18 months ago. D) The patient has a family history of breast cancer. B Feedback: Stress is a psychoneuroimmunologic factor that is known to depress the immune response. Use of beta blockers, a family history of cancer, and a prior PE are significant assessment findings, but none represents an immediate threat to immune function. 34. The nurse is completing a focused assessment addressing a patients immune function. What should the nurse prioritize in the physical assessment? A) Percussion of the patients abdomen B) Palpation of the patients liver C) Auscultation of the patients apical heart rate D) Palpation of the patients lymph nodes D Feedback: During the assessment of immune function, the anterior and posterior cervical, supraclavicular, axillary, and inguinal lymph nodes are palpated for enlargement. If palpable nodes are detected, their location, size, consistency, and reports of tenderness on palpation are noted. Because of the central role of lymph nodes in the immune system, they are prioritized over the heart, liver, and abdomen, even though these would be assessed. 35. A patients exposure to which of the following microorganisms is most likely to trigger a cellular response? A) Herpes simplex B) Staphylococcus aureus C) Pseudomonas aeruginosa D) Beta hemolytic Streptococcus A Feedback: Viral, rather than bacterial antigens, induce a cellular response. 36. A patient was recently exposed to infectious microorganisms and many T lymphocytes are now differentiating into killer T cells. This process characterizes what stage of the immune response? A) Effector B) Proliferation C) Response D) Recognition B Feedback: In the proliferation stage, T lymphocytes differentiate into cytotoxic (or killer) T cells, whereas B lymphocytes produce and release antibodies. This does not occur in the response, recognition, or effector stages. 37. The nurse knows that the response of natural immunity is enhanced by processes that are inherent in the physical and chemical barriers of the body. What is a chemical barrier that enhances the response of natural immunity? A) Cell cytoplasm B) Interstitial fluid C) Gastric secretions D) Cerebrospinal fluid C Feedback: Chemical barriers, such as mucus, acidic gastric secretions, enzymes in tears and saliva, and substances in sebaceous and sweat secretions, act in a nonspecific way to destroy invading bacteria and fungi. Not all body fluids are chemical barriers, however. Cell cytoplasm, interstitial fluid, and CSF are not normally categorized as chemical barriers to infection. 38. A nursing student is giving a report on the immune system. What function of cytokines should the student describe? A) Determining whether a cell is foreign B) Determining if lymphokines will be activated C) Determining whether the T cells will remain in the nodes and retain a memory of the antigen D) Determining whether the immune response will be the production of antibodies or a cell-mediated response D Feedback: Separate subpopulations of helper T cells produce different types of cytokines and determine whether the immune response will be the production of antibodies or a cell-mediated immune response. Cytokines do not determine whether cells are foreign, determine if lymphokines will be activated, or determine the role of memory T cells. 39. A patient has undergone treatment for septic shock and received high doses of numerous antibiotics during the course of treatment. When planning the patients subsequent care, the nurse should be aware of what potential effect on the patients immune function? A) Bone marrow suppression B) Uncontrolled apoptosis C) Thymus atrophy D) Lymphoma A Feedback: Large doses of antibiotics can precipitate bone marrow suppression, affecting immune function. Antibiotics are not noted to cause apoptosis, thymus atrophy, or lymphoma. 40. A patients recent diagnostic testing included a total lymphocyte count. The results of this test will allow the care team to gauge what aspect of the patients immunity? A) Humoral immune function B) Antigen recognition C) Cell-mediated immune function D) Antibody production C Feedback: A total lymphocyte count is a test used to determine cellular immune function. It is not normally used for testing humoral immune function and the associated antigenantibody. Chapter 36: Management of Patients With Immune Deficiency Disorders 1. Since the emergence of HIV/AIDS, there have been significant changes in epidemiologic trends. Members of what group currently have the greatest risk of contracting HIV? A) Gay, bisexual, and other men who have sex with men B) Recreational drug users C) Blood transfusion recipients D) Health care providers A Feedback: Gay, bisexual, and other men who have sex with men remain the population most affected by HIV and account for 2% of the population but 61% of the new infections. This exceeds the incidence among drug users, health care workers, and transfusion recipients. 2. A clinic nurse is caring for a patient admitted with AIDS. The nurse has assessed that the patient is experiencing a progressive decline in cognitive, behavioral, and motor functions. The nurse recognizes that these symptoms are most likely related to the onset of what complication? A) HIV encephalopathy B) B-cell lymphoma C) Kaposis sarcoma D) Wasting syndrome A Feedback: HIV encephalopathy is a clinical syndrome characterized by a progressive decline in cognitive, behavioral, and motor functions. The other listed complications do not normally have cognitive and behavioral manifestations. 3. A nurse is assessing a 28-year-old man with HIV who has been admitted with pneumonia. In assessing the patient, which of the following observations takes immediate priority? A) Oral temperature of 100F B) Tachypnea and restlessness C) Frequent loose stools D) Weight loss of 1 pound since yesterday B Feedback: In prioritizing care, the pneumonia would be assessed first by the nurse. Tachypnea and restlessness are symptoms of altered respiratory status and need immediate priority. Weight loss of 1 pound is probably fluid related; frequent loose stools would not take short-term precedence over a temperature or tachypnea and restlessness. An oral temperature of 100F is not considered a fever and would not be the first issue addressed. 4. A patient has come into the free clinic asking to be tested for HIV infection. The patient asks the nurse how the test works. The nurse responds that if the testing shows that antibodies to the AIDS virus are present in the blood, this indicates what? A) The patient is immune to HIV. B) The patients immune system is intact. C) The patient has AIDS-related complications. D) The patient has been infected with HIV. D Feedback: Positive test results indicate that antibodies to the AIDS virus are present in the blood. The presence of antibodies does not imply an intact immune system or specific immunity to HIV. This finding does not indicate the presence of AIDS-related complications. A hospital patient is immunocompromised because of stage 3 HIV infection and the physician has ordered a chest radiograph. How should the nurse most safely facilitate the test? A) Arrange for a portable x-ray machine to be used. B) Have the patient wear a mask to the x-ray department. C) Ensure that the radiology department has been disinfected prior to the test. D) Send the patient to the x-ray department, and have the staff in the department wear masks. A Feedback: A patient who is immunocompromised is at an increased risk of contracting nosocomial infections due to suppressed immunity. The safest way the test can be facilitated is to have a portable x-ray machine in the patients room. This confers more protection than disinfecting the radiology department or using masks. 6. The mother of two young children has been diagnosed with HIV and expresses fear of dying. How should the nurse best respond to the patient? A) Would you like me to have the chaplain come speak with you? B) Youll learn much about the promise of a cure for HIV. C) Can you tell me what concerns you most about dying? D) You need to maintain hope because you may live for several years. C Feedback: The nurse can help the patient verbalize feelings and identify resources for support. The nurse should respond with an open-ended question to help the patient to identify fears about being diagnosed with a life-threatening chronic illness. Immediate deferral to spiritual care is not a substitute for engaging with the patient. The nurse should attempt to foster hope, but not in a way that downplays the patients expressed fears. 7. The nurse is addressing condom use in the context of a health promotion workshop. When discussing the correct use of condoms, what should the nurse tell the attendees? A) Attach the condom prior to erection. B) A condom may be reused with the same partner if ejaculation has not occurred. C) Use skin lotion as a lubricant if alternatives are unavailable. D) Hold the condom by the cuff upon withdrawal. D Feedback: The condom should be unrolled over the hard penis before any kind of sex. The condom should be held by the tip to squeeze out air. Skin lotions, baby oil, petroleum jelly, or cold cream should not be used with condoms because they cause latex deterioration/condom breakage. The condom should be held during withdrawal so it does not come off the penis. Condoms should never be reused. A nurse is planning the care of a patient with AIDS who is admitted to the unit withPneumocystis pneumonia (PCP). Which nursing diagnosis has the highest priority for this patient? A) Ineffective Airway Clearance B) Impaired Oral Mucous Membranes C) Imbalanced Nutrition: Less than Body Requirements D) Activity Intolerance A Feedback: Although all these nursing diagnoses are appropriate for a patient with AIDS, Ineffective Airway Clearance is the priority nursing diagnosis for the patient with Pneumocystis pneumonia (PCP). Airway and breathing take top priority over the other listed concerns. 9. A public health nurse is preparing an educational campaign to address a recent local increase in the incidence of HIV infection. The nurse should prioritize which of the following interventions? A) Lifestyle actions that improve immune function B) Educational programs that focus on control and prevention C) Appropriate use of standard precautions D) Screening programs for youth and young adults B Feedback: Until an effective vaccine is developed, preventing HIV by eliminating and reducing risk behaviors is essential. Educational interventions are the primary means by which behaviors can be influenced. Screening is appropriate, but education is paramount. Enhancing immune function does not prevent HIV infection. Ineffective use of standard precautions apply to very few cases of HIV infection. 10. A nurse is working with a patient who was diagnosed with HIV several months earlier. The nurse should recognize that a patient with HIV is considered to have AIDS at the point when the CD4+ T-lymphocyte cell count drops below what threshold? A) 75 cells/mm3 of blood B) 200 cells/mm3 of blood C) 325 cells/mm3 of blood D) 450 cells/mm3 of blood B Feedback: When CD4+ T-cell levels drop below 200 cells/mm3 of blood, the person is said to have AIDS. During the admission assessment of an HIV-positive patient whose CD4+ count has recently fallen, the nurse carefully assesses for signs and symptoms related to opportunistic infections. What is the most common life-threatening infection? A) Salmonella infection B) Mycobacterium tuberculosis C) Clostridium difficile D) Pneumocystis pneumonia D Feedback: There are a number of opportunistic infections that can infect individuals with AIDS. The most common life-threatening infection in those living with AIDS is Pneumocystis pneumonia (PCP), caused by P. jiroveci (formerly carinii). Other opportunistic infections may involve Salmonella,Mycobacterium tuberculosis, and Clostridium difficile. 12. A patients current antiretroviral regimen includes nucleoside reverse transcriptase inhibitors (NRTIs). What dietary counseling will the nurse provide based on the patients medication regimen? A) Avoid high-fat meals while taking this medication. B) Limit fluid intake to 2 liters a day. C) Limit sodium intake to 2 grams per day. D) Take this medication without regard to meals. D Feedback: Many NRTIs exist, but all of them may be safely taken without regard to meals. Protein, fluid, and sodium restrictions play no role in relation to these drugs. 13. A nurse is performing an admission assessment on a patient with stage 3 HIV. After assessing the patients gastrointestinal system and analyzing the data, what is most likely to be the priority nursing diagnosis? A) Acute Abdominal Pain B) Diarrhea C) Bowel Incontinence D) Constipation B Feedback: Diarrhea is a problem in 50% to 60% of all AIDS patients. As such, this nursing diagnosis is more likely than abdominal pain, incontinence, or constipation, though none of these diagnoses is guaranteed not to apply. 14. A patient with a recent diagnosis of HIV infection expresses an interest in exploring alternative and complementary therapies. How should the nurse best respond? A) Complementary therapies generally have not been approved, so patients are usually discouraged from using them. B) Researchers have not looked at the benefits of alternative therapy for patients with HIV, so we suggest that you stay away from these therapies until there is solid research data available. C) Many patients with HIV use some type of alternative therapy and, as with most health treatments, there are benefits and risks. D) Youll need to meet with your doctor to choose between an alternative approach to treatment and a medical approach. C Feedback: The nurse should approach the topic of alternative or complementary therapies from an open-ended, supportive approach, emphasizing the need to communicate with care providers. Complementary therapies and medical treatment are not mutually exclusive, though some contraindications exist. Research supports the efficacy of some forms of complementary and alternative treatment. 15. A patient was tested for HIV using enzyme immunoassay (EIA) and results were positive. The nurse should expect the primary care provider to order what test to confirm the EIA test results? A) Another EIA test B) Viral load test C) Western blot test D) CD4/CD8 ratio C Feedback: The Western blot test detects antibodies to HIV and is used to confirm the EIA test results. The viral load test measures HIV RNA in the plasma and is not used to confirm EIA test results, but instead to track the progression of the disease process. The CD4/CD8 ratio test evaluates the ratio of CD4 and CD8 cells but is not used to confirm results of EIA testing. 16. The nurses plan of care for a patient with stage 3 HIV addresses the diagnosis of Risk for Impaired Skin Integrity Related to Candidiasis. What nursing intervention best addresses this risk? A) Providing thorough oral care before and after meals B) Administering prophylactic antibiotics C) Promoting nutrition and adequate fluid intake D) Applying skin emollients as needed A Feedback: Thorough mouth care has the potential to prevent or limit the severity of this infection. Antibiotics are irrelevant because of the fungal etiology. The patient requires adequate food and fluids, but these do not necessarily prevent candidiasis. Skin emollients are not appropriate because candidiasis is usually oral. 17. A patient with HIV infection has begun experiencing severe diarrhea. What is the most appropriate nursing intervention to help alleviate the diarrhea? A) Administer antidiarrheal medications on a scheduled basis, as ordered. B) Encourage the patient to eat three balanced meals and a snack at bedtime. C) Increase the patients oral fluid intake. D) Encourage the patient to increase his or her activity level. A Feedback: Administering antidiarrheal agents on a regular schedule may be more beneficial than administering them on an as-needed basis, provided the patients diarrhea is not caused by an infectious microorganism. Increased oral fluid may exacerbate diarrhea; IV fluid replacement is often indicated. Small, more frequent meals may be beneficial, and it is unrealistic to increase activity while the patient has frequent diarrhea. 18. A nurse is caring for a patient hospitalized with AIDS. A friend comes to visit the patient and privately asks the nurse about the risk of contracting HIV when visiting the patient. What is the nurses best response? A) Do you think that you might already have HIV? B) Dont worry. Your immune system is likely very healthy. C) AIDS isnt transmitted by casual contact. D) You cant contract AIDS in a hospital setting. C Feedback: AIDS is commonly transmitted by contact with blood and body fluids. Patients, family, and friends must be reassured that HIV is not spread through casual contact. A healthy immune system is not necessarily a protection against HIV. A hospital setting does not necessarily preclude HIV infection. 19. A patient with HIV has a nursing diagnosis of Risk for Impaired Skin Integrity. What nursing intervention best addresses this risk? A) Utilize a pressure-reducing mattress. B) Limit the patients physical activity. C) Apply antibiotic ointment to dependent skin surfaces. D) Avoid contact with synthetic fabrics. A Feedback: Devices such as alternating-pressure mattresses and low-air-loss beds are used to prevent skin breakdown. Activity should be promoted, not limited, and contact with synthetic fabrics does not necessary threaten skin integrity. Antibiotic ointments are not normally used unless there is a break in the skin surface. 20. A nurse would identify that a colleague needs additional instruction on standard precautions when the colleague exhibits which of the following behaviors? A) The nurse wears face protection, gloves, and a gown when irrigating a wound. B) The nurse washes the hands with a waterless antiseptic agent after removing a pair of soiled gloves. C) The nurse puts on a second pair of gloves over soiled gloves while performing a bloody procedure. D) The nurse places a used needle and syringe in the puncture-resistant container without capping the needle. C Feedback: Gloves must be changed after contact with materials that may contain high concentration of microorganisms, even when working with the same patient. Each of the other listed actions adheres to standard precautions. 21. An 18-year-old pregnant female has tested positive for HIV and asks the nurse if her baby is going to be born with HIV. What is the nurses best response? A) There is no way to know that for certain, but we do know that your baby has a one in four chance of being born with HIV. B) Your physician is likely the best one to ask that question. C) If the baby is HIV positive there is nothing that can be done until it is born, so try your best not to worry about it now. D) Its possible that your baby could contract HIV, either before, during, or after delivery. D Feedback: Mother-to-child transmission of HIV-1 is possible and may occur in utero, at the time of delivery, or through breast-feeding. There is no evidence that the infants risk is 25%. Deferral to the physician is not a substitute for responding appropriately to the patients concern. Downplaying the patients concerns is inappropriate. 22. A nurse is addressing the incidence and prevalence of HIV infection among older adults. What principle should guide the nurses choice of educational interventions? A) Many older adults do not see themselves as being at risk for HIV infection. B) Many older adults are not aware of the difference between HIV and AIDS. C) Older adults tend to have more sex partners than younger adults. D) Older adults have the highest incidence of intravenous drug use. A Feedback: It is known that many older adults do not see themselves as being at risk for HIV infection. Knowledge of the relationship between HIV infection and AIDS is not known to affect the incidence of new cases. The statements about sex partners and IV drug use are untrue. 23. A 16-year-old has come to the clinic and asks to talk to a nurse. The nurse asks the teen what she needs and the teen responds that she has become sexually active and is concerned about getting HIV. The teen asks the nurse what she can do keep from getting HIV. What would be the nurses best response? A) Theres no way to be sure you wont get HIV except to use condoms correctly. B) Only the correct use of a female condom protects against the transmission of HIV. C) There are new ways of protecting yourself from HIV that are being discovered every day. D) Other than abstinence, only the consistent and correct use of condoms is effective in preventing HIV. D Feedback: Other than abstinence, consistent and correct use of condoms is the only effective method to decrease the risk of sexual transmission of HIV infection. Both female and male condoms confer significant protection. New prevention techniques are not commonly discovered, though advances in treatment are constant. 24. A patient is in the primary infection stage of HIV. What is true of this patients current health status? A) The patients HIV antibodies are successfully, but temporarily, killing the virus. B) The patient is infected with HIV but lacks HIV-specific antibodies. C) The patients risk for opportunistic infections is at its peak. D) The patient may or may not develop long-standing HIV infection. B Feedback: The period from infection with HIV to the development of HIV-specific antibodies is known as primary infection. The virus is not being eradicated and infection is certain. Opportunistic infections emerge much later in the course of the disease. 25. A patients primary infection with HIV has subsided and an equilibrium now exists between HIV levels and the patients immune response. This physiologic state is known as which of the following? A) Static stage B) Latent stage C) Viral set point D) Window period C Feedback: The remaining amount of virus in the body after primary infection is referred to as the viral set point, which results in a steady state of infection that lasts for years. This is not known as the static or latent stage. The window period is the time a person infected with HIV tests negative even though he or she is infected. 26. A patient with HIV will be receiving care in the home setting. What aspect of self-care should the nurse emphasize during discharge education? A) Appropriate use of prophylactic antibiotics B) Importance of personal hygiene C) Signs and symptoms of wasting syndrome D) Strategies for adjusting antiretroviral dosages B Feedback: Infection control is of high importance in patients living with HIV, thus personal hygiene is paramount. This is a more important topic than signs and symptoms of one specific complication (wasting syndrome). Drug dosages should never be independently adjusted. Prophylactic antibiotics are not normally prescribed unless the patients CD4 count is below 50. 27. A patient is beginning an antiretroviral drug regimen shortly after being diagnosed with HIV. What nursing action is most likely to increase the likelihood of successful therapy? A) Promoting appropriate use of complementary therapies B) Addressing possible barriers to adherence C) Educating the patient about the pathophysiology of HIV D) Teaching the patient about the need for follow-up blood work B Feedback: ART is highly dependent on adherence to treatment, and the nurse should proactively address this. Blood work is necessary, but this will not have a direct bearing on the success or failure of treatment. Complementary therapies are appropriate, but are not the main factor in successful treatment. The patient may or may not benefit from teaching about HIV pathophysiology. 28. The nurse is caring for a patient who has been admitted for the treatment of AIDS. In the morning, the patient tells the nurse that he experienced night sweats and recently coughed up some blood. What is the nurses most appropriate action? A) Assess the patient for additional signs and symptoms of Kaposis sarcoma. B) Review the patients most recent viral load and CD4+ count. C) Place the patient on respiratory isolation and inform the physician. D) Perform oral suctioning to reduce the patients risk for aspiration. C Feedback: These signs and symptoms are suggestive of tuberculosis, not Kaposis sarcoma; prompt assessment and treatment is necessary. There is no indication of a need for oral suctioning and the patients blood work will not reflect the onset of this opportunistic infection. A patient has come into contact with HIV. As a result, HIV glycoproteins have fused with the patients CD4+ T-cell membranes. This process characterizes what phase in the HIV life cycle? A) Integration B) Attachment C) Cleavage D) Budding B Feedback: During the process of attachment, glycoproteins of HIV bind with the hosts uninfected CD4+ receptor and chemokine coreceptors, which results in fusion of HIV with the CD4+ T-cell membrane. Integration, cleavage, and budding are steps that are subsequent to this initial phase of the HIV life cycle. An HIV-infected patient presents at the clinic for a scheduled CD4+ count. The results of the test are 45 cells/mL, and the nurse recognizes the patients increased risk for Mycobacterium aviumcomplex (MAC disease). The nurse should anticipate the administration of what drug? A) Azithromycin B) Vancomycin C) Levofloxacin D) Fluconazole A Feedback: HIV-infected adults and adolescents should receive chemoprophylaxis against disseminatedMycobacterium avium complex (MAC disease) if they have a CD4+ count less than 50 cells/L. Azithromycin (Zithromax) or clarithromycin (Biaxin) are the preferred prophylactic agents. Vancomycin, levofloxacin, and fluconazole are not prophylactic agents for MAC. 31. A patient with HIV is admitted to the hospital because of chronic severe diarrhea. The nurse caring for this patient should expect the physician to order what drug for the management of the patients diarrhea? A) Zithromax B) Sandostatin C) Levaquin D) Biaxin B Feedback: Therapy with octreotide acetate (Sandostatin), a synthetic analogue of somatostatin, has been shown to be effective in managing chronic severe diarrhea. Zithromax, Levaquin, and Biaxin are not used to treat chronic severe diarrhea. 32. A patient with AIDS is admitted to the hospital with AIDS-related wasting syndrome and AIDS-related anorexia. What drug has been found to promote significant weight gain in AIDS patients by increasing body fat stores? A) Advera B) Momordicacharantia C) Megestrol D) Ranitidine C Feedback: Megestrol acetate (Megace), a synthetic oral progesterone preparation, promotes significant weight gain. In patients with HIV infection, it increases body weight primarily by increasing body fat stores. Advera is a nutritional supplement that has been developed specifically for people with HIV infection and AIDS. Momordicacharantia (bitter melon) is given as an enema and is part of alternative treatment for HIV/AIDS. Ranitidine prevents ulcers. 33. A nurse is completing a nutritional status of a patient who has been admitted with AIDS-related complications. What components should the nurse include in this assessment? Select all that apply. A) Serum albumin level B) Weight history C) White blood cell count D) Body mass index E) Blood urea nitrogen (BUN) level A, B, D, E Feedback: Nutritional status is assessed by obtaining a dietary history and identifying factors that may interfere with oral intake, such as anorexia, nausea, vomiting, oral pain, or difficulty swallowing. In addition, the patients ability to purchase and prepare food is assessed. Weight history (i.e., changes over time); anthropometric measurements; and blood urea nitrogen (BUN), serum protein, albumin, and transferrin levels provide objective measurements of nutritional status. White cell count is not a typical component of a nutritional assessment. 34. A nurse is assessing the skin integrity of a patient who has AIDS. When performing this inspection, the nurse should prioritize assessment of what skin surfaces? A) Perianal region and oral mucosa B) Sacral region and lower abdomen C) Scalp and skin over the scapulae D) Axillae and upper thorax A Feedback: The nurse should inspect all the patients skin surfaces and mucous membranes, but the oral mucosa and perianal region are particularly vulnerable to skin breakdown and fungal infection. 35. A hospital nurse has experienced percutaneous exposure to an HIV-positive patients blood as a result of a needlestick injury. The nurse has informed the supervisor and identified the patient. What action should the nurse take next? A) Flush the wound site with chlorhexidine. B) Report to the emergency department or employee health department. C) Apply a hydrocolloid dressing to the wound site. D) Follow up with the nurses primary care provider. B Feedback: After initiating the emergency reporting system, the nurse should report as quickly as possible to the employee health services, the emergency department, or other designated treatment facility. Flushing is recommended, but chlorhexidine is not used for this purpose. Applying a dressing is not recommended. Following up with the nurses own primary care provider would require an unacceptable delay. 36. The nurse care plan for a patient with AIDS includes the diagnosis of Risk for Impaired Skin Integrity. What nursing intervention should be included in the plan of care? A) Maximize the patients fluid intake. B) Provide total parenteral nutrition (TPN). C) Keep the patients bed linens free of wrinkles. D) Provide the patient with snug clothing at all times. C Feedback: Skin surfaces are protected from friction and rubbing by keeping bed linens free of wrinkles and avoiding tight or restrictive clothing. Fluid intake should be adequate, and must be monitored, but maximizing fluid intake is not a goal. TPN is a nutritional intervention of last resort. 37. A patient has been diagnosed with AIDS complicated by chronic diarrhea. What nursing intervention would be appropriate for this patient? A) Position the patient in the high Fowlers position whenever possible. B) Temporarily eliminate animal protein from the patients diet. C) Make sure the patient eats at least two servings of raw fruit each day. D) Obtain a stool culture to identify possible pathogens. D Feedback: A stool culture should be obtained to determine the possible presence of microorganisms that cause diarrhea. Patients should generally avoid raw fruit when having diarrhea. There is no need to avoid animal protein or increase the height of the patients bed. 38. A patient who has AIDS is being treated in the hospital and admits to having periods of extreme anxiety. What would be the most appropriate nursing intervention? A) Teach the patient guided imagery. B) Give the patient more control of her antiretroviral regimen. C) Increase the patients activity level. D) Collaborate with the patients physician to obtain an order for hydromorphone. A Feedback: Measures such as relaxation and guided imagery may be beneficial because they decrease anxiety, which contributes to weakness and fatigue. Increased activity may be of benefit, but for other patients this may exacerbate feelings of anxiety or loss. Granting the patient control has the potential to reduce anxiety, but the patient is not normally given unilateral control of the ART regimen. Hydromorphone is not used to treat anxiety. 39. A patient who has AIDS has been admitted for the treatment of Kaposis sarcoma. What nursing diagnosis should the nurse associate with this complication of AIDS? A) Risk for Disuse Syndrome Related to Kaposis Sarcoma B) Impaired Skin Integrity Related to Kaposis Sarcoma C) Diarrhea Related to Kaposis Sarcoma D) Impaired Swallowing Related to Kaposis Sarcoma B Feedback: Kaposis sarcoma (KS) is a disease that involves the endothelial layer of blood and lymphatic vessels. This malignancy does not directly affect swallowing or bowel motility and it does not constitute a risk for disuse syndrome. 40. A nurse is performing the admission assessment of a patient who has AIDS. What components should the nurse include in this comprehensive assessment? Select all that apply. A) Current medication regimen B) Identification of patients support system C) Immune system function D) Genetic risk factors for HIV E) History of sexual practices A, B, C, E Feedback: Nursing assessment includes numerous focuses, including identification of medication use, support system, immune function and sexual history. HIV does not have a genetic component. Chapter 37: Assessment and Management of Patients With Allergic Disorders 1. A patient with a family history of allergies has suffered an allergic response based on a genetic predisposition. This atopic response is usually mediated by what immunoglobulin? A) Immunoglobulin A B) Immunoglobulin M C) Immunoglobulin G D) Immunoglobulin E D Feedback: Atopy refers to allergic reactions characterized by the action of IgE antibodies and a genetic predisposition to allergic reactions. 2. An office worker takes a cupcake that contains peanut butter. He begins wheezing, with an inspiratory stridor and air hunger and the occupational health nurse is called to the office. The nurse should recognize that the worker is likely suffering from which type of hypersensitivity? A) Anaphylactic (type 1) B) Cytotoxic (type II) C) Immune complex (type III) D) Delayed-type (type IV) A Feedback: The most severe form of a hypersensitivity reaction is anaphylaxis. An unanticipated severe allergic reaction that is often explosive in onset, anaphylaxis is characterized by edema in many tissues, including the larynx, and is often accompanied by hypotension, bronchospasm, and cardiovascular collapse in severe cases. Type II, or cytotoxic, hypersensitivity occurs when the system mistakenly identifies a normal constituent of the body as foreign. Immune complex (type III) hypersensitivity involves immune complexes formed when antigens bind to antibodies. Type III is associated with systemic lupus erythematosus, rheumatoid arthritis, certain types of nephritis, and bacterial endocarditis. Delayed-type (type IV), also known as cellular hypersensitivity, occurs 24 to 72 hours after exposure to an allergen. 3. A patient is learning about his new diagnosis of asthma with the asthma nurse. What medication has the ability to prevent the onset of acute asthma exacerbations? A) Diphenhydramine (Benadryl) B) Montelukast (Singulair) C) Albuterol sulfate (Ventolin) D) Epinephrine B Feedback: Many manifestations of inflammation can be attributed in part to leukotrienes. Medications categorized as leukotriene antagonists or modifiers such as montelukast (Singulair) block the synthesis or action of leukotrienes and prevent signs and symptoms associated with asthma. Diphenhydramine prevents histamines effect on smooth muscle. Albuterol sulfate relaxes smooth muscle during an asthma attack. Epinephrine relaxes bronchial smooth muscle but is not used on a preventative basis. 4. A nurse is preparing a patient for allergy skin testing. Which of the following precautionary steps is most important for the nurse to follow? A) The patient must not have received an immunization within 7 days. B) The nurse should administer albuterol 30 to 45 minutes prior to the test. C) Prophylactic epinephrine should be administered before the test. D) Emergency equipment should be readily available. D Feedback: Emergency equipment must be readily available during testing to treat anaphylaxis. Immunizations do not contraindicate testing. Neither epinephrine nor albuterol is given prior to testing. 5. A patient who is scheduled for a skin test informs the nurse that he has been taking corticosteroids to help control his allergy symptoms. What nursing intervention should the nurse implement? A) The patient should take his corticosteroids regularly prior to testing. B) The patient should only be tested for grass, mold, and dust initially. C) The nurse should have an emergency cart available in case of anaphylaxis during the test. D) The patients test should be cancelled until he is off his corticosteroids. D Feedback: Corticosteroids and antihistamines, including over-the-counter allergy medications, suppress skin test reactivity and should be stopped 48 to 96 hours before testing, depending on the duration of their activity. Emergency equipment must be at hand during allergy testing, but the test would be postponed. 6. A patient has developed severe contact dermatitis with burning, itching, cracking, and peeling of the skin on her hands. What should the nurse teach the patient to do? A) Wear powdered latex gloves when in public. B) Wash her hands with antibacterial soap every few hours. C) Maintain room temperature at 75F to 80F whenever possible. D) Keep her hands well-moisturized at all times. D Feedback: Powdered latex gloves can cause contact dermatitis. Skin should be kept well-hydrated and should be washed with mild soap. Maintaining roomtemperature at 75F to 80F is not necessary. 7. A patient with severe environmental allergies is scheduled for an immunotherapy injection. What should be included in teaching the patient about this treatment? A) The patient will be given a low dose of epinephrine before the treatment. B) The patient will remain in the clinic to be monitored for 30 minutes following the injection. C) Therapeutic failure occurs if the symptoms to the allergen do not decrease after 3 months. D) The allergen will be administered by the peripheral intravenous route. B Feedback: Although severe systemic reactions are rare, the risk of systemic and potentially fatal anaphylaxis exists. Because of this risk, the patient must remain in the office or clinic for at least 30 minutes after the injection and is observed for possible systemic symptoms. Therapeutic failure is evident when a patient does not experience a decrease in symptoms within 12 to 24 months. Epinephrine is not given prior to treatment and the IV route is not used. 8. The nurse in an allergy clinic is educating a new patient about the pathology of the patients health problem. What response should the nurse describe as a possible consequence of histamine release? A) Constriction of small venules B) Contraction of bronchial smooth muscle C) Dilation of large blood vessels D) Decreased secretions from gastric and mucosal cells B Feedback: Histamines effects during the immune response include contraction of bronchial smooth muscle, resulting in wheezing and bronchospasm, dilation of small venules, constriction of large blood vessels, and an increase in secretion of gastric and mucosal cells. 9. The nurse is providing care for a patient who has experienced a type I hypersensitivity reaction. What condition is an example of such a reaction? A) Anaphylactic reaction after a bee sting B) Skin reaction resulting from adhesive tape C) Myasthenia gravis D) Rheumatoid arthritis A Feedback: Anaphylactic (type I) hypersensitivity is an immediate reaction mediated by IgE antibodies and requires previous exposure to the specific antigen. Skin reactions are more commonly type IV and myasthenia gravis is thought to be a type II reaction. Rheumatoid arthritis is not a type I hypersensitivity reaction. 10. A nurse is caring for a teenage girl who has had an anaphylactic reaction after a bee sting. The nurse is providing patient teaching prior to the patients discharge. In the event of an anaphylactic reaction, the nurse informs the patient that she should self-administer epinephrine in what site? A) Forearm B) Thigh C) Deltoid muscle D) Abdomen B Feedback: The patient is taught to position the device at the middle portion of the thigh and push the device into the thigh as far as possible. The device will autoinject a premeasured dose of epinephrine into the subcutaneous tissue. 11. A nurse has included the nursing diagnosis of Risk for Latex Allergy Response in a patients plan of care. The presence of what chronic health problem would most likely prompt this diagnosis? A) Herpes simplex B) HIV C) Spina bifida D) Hypogammaglobulinemia C Feedback: Patients with spina bifida are at a particularly high risk for developing a latex allergy. This is not true of patients with herpes simplex, HIV, or hypogammaglobulinemia. 12. A patient has a documented history of allergies presents to the clinic. She states that she is frustrated by her chronic nasal congestion, anosmia (inability to smell) and inability to concentrate. The nurse should identify which of the following nursing diagnoses? A) Deficient Knowledge of Self-Care Practices Related to Allergies B) Ineffective Individual Coping with Chronicity of Condition and Need for Environmental Modification C) Acute Confusion Related to Cognitive Effects of Allergic Rhinitis D) Disturbed Body Image Related to Sequelae of Allergic Rhinitis B Feedback: The most appropriate nursing diagnosis is Ineffective Individual Coping with Chronicity of Condition and Need for Environmental Modification. This nursing diagnosis is all encompassing of the subjective and objective data. Altered body image and acute confusion are not evidenced by the data. The patients condition is not necessary attributable to a knowledge deficit. 13. A patients decline in respiratory and renal function has been attributed to Goodpasture syndrome, which is a type II hypersensitivity reaction. What pathologic process underlies the patients health problem? A) Antigens have bound to antibodies and formed inappropriate immune complexes. B) The patients body has mistakenly identified a normal constituent of the body as foreign. C) Sensitized T cells have caused cell and tissue damage. D) Mast cells have released histamines that directly cause cell lysis. B Feedback: Type II reactions, or cytotoxic hypersensitivity, occur when the system mistakenly identifies a normal constituent of the body as foreign. An example of this type of reaction is Goodpasture syndrome. Type III, or immune complex, hypersensitivity involves immune complexes that are formed when antigens bind to antibodies. Type IV hypersensitivity is mediated by sensitized T cells that cause cell and tissue damage. Histamine does not directly cause cell lysis. 14. A child is undergoing testing for food allergies after experiencing unexplained signs and symptoms of hypersensitivity. What food items would the nurse inform the parents are common allergens? A) Citrus fruits and rice B) Root vegetables and tomatoes C) Eggs and wheat D) Hard cheeses and vegetable oils C Feedback: The most common causes of food allergies are seafood (lobster, shrimp, crab, clams, fish), legumes (peanuts, peas, beans, licorice), seeds (sesame, cottonseed, caraway, mustard, flaxseed, sunflower seeds), tree nuts, berries, egg white, buckwheat, milk, and chocolate. 15. A patient has been admitted to the emergency department with signs of anaphylaxis following a bee sting. The nurse knows that if this is a true allergic reaction the patient will present with what alteration in laboratory values? A) Increased eosinophils B) Increased neutrophils C) Increased serum albumin D) Decreased blood glucose A Feedback: Higher percentages of eosinophils are considered moderate to severe eosinophilia. Moderate eosinophilia is defined as 15% to 40% eosinophils and is found in patients with allergic disorders. Hypersensitivity does not result in hypoglycemia or increased albumin and neutrophil counts. 16. A nurse is aware of the need to assess patients risks for anaphylaxis. What health care procedure constitutes the highest risk for anaphylaxis? A) Administration of the measles-mumps-rubella (MMR) vaccine B) Rapid administration of intravenous fluids C) Computed tomography with contrast solution D) Administration of nebulized bronchodilators C Feedback: Radiocontrast agents present a significant threat of anaphylaxis in the hospital setting. Vaccinations less often cause anaphylaxis. Bronchodilators and IV fluids are not implicated in hypersensitivity reactions. 17. After the completion of testing, a childs allergies have been attributed to her familys cat. When introducing the family to the principles of avoidance therapy, the nurse should promote what action? A) Removing the cat from the familys home B) Administering OTC antihistamines to the child regularly C) Keeping the cat restricted from the childs bedroom D) Maximizing airflow in the house A Feedback: In avoidance therapy, every attempt is made to remove the allergens that act as precipitating factors. Fully removing the cat from the environment is preferable to just keeping the cat out of the childs bedroom. Avoidance therapy does not involve improving airflow or using antihistamines. 18. The nurse is providing health education to the parents of a toddler who has been diagnosed with food allergies. What should the nurse teach this family about the childs health problem? A) Food allergies are a life-long condition, but most families adjust quite well to the necessary lifestyle changes. B) Consistent use of over-the-counter antihistamines can often help a child overcome food allergies. C) Make sure that you carry a steroid inhaler with you at all times, especially when you eat in restaurants. D) Many children outgrow their food allergies in a few years if they avoid the offending foods. D Feedback: Many food allergies disappear with time, particularly in children. About one-third of proven allergies disappear in 1 to 2 years if the patient carefully avoids the offending food. Antihistamines do not cure allergies and an EpiPen is carried, not a steroid inhaler. 19. A child has been diagnosed with a severe walnut allergy after suffering an anaphylactic reaction. What is a priority for health education? A) The need to begin immunotherapy as soon as possible B) The need for the parents to carry an epinephrine pen C) The need to vigilantly maintain the childs immunization status D) The need for the child to avoid all foods that have a high potential for allergies B Feedback: All patients with food allergies, especially seafood and nuts, should have an EpiPen device prescribed. The child does not necessarily need to avoid all common food allergens. Immunotherapy is not indicated in the treatment of childhood food allergies. Immunizations are important, but do not address food allergies. 20. An adolescent patients history of skin hyperreactivity and inflammation has been attributed to atopic dermatitis. The nurse should recognize that this patient consequently faces an increased risk of what health problem? A) Bronchitis B) Systemic lupus erythematosus (SLE) C) Rheumatoid arthritis D) Asthma D Feedback: Nurses should be aware that atopic dermatitis is often the first step in a process that leads to asthma and allergic rhinitis. It is not linked as closely to bronchitis, SLE, and RA. 21. The nurse is planning the care of a patient who has a diagnosis of atopic dermatitis, which commonly affects both of her hands and forearms. What risk nursing diagnosis should the nurse include in the patients care plan? A) Risk for Disturbed Body Image Related to Skin Lesions B) Risk for Disuse Syndrome Related to Dermatitis C) Risk for Ineffective Role Performance Related to Dermatitis D) Risk for Self-Care Deficit Related to Skin Lesions A Feedback: The highly visible skin lesions associated with atopic dermatitis constitute a risk for disturbed body image. This may culminate in ineffective role performance, but this is not likely the case for the majority of patients. Dermatitis is unlikely to cause a disuse syndrome or self-care deficit. 22. A patient has been brought to the emergency department by EMS after being found unresponsive. Rapid assessment reveals anaphylaxis as a potential cause of the patients condition. The care team should attempt to assess for what potential causes of anaphylaxis? Select all that apply. A) Foods B) Medications C) Insect stings D) Autoimmunity E) Environmental pollutants A, B, C Feedback: Substances that most commonly cause anaphylaxis include foods, medications, insect stings, and latex. Pollutants do not commonly cause anaphylaxis and autoimmune processes are more closely associated with types II and III hypersensitivities. 23. A school nurse is caring for a child who appears to be having an allergic response. What should be the initial action of the school nurse? A) Assess for signs and symptoms of anaphylaxis. B) Assess for erythema and urticaria. C) Administer an OTC antihistamine. D) Administer epinephrine. A Feedback: If a patient is experiencing an allergic response, the nurses initial action is to assess the patient for signs and symptoms of anaphylaxis. Erythema and urticaria may be present, but these are not the most significant or most common signs of anaphylaxis. Assessment must precede interventions, such as administering an antihistamine. Epinephrine is indicated in the treatment of anaphylaxis, not for every allergic reaction. 24. A patient is receiving a transfusion of packed red blood cells. Shortly after initiation of the transfusion, the patient begins to exhibit signs and symptoms of a transfusion reaction. The patient is suffering from which type of hypersensitivity? A) Anaphylactic (type 1) B) Cytotoxic (type II) C) Immunecomplex (type III) D) Delayed type (type IV) B Feedback: A type II hypersensitivity reaction resulting in red blood cell destruction is associated with blood transfusions. This type of reaction does not result from types I, III, or IV reactions. 25. Which of the following individuals would be the most appropriate candidate for immunotherapy? A) A patient who had an anaphylactic reaction to an insect sting B) A child with allergies to eggs and dairy C) A patient who has had a positive tuberculin skin test D) A patient with severe allergies to grass and tree pollen D Feedback: The benefit of immunotherapy has been fairly well established in instances of allergic rhinitis and bronchial asthma that are clearly due to sensitivity to one of the common pollens, molds, or household dust. Immunotherapy is not used to treat type I hypersensitivities. A positive tuberculin skin test is not an indication for immunotherapy. 26. A nurse has asked the nurse educator if there is any way to predict the severity of a patients anaphylactic reaction. What would be the nurses best response? A) The faster the onset of symptoms, the more severe the reaction. B) The reaction will be about one-third more severe than the patients last reaction to the same antigen. C) There is no way to gauge the severity of a patients anaphylaxis, even if it has occurred repeatedly in the past. D) The reaction will generally be slightly less severe than the last reaction to the same antigen. A Feedback: The time from exposure to the antigen to onset of symptoms is a good indicator of the severity of the reaction: the faster the onset, the more severe the reaction. None of the other statements is an accurate description of the course of anaphylactic reactions. 27. A nurse knows of several patients who have achieved adequate control of their allergy symptoms using over-the-counter antihistamines. Antihistamines would be contraindicated in the care of which patient? A) A patient who has previously been treated for tuberculosis B) A pregnant woman at 30 weeks gestation C) A patient who is on estrogen-replacement therapy D) A patient with a severe allergy to eggs B Feedback: Antihistamines are contraindicated during the third trimester of pregnancy. Previous tuberculosis, hormone therapy, and food allergies do not contraindicate the use of antihistamines. 28. A patient has been living with seasonal allergies for many years, but does not take antihistamines, stating, When I was young I used to take antihistamines, but they always put me to sleep. How should the nurse best respond? A) Newer antihistamines are combined with a stimulant that offsets drowsiness. B) Most people find that they develop a tolerance to sedation after a few months. C) The newer antihistamines are different than in years past, and cause less sedation. D) Have you considered taking them at bedtime instead of in the morning? C Feedback: Unlike first-generation H1 receptor antagonists, newer antihistamines bind to peripheral rather than central nervous system H1 receptors, causing less sedation, if at all. Tolerance to sedation did not usually occur with first-generation drugs and newer antihistamines are not combined with a stimulant. 29. A child has been transported to the emergency department (ED) after a severe allergic reaction. The ED nurse is evaluating the patients respiratory status. How should the nurse evaluate the patients respiratory status? Select all that apply. A) Facilitate lung function testing. B) Assess breath sounds. C) Measure the childs oxygen saturation by oximeter. D) Monitor the childs respiratory pattern. E) Assess the childs respiratory rate. B, C, D, E Feedback: The respiratory status is evaluated by monitoring the respiratory rate and pattern and by assessing for breathing difficulties, low oxygen saturation, or abnormal lung sounds such as wheezing. Lung function testing is a lengthy procedure that is not appropriate in an emergency context. 30. A patient with multiple food and environmental allergies tells the nurse that he is frustrated and angry about having to be so watchful all the time and wonders if it is really worth it. What would be the nurses best response? A) I can only imagine how you feel. Would you like to talk about it? B) Lets find a quiet spot and Ill teach you a few coping strategies. C) Thats the same way that most patients who have a chronic illness feel. D) Do you think that maybe you could be managing things more efficiently? A Feedback: To assist the patient in adjusting to these modifications, the nurse must have an appreciation of the difficulties encountered by the patient. The patient is encouraged to verbalize feelings and concerns in a supportive environment and to identify strategies to deal with them effectively. The nurse should not suggest that the patient has been mismanaging his health problem and the nurse should not make comparisons with other patients. Further assessment should precede educational interventions. 31. A nurse at an allergy clinic is providing education for a patient starting immunotherapy for the treatment of allergies. What education should the nurse prioritize? A) The importance of scheduling appointments for the same time each month B) The importance of keeping appointments for desensitization procedures C) The importance of avoiding antihistamines for the duration of treatment D) The importance of keeping a diary of reactions to the immunotherapy B Feedback: The nurse informs and reminds the patient of the importance of keeping appointments for desensitization procedures, because dosages are usually adjusted on a weekly basis, and missed appointments may interfere with the dosage adjustment. Appointments are more frequent than monthly and antihistamines are not contraindicated. There is no need to keep a diary of reactions. 32. A patient has presented with signs and symptoms that are consistent with contact dermatitis. What aspect of care should the nurse prioritize when working with this patient? A) Promoting adequate perfusion in affected regions B) Promoting safe use of topical antihistamines C) Identifying the offending agent, if possible D) Teaching the patient to safely use an EpiPen C Feedback: Identifying the offending agent is a priority in the care of a patient with dermatitis. Antihistamines are not administered topically and epinephrine is not used to treat dermatitis. Inadequate perfusion occurs with PAD or vasoconstriction. 33. A patient was prescribed an oral antibiotic for the treatment of sinusitis. The patient has now stopped, stating she developed a rash shortly after taking the first dose of the drug. What is the nurses most appropriate response? A) Encourage the woman to continue with the medication while monitoring her skin condition closely. B) Refer the woman to her primary care provider to have the medication changed. C) Arrange for the woman to go to the nearest emergency department. D) Encourage the woman to take an OTC antihistamine with each dose of the antibiotic. B Feedback: On discovery of a medication allergy, patients are warned that they have a hypersensitivity to a particular medication and are advised not to take it again. As a result, the patient would need to liaise with the primary care provider. There is no need for emergency care unless symptoms worsen to involve respiratory function. An antihistamine would not be an adequate or appropriate recommendation from the nurse. 34. A patient has sought care, stating that she developed hives overnight. The nurses inspection confirms the presence of urticaria. What type of allergic hypersensitivity reaction has the patient developed? A) Type I B) Type II C) Type III D) Type IV A Feedback: Urticaria (hives) is a type I hypersensitive allergic reaction 35. The nurse is providing care for a patient who has a diagnosis of hereditary angioedema. When planning this patients care, what nursing diagnosis should be prioritized? A) Risk for Infection Related to Skin Sloughing B) Risk for Acute Pain Related to Loss of Skin Integrity C) Risk for Impaired Skin Integrity Related to Cutaneous Lesions D) Risk for Impaired Gas Exchange Related to Airway Obstruction D Feedback: Edema of the respiratory tract can compromise the airway in patients with hereditary angioedema. As such, this is a priority nursing diagnosis over pain and possible infection. Skin integrity is not threatened by angioedema. 36. A junior nursing student is having an observation day in the operating room. Early in the day, the student tells the OR nurse that her eyes are swelling and she is having trouble breathing. What should the nurse suspect? A) Cytotoxic reaction due to contact with the powder in the gloves B) Immune complex reaction due to contact with anesthetic gases C) Anaphylaxis due to a latex allergy D) Delayed reaction due to exposure to cleaning products C Feedback: Immediate hypersensitivity to latex, a type I allergic reaction, is mediated by the IgE mast cell system. Symptoms can include rhinitis, conjunctivitis, asthma, and anaphylaxis. The term latex allergy is usually used to describe the type I reaction. The rapid onset is not consistent with a cytotoxic reaction, an immune complex reaction, or a delayed reaction. 37. A nurse is caring for a patient who has allergic rhinitis. What intervention would be most likely to help the patient meet the goal of improved breathing pattern? A) Teach the patient to take deep breaths and cough frequently. B) Use antihistamines daily throughout the year. C) Teach the patient to seek medical attention at the first sign of an allergic reaction. D) Modify the environment to reduce the severity of allergic symptoms. D Feedback: The patient is instructed and assisted to modify the environment to reduce the severity of allergic symptoms or to prevent their occurrence. Deep breathing and coughing are not indicated unless an infection is present. Anaphylaxis requires prompt medical attention, but a minority of allergic reactions are anaphylaxis. Overuse of antihistamines reduces their effectiveness. 38. The nurse is creating a care plan for a patient suffering from allergic rhinitis. Which of the following outcomes should the nurse identify? A) Appropriate use of prophylactic antibiotics B) Safe injection of corticosteroids C) Improved skin integrity D) Improved coping with lifestyle modifications D Feedback: The goals for the patient with allergies may include restoration of normal breathing pattern, increased knowledge about the causes and control of allergic symptoms, improved coping with alterations and modifications, and absence of complications. Antibiotics are not used to treat allergies and corticosteroids, if needed, are not administered parenterally. Allergies do not normally threaten skin integrity. 39. A 5-year-old boy has been diagnosed with a severe food allergy. What is an important parameter to address when educating the parents of this child about his allergy and care? A) Wear a medical identification bracelet. B) Know how to use the antihistamine pen. C) Know how to give injections of lidocaine. D) Avoid live attenuated vaccinations. A Feedback: The nurse also advises the patient to wear a medical identification bracelet or to carry emergency equipment at all times. Patients and their families do not carry antihistamine pens, they carry epinephrine pens. Lidocaine is not self-administered to treat allergies. The patient may safely be vaccinated. 40. A patient is brought to the emergency department (ED) in a state of anaphylaxis. What is the ED nurses priority for care? A) Monitor the patients level of consciousness. B) Protect the patients airway. C) Provide psychosocial support. D) Administer medications as ordered. B Feedback: Anaphylaxis severely threatens a patients airway; the nurses priority is preserving airway patency and breathing pattern. This is a higher priority than other valid aspects of care, including medication administration, psychosocial support, and assessment of LOC. Chapter 38: Assessment and Management of Patients With Rheumatic Disorders 1. A patient is suspected of having rheumatoid arthritis and her diagnostic regimen includes aspiration of synovial fluid from the knee for a definitive diagnosis. The nurse knows that which of the following procedures will be involved? A) Angiography B) Myelography C) Paracentesis D) Arthocentesis D Feedback: Arthrocentesis involves needle aspiration of synovial fluid. Angiography is an x-ray study of circulation with a contrast agent injected into a selected artery. Myelography is an x-ray of the spinal subarachnoid space taken after the injection of a contrast agent into the spinal subarachnoid space through a lumbar puncture. Paracentesis is removal of fluid (ascites) from the peritoneal cavity through a small surgical incision or puncture made through the abdominal wall under sterile conditions. 2. A nurse is providing care for a patient who has just been diagnosed as being in the early stage of rheumatoid arthritis. The nurse should anticipate the administration of which of the following? A) Hydromorphone (Dilaudid) B) Methotrexate (Rheumatrex) C) Allopurinol (Zyloprim) D) Prednisone B Feedback: In the past, a step-wise approach starting with NSAIDs was standard of care. However, evidence clearly documenting the benefits of early DMARD (methotrexate [Rheumatrex], antimalarials, leflunomide [Arava], or sulfasalazine [Azulfidine]) treatment has changed national guidelines for management. Now it is recommended that treatment with the non-biologic DMARDs begin within 3 months of disease onset. Allopurinol is used to treat gout. Opioids are not indicated in early RA. Prednisone is used in unremitting RA. 3. A nurse is performing the initial assessment of a patient who has a recent diagnosis of systemic lupus erythematosus (SLE). What skin manifestation would the nurse expect to observe on inspection? A) Petechiae B) Butterfly rash C) Jaundice D) Skin sloughing B Feedback: An acute cutaneous lesion consisting of a butterfly-shaped rash across the bridge of the nose and cheeks occurs in SLE. Petechiae are pinpoint skin hemorrhages, which are not a clinical manifestation of SLE. Patients with SLE do not typically experience jaundice or skin sloughing. 4. A clinic nurse is caring for a patient with suspected gout. While explaining the pathophysiology of gout to the patient, the nurse should describe which of the following? A) Autoimmune processes in the joints B) Chronic metabolic acidosis C) Increased uric acid levels D) Unstable serum calcium levels C Feedback: Gout is caused by hyperuricemia (increased serum uric acid). Gout is not categorized as an autoimmune disease and it does not result from metabolic acidosis or unstable serum calcium levels. 5. A nurse is planning the care of a patient who has a long history of chronic pain, which has only recently been diagnosed as fibromyalgia. What nursing diagnosis is most likely to apply to this womans care needs? A) Ineffective Role Performance Related to Pain B) Risk for Impaired Skin Integrity Related to Myalgia C) Risk for Infection Related to Tissue Alterations D) Unilateral Neglect Related to Neuropathic Pain A Feedback: Typically, patients with fibromyalgia have endured their symptoms for a long period of time. The neuropathic pain accompanying FM can often impair a patients ability to perform normal roles and functions. Skin integrity is unaffected and the disease has no associated infection risk. Activity limitations may result in neglect, but not of a unilateral nature. 6. A patients decreased mobility is ultimately the result of an autoimmune reaction originating in the synovial tissue, which caused the formation of pannus. This patient has been diagnosed with what health problem? A) Rheumatoid arthritis (RA) B) Systemic lupus erythematosus C) Osteoporosis D) Polymyositis A Feedback: In RA, the autoimmune reaction results in phagocytosis, producing enzymes within the joint that break down collagen, cause edema and proliferation of the synovial membrane, and ultimately form pannus. Pannus destroys cartilage and bone. SLE, osteoporosis, and polymyositis do not involve pannus formation. 7. A nurse is performing the health history and physical assessment of a patient who has a diagnosis of rheumatoid arthritis (RA). What assessment finding is most consistent with the clinical presentation of RA? A) Cool joints with decreased range of motion B) Signs of systemic infection C) Joint stiffness, especially in the morning D) Visible atrophy of the knee and shoulder joints C Feedback: In addition to joint pain and swelling, another classic sign of RA is joint stiffness, especially in the morning. Joints are typically swollen, not atrophied, and systemic infection does not accompany the disease. Joints are often warm rather than cool. 8. A patient has a diagnosis of rheumatoid arthritis and the primary care provider has now prescribed cyclophosphamide (Cytoxan). The nurses subsequent assessments should address what potential adverse effect? A) Infection B) Acute confusion C) Sedation D) Malignant hyperthermia A Feedback: When administering immunosuppressives such as Cytoxan, the nurse should be alert to manifestations of bone marrow suppression and infection. Confusion and sedation are atypical adverse effects. Malignant hyperthermia is a surgical complication and not a possible adverse effect. 9. A clinic nurse is caring for a patient newly diagnosed with fibromyalgia. When developing a care plan for this patient, what would be a priority nursing diagnosis for this patient? A) Impaired Urinary Elimination Related to Neuropathy B) Altered Nutrition Related to Impaired Absorption C) Disturbed Sleep Pattern Related to CNS Stimulation D) Fatigue Related to Pain D Feedback: Fibromyalgia is characterized by fatigue, generalized muscle aching, and stiffness. Impaired urinary elimination is not a common manifestation of the disease. Altered nutrition and disturbed sleep pattern are potential nursing diagnoses, but are not the priority. 10. A nurse is assessing a patient for risk factors known to contribute to osteoarthritis. What assessment finding would the nurse interpret as a risk factor? A) The patient has a 30 pack-year smoking history. B) The patients body mass index is 34 (obese). C) The patient has primary hypertension. D) The patient is 58 years old. B Feedback: Risk factors for osteoarthritis include obesity and previous joint damage. Risk factors of OA do not include smoking or hypertension. Incidence increases with age, but a patient who is 58 would not yet face a significantly heightened risk. 11. A patient is undergoing diagnostic testing to determine the etiology of recent joint pain. The patient asks the nurse about the difference between osteoarthritis (OA) and rheumatoid arthritis (RA). What is the best response by the nurse? A) OA is a considered a noninflammatory joint disease. RA is characterized by inflamed, swollen joints. B) OA and RA are very similar. OA affects the smaller joints such as the fingers, and RA affects the larger, weight-bearing joints like the knees. C) OA originates with an infection. RA is a result of your bodys cells attacking one another. D) OA is associated with impaired immune function; RA is a consequence of physical damage. A Feedback: OA is a degenerative arthritis with a noninflammatory etiology, characterized by the loss of cartilage on the articular surfaces of weight-bearing joints, with spur development. RA is characterized by inflammation of synovial membranes and surrounding structures. The diseases are not distinguished by the joints affected and neither has an infectious etiology. 12. A patient with systemic lupus erythematosus (SLE) is preparing for discharge. The nurse knows that the patient has understood health education when the patient makes what statement? A) Ill make sure I get enough exposure to sunlight to keep up my vitamin D levels. B) Ill try to be as physically active as possible between flare-ups. C) Ill make sure to monitor my body temperature on a regular basis. D) Ill stop taking my steroids when I get relief from my symptoms. C Feedback: Fever can signal an exacerbation and should be reported to the physician. Sunlight and other sources of ultraviolet light may precipitate severe skin reactions and exacerbate the disease. Fatigue can cause a flare-up of SLE. Patients should be encouraged to pace activities and plan rest periods. Corticosteroids must be gradually tapered because they can suppress the function of the adrenal gland. As well, these drugs should not be independently adjusted by the patient. A patient with an exacerbation of systemic lupus erythematosus (SLE) has been hospitalized on the medical unit. The nurse observes that the patient expresses angerand irritation when her call bell isnt answered immediately. What would be the most appropriate response? A) You seem like youre feeling angry. Is that something that we could talk about? B) Try to remember that stress can make your symptoms worse. C) Would you like to talk about the problem with the nursing supervisor? D) I can see youre angry. Ill come back when youve calmed down. A Feedback: The changes and the unpredictable course of SLE necessitate expert assessment skills and nursing care, as well as sensitivity to the psychological reactions of the patient. Offering to listen to the patient express anger can help the nurse and the patient understand its cause and begin to deal with it. Although stress can exacerbate the symptoms of SLE, telling the patient to calm down doesnt acknowledge her feelings. Ignoring the patients feelings suggests that the nurse has no interest in what the patient has said. Offering to get the nursing supervisor also does not acknowledge the patients feelings. 14. A nurse is caring for a 78-year-old patient with a history of osteoarthritis (OA). When planning the patients care, what goal should the nurse include? A) The patient will express satisfaction with her ability to perform ADLs. B) The patient will recover from OA within 6 months. C) The patient will adhere to the prescribed plan of care. D) The patient will deny signs or symptoms of OA. A Feedback: Pain management and optimal functional ability are major goals of nursing interventions for OA. Cure is not a possibility and it is unrealistic to expect a complete absence of signs and symptoms. Adherence to the plan of care is highly beneficial, but this is not the priority goal of care. 15. A patient who has been newly diagnosed with systemic lupus erythematosus (SLE) has been admitted to the medical unit. Which of the following nursing diagnoses is the most plausible inclusion in the plan of care? A) Fatigue Related to Anemia B) Risk for Ineffective Tissue Perfusion Related to Venous Thromboembolism C) Acute Confusion Related to Increased Serum Ammonia Levels D) Risk for Ineffective Tissue Perfusion Related to Increased Hematocrit A Feedback: Patients with SLE nearly always experience fatigue, which is partly attributable to anemia. Ammonia levels are not affected and hematocrit is typically low, not high. VTE is not one of the central complications of SLE. 16. The nurse is preparing to care for a patient who has scleroderma. The nurse refers to resources that describe CREST syndrome. Which of the following is a component of CREST syndrome? A) Raynauds phenomenon B) Thyroid dysfunction C) Esophageal varices D) Osteopenia A Feedback: The R in CREST stands for Raynauds phenomenon. Thyroid dysfunction, esophageal varices, and osteopenia are not associated with scleroderma. 17. Allopurinol (Zyloprim) has been ordered for a patient receiving treatment for gout. The nurse caring for this patient knows to assess the patient for bone marrow suppression, which may be manifested by which of the following diagnostic findings? A) Hyperuricemia B) Increased erythrocyte sedimentation rate C) Elevated serum creatinine D) Decreased platelets D Feedback: Thrombocytopenia occurs in bone marrow suppression. Hyperuricemia occurs in gout, but is not caused by bone marrow suppression. Increased erythrocyte sedimentation rate may occur from inflammation associated with gout, but is not related to bone marrow suppression. An elevated serum creatinine level may indicate renal damage, but this is not associated with the use of allopurinol. 18. A patient with rheumatic disease is complaining of stomatitis. The nurse caring for the patient should further assess the patient for the adverse effects of what medications? A) Corticosteroids B) Gold-containing compounds C) Antimalarials D) Salicylate therapy B Feedback: Stomatitis is an adverse effect that is associated with gold therapy. Steroids, antimalarials, and salicylates do not normally have this adverse effect. 19. A nurse is planning patient education for a patient being discharged home with a diagnosis of rheumatoid arthritis. The patient has been prescribed antimalarials for treatment, so the nurse knows to teach the patient to self-monitor for what adverse effect? A) Tinnitus B) Visual changes C) Stomatitis D) Hirsutism B Feedback: Antimalarials may cause visual changes; regular ophthalmologic examinations are necessary.Tinnitus is associated with salicylate therapy, stomatitis is associated with gold therapy, and hirsutism is associated with corticosteroid therapy. 20. A nurse is working with a patient with rheumatic disease who is being treated with salicylate therapy. What statement would indicate that the patient is experiencing adverse effects of this drug? A) I have this ringing in my ears that just wont go away. B) I feel so foggy in the mornings and it takes me so long to wake up. C) When I eat a meal thats high in fat, I get really nauseous. D) I seem to have lost my appetite, which is unusual for me. A Feedback: Tinnitus is associated with salicylate therapy. Salicylates do not normally cause drowsiness, intolerance of high-fat meals, or anorexia. 21. A patient has been admitted to a medical unit with a diagnosis of polymyalgia rheumatica (PMR). The nurse should be aware of what aspects of PMR? Select all that apply. A) PMR has an association with the genetic marker HLA-DR4. B) Immunoglobulin deposits occur in PMR. C) PMR is considered to be a wear-and-tear disease. D) Foods high in purines exacerbate the biochemical processes that occur in PMR. E) PMR occurs predominately in Caucasians. A, B, E Feedback: The underlying mechanism involved with polymyalgia rheumatica is unknown. This disease occurs predominately in Caucasians and often in first-degree relatives. An association with the genetic marker HLA-DR4 suggests a familial predisposition. Immunoglobulin deposits in the walls of inflamed temporal arteries also suggest an autoimmune process. Purines are unrelated and it is not a result of physical degeneration. 22. A nurse is providing care for a patient who has a recent diagnosis of giant cell arteritis (GCA). What aspect of physical assessment should the nurse prioritize? A) Assessment for subtle signs of bleeding disorders B) Assessment of the metatarsal joints and phalangeal joints C) Assessment for thoracic pain that is exacerbated by activity D) Assessment for headaches and jaw pain D Feedback: Assessment of the patient with GCA focuses on musculoskeletal tenderness, weakness, and decreased function. Careful attention should be directed toward assessing the head (for changes in vision, headaches, and jaw claudication). There is not a particular clinical focus on the potential for bleeding, hand and foot pain, or thoracic pain. 23. A nurse is educating a patient with gout about lifestyle modifications that can help control the signs and symptoms of the disease. What recommendation should the nurse make? A) Ensuring adequate rest B) Limiting exposure to sunlight C) Limiting intake of alcohol D) Smoking cessation C Feedback: Alcohol and red meat can precipitate an acute exacerbation of gout. Each of the other listed actions is consistent with good health, but none directly addresses the factors that exacerbate gout. 24. A patients rheumatoid arthritis (RA) has failed to respond appreciably to first-line treatments and the primary care provider has added prednisone to the patients drug regimen. What principle will guide this aspect of the patients treatment? A) The patient will need daily blood testing for the duration of treatment. B) The patient must stop all other drugs 72 hours before starting prednisone. C) The drug should be used at the highest dose the patient can tolerate. D) The drug should be used for as short a time as possible. D Feedback: Corticosteroids are used for shortest duration and at lowest dose possible to minimize adverse effects. Daily blood work is not necessary and the patient does not need to stop other drugs prior to using corticosteroids. 25. A nurse is caring for a patient who is suspected of having giant cell arteritis (GCA). What laboratory tests are most useful in diagnosing this rheumatic disorder? Select all that apply. A) Erythrocyte count B) Erythrocyte sedimentation rate C) Creatinine clearance D) C-reactive protein E) D-dimer B, D Feedback: Simultaneous elevation in the ESR and CRP have a sensitivity of 88% and a specificity of 98% in making the diagnosis of GCA when coupled with clinical findings. Erythrocyte counts, creatinine clearance, and D-dimer are not diagnostically useful. 26. A patient with SLE has come to the clinic for a routine check-up. When auscultating the patients apical heart rate, the nurse notes the presence of a distinct scratching sound. What is the nurses most appropriate action? A) Reposition the patient and auscultate posteriorly. B) Document the presence of S3 and monitor the patient closely. C) Inform the primary care provider that a friction rub may be present. D) Inform the primary care provider that the patient may have pneumonia. C Feedback: Patients with SLE are susceptible to developing a pericardial friction rub, possibly associated with myocarditis and accompanying pleural effusions; this warrants prompt medical follow-up. This finding is not characteristic of pneumonia and does not constitute S3. Posterior auscultation is unlikely to yield additional meaningful data. 27. A community health nurse is performing a visit to the home of a patient who has a history of rheumatoid arthritis (RA). On what aspect of the patients health should the nurse focus most closely during the visit? A) The patients understanding of rheumatoid arthritis B) The patients risk for cardiopulmonary complications C) The patients social support system D) The patients functional status D Feedback: The patients functional status is a central focus of home assessment of the patient with RA. The nurse may also address the patients understanding of the disease, complications, and social support, but the patients level of function and quality of life is a primary concern. 28. A 21-year-old male has just been diagnosed with a spondyloarthropathy. What will be a priority nursing intervention for this patient? A) Referral for assistive devices B) Teaching about symptom management C) Referral to classes to stop smoking D) Setting up an exercise program B Feedback: Major nursing interventions in the spondyloarthropathies are related to symptom management and maintenance of optimal functioning. This is a priority over the use of assistive devices, smoking cessation, and exercise programs, though these topics may be of importance for some patients. 29. A patient with SLE asks the nurse why she has to come to the office so often for check-ups. What would be the nurses best response? A) Taking care of you in the best way involves seeing you face to face. B) Taking care of you in the best way involves making sure you are taking your medication the way it is ordered. C) Taking care of you in the best way involves monitoring your disease activity and how well the prescribed treatment is working. D) Taking care of you in the best way involves drawing blood work every month. C Feedback: The goals of treatment include preventing progressive loss of organ function, reducing the likelihood of acute disease, minimizing disease-related disabilities, and preventing complications from therapy. Management of SLE involves regular monitoring to assess disease activity and therapeutic effectiveness. Stating the benefit of face-to-face interaction does not answer the patients question. Blood work is not necessarily drawn monthly and assessing medication adherence is not the sole purpose of visits. A patient is diagnosed with giant cell arteritis (GCA) and is placed on corticosteroids. A concern for this patient is that he will stop taking the medication as soon as he starts to feel better. Why must the nurse emphasize the need for continued adherence to the prescribed medication? A) To avoid complications such as venous thromboembolism B) To avoid the progression to osteoporosis C) To avoid the progression of GCA to degenerative joint disease D) To avoid complications such as blindness D Feedback: The nurse must emphasize to the patient the need for continued adherence to the prescribed medication regimen to avoid complications of giant cell arteritis, such as blindness. VTE, OP, and degenerative joint disease are not among the most common complications for GCA. 31. A patient with polymyositisis experiencing challenges with activities of daily living as a result of proximal muscle weakness. What is the most appropriate nursing action? A) Initiate a program of passive range of motion exercises B) Facilitate referrals to occupational and physical therapy C) Administer skeletal muscle relaxants as ordered D) Encourage a progressive program of weight-bearing exercise B Feedback: Patients with polymyositis may have symptoms similar to those of other inflammatory diseases. However, proximal muscle weakness is characteristic, making activities such as hair combing, reaching overhead, and using stairs difficult. Therefore, use of assistive devices may be recommended, and referral to occupational or physical therapy may be warranted. The muscle weakness is a product of the disease process, not lack of exercise. Skeletal muscle relaxants are not used in the treatment of polymyositis. 32. A nurse is creating a teaching plan for a patient who has a recent diagnosis of scleroderma. What topics should the nurse address during health education? Select all that apply. A) Surgical treatment options B) The importance of weight loss C) Managing Raynauds-type symptoms D) Smoking cessation E) The importance of vigilant skin care C, D, E Feedback: Patient teaching for the patient with scleroderma focuses on management of Raynauds phenomenon, smoking cessation, and meticulous skin care. Surgical treatment options do not exist and weight loss is not a central concern. 33. A 40-year-old woman was diagnosed with Raynauds phenomenon several years earlier and has sought care because of a progressive worsening of her symptoms. The patient also states that many of her skin surfaces are stiff, like the skin is being stretched from all directions. The nurse should recognize the need for medical referral for the assessment of what health problem? A) Giant cell arteritis (GCA) B) Fibromyalgia (FM) C) Rheumatoid arthritis (RA) D) Scleroderma D Feedback: Scleroderma starts insidiously with Raynauds phenomenon and swelling in the hands. Later, the skin and the subcutaneous tissues become increasingly hard and rigid and cannot be pinched up from the underlying structures. This progression of symptoms is inconsistent with GCA, FM, or RA. 34. A patient with rheumatoid arthritis comes to the clinic complaining of pain in the joint of his right great toe and is eventually diagnosed with gout. When planning teaching for this patient, what management technique should the nurse emphasize? A) Take OTC calcium supplements consistently. B) Restrict consumption of foods high in purines. C) Ensure fluid intake of at least 4 liters per day. D) Restrict weight-bearing on right foot. B Feedback: Although severe dietary restriction is not necessary, the nurse should encourage the patient to restrict consumption of foods high in purines, especially organ meats. Calcium supplementation is not necessary and activity should be maintained as tolerated. Increased fluid intake is beneficial, but it is not necessary for the patient to consume more than 4 liters daily. A clinic nurse is caring for a patient diagnosed with rheumatoid arthritis (RA). The patient tells the nurse that she has not been taking her medication because she usually cannot remove the childproof medication lids. How can the nurse best facilitate the patients adherence to her medication regimen? A) Encourage the patient to store the bottles with their tops removed. B) Have a trusted family member take over the management of the patients medication regimen. C) Encourage her to have her pharmacy replace the tops with alternatives that are easier to open. D) Have the patient approach her primary care provider to explore medication alternatives. C Feedback: The patients pharmacy will likely be able to facilitate a practical solution that preserves the patients independence while still fostering adherence to treatment. There should be no need to change medications, and storing open medication containers is unsafe. Delegating medications to a family member is likely unnecessary at this point and promotes dependence. 36. A nurses plan of care for a patient with rheumatoid arthritis includes several exercise-based interventions. Exercises for patients with rheumatoid disorders should have which of the following goals? A) Maximize range of motion while minimizing exertion B) Increase joint size and strength C) Limit energy output in order to preserve strength for healing D) Preserve and increase range of motion while limiting joint stress D Feedback: Exercise is vital to the management of rheumatic disorders. Goals should be preserving and promoting mobility and joint function while limiting stress on the joint and possible damage. Cardiovascular exertion should remain within age-based limits and individual ability, but it is not a goal to minimize exertion. Increasing joint size is not a valid goal. 37. A patient has just been told by his physician that he has scleroderma. The physician tells the patient that he is going to order some tests to assess for systemic involvement. The nurse knows that priority systems to be assessed include what? A) Hepatic B) Gastrointestinal C) Genitourinary D) Neurologic B Feedback: Assessment of systemic involvement with scleroderma requires a systems review with special attention to gastrointestinal, pulmonary, renal, and cardiac systems. Liver, GU, and neurologic functions are not central priorities. 38. A nurse is providing care for a patient who has a rheumatic disorder. The nurses comprehensive assessment includes the patients mood, behavior, LOC, and neurologic status. What is this patients most likely diagnosis? A) Osteoarthritis (OA) B) Systemic lupus erythematosus (SLE) C) Rheumatoid arthritis (RA) D) Gout B Feedback: SLE has a high degree of neurologic involvement, and can result in central nervous system changes. The patient and family members are asked about any behavioral changes, including manifestations of neurosis or psychosis. Signs of depression are noted, as are reports of seizures, chorea, or other central nervous system manifestations. OA, RA, and gout lack this dimension. 39. A patient with rheumatoid arthritis comes into the clinic for a routine check-up. On assessment the nurse notes that the patient appears to have lost some of her ability to function since her last office visit. Which of the following is the most appropriate action? A) Arrange a family meeting in order to explore assisted living options. B) Refer the patient to a support group. C) Arrange for the patient to be assessed in her home environment. D) Refer the patient to social work. C Feedback: Assessment in the patients home setting can often reveal more meaningful data than an assessment in the health care setting. There is no indication that assisted living is a pressing need or that the patient would benefit from social work or a support group. 40. A nurse is assessing a patient with rheumatoid arthritis. The patient expresses his intent to pursue complementary and alternative therapies. What fact should underlie the nurses response to the patient? A) New evidence shows CAM to be as effective as medical treatment. B) CAM therapies negate many of the benefits of medications. C) CAM therapies typically do more harm than good. D) Evidence shows minimal benefits from most CAM therapies. D Feedback: A recent systematic review of complementary and alternative medicine (CAM) examined the efficacy of herbal medicine, acupuncture, Tai chi and biofeedback for the treatment of rheumatoid arthritis and osteoarthritis. Although acupuncture treatment for pain management showed some promise, in all modalities the evidence was ambiguous. There is not enough evidence of the effectiveness of CAM and more rigorous research is needed. Chapter 39: Assessment of Musculoskeletal Function 1. A nurse on the orthopedic unit is assessing a patients peroneal nerve. The nurse will perform this assessment by doing which of the following actions? A) Pricking the skin between the great and second toe B) Stroking the skin on the sole of the patients foot C) Pinching the skin between the thumb and index finger D) Stroking the distal fat pad of the small finger A Feedback: The nurse will evaluate the sensation of the peroneal nerve by pricking the skin centered between the great and second toe. None of the other listed actions elicits the function of one of the peripheral nerves. 2. A public health nurse is organizing a campaign that will address the leading cause of musculoskeletal- related disability in the United States. The nurse should focus on what health problem? A) Osteoporosis B) Arthritis C) Hip fractures D) Lower back pain B Feedback: The leading cause of musculoskeletal-related disability in the United States is arthritis. 3. A nurse is providing care for a patient whose pattern of laboratory testing reveals longstanding hypocalcemia. What other laboratory result is most consistent with this finding? A) An elevated parathyroid hormone level B) An increased calcitonin level C) An elevated potassium level D) A decreased vitamin D level A Feedback: In the response to low calcium levels in the blood, increased levels of parathyroid hormone prompt the mobilization of calcium and the demineralization of bone. Increased calcitonin levels would exacerbate hypocalcemia. Vitamin D levels do not increase in response to low calcium levels. Potassium levels would likely be unaffected. 4. A nurse is caring for a patient whose cancer metastasis has resulted in bone pain. Which of the following are typical characteristics of bone pain? A) A dull, deep ache that is boring in nature B) Soreness or aching that may include cramping C) Sharp, piercing pain that is relieved by immobilization D) Spastic or sharp pain that radiates A Feedback: Bone pain is characteristically described as a dull, deep ache that is boring in nature, whereas muscular pain is described as soreness or aching and is referred to as muscle cramps. Fracture pain is sharp and piercing and is relieved by immobilization. Sharp pain may also result from bone infection with muscle spasm or pressure on a sensory nerve. 5. A nurse is assessing a patient who is experiencing peripheral neurovascular dysfunction. What assessment findings are most consistent with this diagnosis? A) Hot skin with a capillary refill of 1 to 2 seconds B) Absence of feeling, capillary refill of 4 to 5 seconds, and cool skin C) Pain, diaphoresis, and erythema D) Jaundiced skin, weakness, and capillary refill of 3 seconds B Feedback: Indicators of peripheral neurovascular dysfunction include pale, cyanotic, or mottled skin with a cool temperature; capillary refill greater than 3 seconds; weakness or paralysis with motion; and paresthesia, unrelenting pain, pain on passive stretch, or absence of feeling. Jaundice, diaphoresis, and warmth are inconsistent with peripheral neurovascular dysfunction. 6. An older adult patient has symptoms of osteoporosis and is being assessed during her annual physical examination. The assessment shows that the patient will require further testing related to a possible exacerbation of her osteoporosis. The nurse should anticipate what diagnostic test? A) Bone densitometry B) Hip bone radiography C) Computed tomography (CT) D) Magnetic resonance imaging (MRI) A Feedback: Bone densitometry is considered the most accurate test for osteoporosis and for predicting a fracture. As such, it is more likely to be used than CT, MRI, or x-rays. 7. A patient injured in a motor vehicle accident has sustained a fracture to the diaphysis of the right femur. Of what is the diaphysis of the femur mainly constructed? A) Epiphyses B) Cartilage C) Cortical bone D) Cancellous bone C Feedback: The long bone shaft, which is referred to as the diaphysis, is constructed primarily of cortical bone. 8. An older adult patient has come to the clinic for a regular check-up. The nurses initial inspection reveals an increased thoracic curvature of the patients spine. The nurse should document the presence of which of the following? A) Scoliosis B) Epiphyses C) Lordosis D) Kyphosis D Feedback: Kyphosis is the increase in thoracic curvature of the spine. Scoliosis is a deviation in the lateral curvature of the spine. Epiphyses are the ends of the long bones. Lordosis is the exaggerated curvature of the lumbar spine. 9. When assessing a patients peripheral nerve function, the nurse uses an instrument to prick the fat pad at the top of the patients small finger. This action will assess which of the following nerves? A) Radial B) Ulnar C) Median D) Tibial B Feedback: The ulnar nerve is assessed for sensation by pricking the fat pad at the top of the small finger. The radial, median, and tibial nerves are not assessed in this manner. The results of a nurses musculoskeletal examination show an increase in the lumbar curvature of the spine. The nurse should recognize the presence of what health problem? A) Osteoporosis B) Kyphosis C) Lordosis D) Scoliosis C Feedback: The nurse documents the spinal abnormality as lordosis. Lordosis is an increase in lumbar curvature of the spine. Kyphosis is an increase in the convex curvature of the spine. Scoliosis is a lateral curvature of the spine. Osteoporosis is the significant loss of bone mass and strength with an increased risk for fracture. 11. The human body is designed to protect its vital parts. A fracture of what type of bone may interfere with the protection of vital organs? A) Long bones B) Short bones C) Flat bones D) Irregular bones C Feedback: Flat bones, such as the sternum, provide vital organ protection. Fractures of the flat bones may lead to puncturing of the vital organs or may interfere with the protection of the vital organs. Long, short, and irregular bones do not usually have this physiologic function. 12. A patient has just had an arthroscopy performed to assess a knee injury. What nursing intervention should the nurse implement following this procedure? A) Wrap the joint in a compression dressing. B) Perform passive range of motion exercises. C) Maintain the knee in flexion for up to 30 minutes. D) Apply heat to the knee. A Feedback: Interventions to perform following an arthroscopy include wrapping the joint in a compression dressing, extending and elevating the joint, and applying ice or cold packs. Passive ROM exercises, static flexion, and heat are not indicated. 13. While assessing a patient, the patient tells the nurse that she is experiencing rhythmic muscle contractions when the nurse performs passive extension of her wrist. What is this pattern of muscle contraction referred to as? A) Fasciculations B) Contractures C) Effusion D) Clonus D Feedback: Clonus may occur when the ankle is dorsiflexed or the wrist is extended. It is characterized as rhythmic contractions of the muscle. Fasciculation is involuntary twitching of muscle fiber groups. Contractures are prolonged tightening of muscle groups and an effusion is the pathologic escape of body fluid. A nurse is caring for an older adult who has been diagnosed with geriatric failure to thrive. This patients prolonged immobility creates a risk for what complication? A) Muscle clonus B) Muscle atrophy C) Rheumatoid arthritis D) Muscle fasciculations B Feedback: If a muscle is in disuse for an extended period of time, it is at risk of developing atrophy, which is the decrease in size. Clonus is a pattern of rhythmic muscle contractions and fasciculation is the involuntary twitch of muscle fibers; neither results from immobility. Lack of exercise is a risk factor for rheumatoid arthritis. 15. A nurse is caring for a patient who has been scheduled for a bone scan. What should the nurse teach the patient about this diagnostic test? A) The test is brief and requires that you drink a calcium solution 2 hours before the test. B) You will not be allowed fluid for 2 hours before and 3 hours after the test. C) Youll be encouraged to drink water after the administration of the radioisotope injection. D) This is a common test that can be safely performed on anyone. C Feedback: It is important to encourage the patient to drink plenty of fluids to help distribute and eliminate the isotopic after it is injected. There are important contraindications to the procedure, include pregnancy or an allergy to the radioisotope. The test requires the injection of an intravenous radioisotope and the scan is preformed 2 to 3 hours after the isotope is injected. A calcium solution is not utilized. 16. A nurse is assessing a child who has a diagnosis of muscular dystrophy. Assessment reveals that the childs muscles have greater-than-normal tone. The nurse should document the presence of which of the following? A) Tonus B) Flaccidity C) Atony D) Spasticity D Feedback: A muscle with greater-than-normal tone is described as spastic. Soft and flabby muscle tone is defined as atony. A muscle that is limp and without tone is described as being flaccid. The state of readiness known as muscle tone (tonus) is produced by the maintenance of some of the muscle fibers in a contracted state. 17. The nurses comprehensive assessment of an older adult involves the assessment of the patients gait. How should the nurse best perform this assessment? A) Instruct the patient to walk heel-to-toe for 15 to 20 steps. B) Instruct the patient to walk in a straight line while not looking at the floor. C) Instruct the patient to walk away from the nurse for a short distance and then toward the nurse. D) Instruct the patient to balance on one foot for as long as possible and then walk in a circle around the room. C Feedback: Gait is assessed by having the patient walk away from the examiner for a short distance. The examiner observes the patients gait for smoothness and rhythm. Looking at the floor is not disallowed and gait is not assessed by observing balance on one leg. Heel-to-toe walking ability is not gauged during an assessment of normal gait. 18. A clinic nurse is caring for a patient with a history of osteoporosis. Which of the following diagnostic tests best allows the care team to assess the patients risk of fracture? A) Arthrography B) Bone scan C) Bone densitometry D) Arthroscopy C Feedback: Bone densitometry is used to detect bone density and can be used to assess the risk of fracture in osteoporosis. Arthrography is used to detect acute or chronic tears of joint capsule or supporting ligaments. Bone scans can be used to detect metastatic and primary bone tumors, osteomyelitis, certain fractures, and aseptic necrosis. Arthroscopy is used to visualize a joint. 19. A nurse is performing a musculoskeletal assessment of a patient with arthritis. During passive range-of- motion exercises, the nurse hears an audible grating sound. The nurse should document the presence of which of the following? A) Fasciculations B) Clonus C) Effusion D) Crepitus D Feedback: Crepitus is a grating, crackling sound or sensation that occurs as the irregular joint surfaces move across one another, as in arthritic conditions. Fasciculations are involuntary twitching of muscle fiber groups. Clonus is the rhythmic contractions of a muscle. Effusion is the collection of excessive fluid within the capsule of a joint. 20. A patients fracture is healing and callus is being deposited in the bone matrix. This process characterizes what phase of the bone healing process? A) The reparative phase B) The reactive phase C) The remodeling phase D) The revascularization phase A Feedback: Callus formation takes place during the reparative phase of bone healing. The reactive phase occurs immediately after injury and the remodeling phase builds on the reparative phase. There is no discrete revascularization phase. 21. A child is growing at a rate appropriate for his age. What cells are responsible for the secretion of bone matrix that eventually results in bone growth? A) Osteoblasts B) Osteocytes C) Osteoclasts D) Lamellae A Feedback: Osteoblasts function in bone formation by secreting bone matrix. Osteocytes are mature bone cells and osteoclasts are multinuclear cells involved in dissolving and resorbing bone. Lamellae are circles of mineralized bone matrix. 22. A nurse is caring for a patient who has an MRI scheduled. What is the priority safety action prior to this diagnostic procedure? A) Assessing the patient for signs and symptoms of active infection B) Ensuring that the patient can remain immobile for up to 3 hours C) Assessing the patient for a history of nut allergies D) Ensuring that there are no metal objects on or in the patient D Feedback: Absolutely no metal objects can be present during MRItheir presence constitutes a serious safety risk. The procedure takes up to 90 minutes. Nut allergies and infection are not contraindications to MRI. 23. A nurse is taking a health history on a patient with musculoskeletal dysfunction. What is the primary focus of this phase of the nurses assessment? A) Evaluating the effects of the musculoskeletal disorder on the patients function B) Evaluating the patients adherence to the existing treatment regimen C) Evaluating the presence of genetic risk factors for further musculoskeletal disorders D) Evaluating the patients active and passive range of motion A Feedback: The nursing assessment of the patient with musculoskeletal dysfunction includes an evaluation of the effects of the musculoskeletal disorder on the patient. This is a vital focus of the health history and supersedes the assessment of genetic risk factors and adherence to treatment, though these are both valid inclusions to the interview. Assessment of ROM occurs during the physical assessment, not the interview. 24. A patient is scheduled for a bone scan to rule out osteosarcoma of the pelvic bones. What would be most important for the nurse to assess before the patients scan? A) That the patient completed the bowel cleansing regimen B) That the patient emptied the bladder C) That the patient is not allergic to penicillins D) That the patient has fasted for at least 8 hours B Feedback: Before the scan, the nurse asks the patient to empty the bladder, because a full bladder interferes with accurate scanning of the pelvic bones. Bowel cleansing and fasting are not indicated for a bone scan and an allergy to penicillins is not a contraindication. 25. A nurse is explaining a patients decreasing bone density in terms of the balance between bone resorption and formation. What dietary nutrients and hormones play a role in the resorption and formation of adult bones? Select all that apply. A) Thyroid hormone B) Growth hormone C) Estrogen D) Vitamin B12 E) Luteinizing hormone A, B, C Feedback: The balance between bone resorption and formation is influenced by the following factors: physical activity; dietary intake of certain nutrients, especially calcium; and several hormones, including calcitriol (i.e., activated vitamin D), parathyroid hormone (PTH), calcitonin, thyroid hormone, cortisol, growth hormone, and the sex hormones estrogen and testosterone. Luteinizing hormone and vitamin B12 do not play a role in bone formation or resorption. 26. Diagnostic tests show that a patients bone density has decreased over the past several years. The patient asks the nurse what factors contribute to bone density decreasing. What would be the nurses best response? A) For many people, lack of nutrition can cause a loss of bone density. B) Progressive loss of bone density is mostly related to your genes. C) Stress is known to have many unhealthy effects, including reduced bone density. D) Bone density decreases with age, but scientists are not exactly sure why this is the case. A Feedback: Nutrition has a profound effect on bone density, especially later life. Genetics are also an important factor, but nutrition has a more pronounced effect. The pathophysiology of bone density is well understood and psychosocial stress has a minimal effect. 27. A bone biopsy has just been completed on a patient with suspected bone metastases. What assessment should the nurse prioritize in the immediate recovery period? A) Assessment for dehiscence at the biopsy site B) Assessment for pain C) Assessment for hematoma formation D) Assessment for infection B Feedback: Bone biopsy can be painful and the nurse should prioritize relevant assessments. Dehiscence is not a possibility, since the incision is not linear. Signs and symptoms of infection would not be evident in the immediate recovery period and hematoma formation is not a common complication. 28. A nurse is taking a health history on a new patient who has been experiencing unexplained paresthesia. What question should guide the nurses assessment of the patients altered sensations? A) How does the strength in the affected extremity compare to the strength in the unaffected extremity? B) Does the color in the affected extremity match the color in the unaffected extremity? C) How does the feeling in the affected extremity compare with the feeling in the unaffected extremity? D) Does the patient have a family history of paresthesia or other forms of altered sensation? C Feedback: Questions that the nurse should ask regarding altered sensations include How does this feeling compare to sensation in the unaffected extremity? Asking questions about strength and color are not relevant and a family history is unlikely. 29. The nurse is assessing a patient for dietary factors that may influence her risk for osteoporosis. The nurse should question the patient about her intake of what nutrients? Select all that apply. A) Calcium B) Simple carbohydrates C) Vitamin D D) Protein E) Soluble fiber A, C Feedback: A patients risk for osteoporosis is strongly influenced by vitamin D and calcium intake. Carbohydrate, protein, and fiber intake do not have direct effect on the development of osteoporosis. 30. The nurse is performing an assessment of a patients musculoskeletal system and is appraising the patients bone integrity. What action should the nurse perform during this phase of assessment? A) Compare parts of the body symmetrically. B) Assess extremities when in motion rather than at rest. C) Percuss as many joints as are accessible. D) Administer analgesia 30 to 60 minutes before assessment. A Feedback: When assessing bone integrity, symmetric parts of the body, such as extremities, are compared. Analgesia should not be necessary and percussion is not a clinically useful assessment technique. Bone integrity is best assessed when the patient is not moving. 31. A nurse is performing a nursing assessment of a patient suspected of having a musculoskeletal disorder. What is the primary focus of the nursing assessment with a patient who has a musculoskeletal disorder? A) Range of motion B) Activities of daily living C) Gait D) Strength B Feedback: The nursing assessment is primarily a functional evaluation, focusing on the patients ability to perform activities of daily living. The nurse also assesses strength, gait, and ROM, but these are assessed to identify their effect on functional status rather than to identify a medical diagnosis. 32. A nurses assessment of a teenage girl reveals that her shoulders are not level and that she has one prominent scapula that is accentuated by bending forward. The nurse should expect to read about what health problem in the patients electronic health record? A) Lordosis B) Kyphosis C) Scoliosis D) Muscular dystrophy C Feedback: Scoliosis is evidenced by an abnormal lateral curve in the spine, shoulders that are not level, an asymmetric waistline, and a prominent scapula, accentuated by bending forward. Lordosis is the curvature in the lower back; kyphosis is an exaggerated curvature of the upper back. This finding is not suggestive of muscular dystrophy. 33. A patient is receiving ongoing nursing care for the treatment of Parkinsons disease. When assessing this patients gait, what finding is most closely associated with this health problem? A) Spastic hemiparesis gait B) Shuffling gait C) Rapid gait D) Steppage gait B Feedback: A variety of neurologic conditions are associated with abnormal gaits, such as a spastic hemiparesis gait (stroke), steppage gait (lower motor neuron disease), and shuffling gait (Parkinsons disease). A rapid gait is not associated with Parkinsons disease. 34. A nurse is caring for a patient who has just had an arthroscopy as an outpatient and is getting ready to go home. The nurse should teach the patient to monitor closely for what postprocedure complication? A) Fever B) Crepitus C) Fasciculations D) Synovial fluid leakage A Feedback: Following arthroscopy, the patient and family are informed of complications to watch for, including fever. Synovial fluid leakage is unlikely and crepitus would not develop as a postprocedure complication. Fasciculations are muscle twitches and do not involve joint integrity or function. 35. A patient is undergoing diagnostic testing for suspected Pagets disease. What assessment finding is most consistent with this diagnosis? A) Altered serum magnesium levels B) Altered serum calcium levels C) Altered serum potassium levels D) Altered serum sodium levels B Feedback: Serum calcium levels are altered in patients with osteomalacia, parathyroid dysfunction, Pagets disease, metastatic bone tumors, or prolonged immobilization. Pagets disease is not directly associated with altered magnesium, potassium, or sodium levels. 36. A nurse is caring for a patient with a diagnosis of cancer that has metastasized. What laboratory value would the nurse expect to be elevated in this patient? A) Bilirubin B) Potassium C) Alkaline phosphatase D) Creatinine C Feedback: Alkaline phosphatase is elevated during early fracture healing and in diseases with increased osteoblastic activity (e.g., metastatic bone tumors). Elevated bilirubin, potassium, and creatinine would not be expected in a patient with metastatic bone tumors. 37. A patient has had a cast placed for the treatment of a humeral fracture. The nurses most recent assessment shows signs and symptoms of compartment syndrome. What is the nurses most appropriate action? A) Arrange for a STAT assessment of the patients serum calcium levels. B) Perform active range of motion exercises. C) Assess the patients joint function symmetrically. D) Contact the primary care provider immediately. D Feedback: This major neurovascular problem is caused by pressure within a muscle compartment that increases to such an extent that microcirculation diminishes, leading to nerve and muscle anoxia and necrosis. Function can be permanently lost if the anoxic situation continues for longer than 6 hours. Therefore, immediate medical care is a priority over further nursing assessment. Assessment of calcium levels is unnecessary. 38. A patient has been experiencing an unexplained decline in knee function and has consequently been scheduled for arthrography. The nurse should teach the patient about what process? A) Injection of a contrast agent into the knee joint prior to ROM exercises B) Aspiration of synovial fluid for serologic testing C) Injection of corticosteroids into the patients knee joint to facilitate ROM D) Replacement of the patients synovial fluid with a synthetic substitute A Feedback: During arthrography, a radiopaque contrast agent or air is injected into the joint cavity to visualize the joint structures such as the ligaments, cartilage, tendons, and joint capsule. The joint is put through its range of motion to distribute the contrast agent while a series of x-rays are obtained. Synovial fluid is not aspirated or replaced and corticosteroids are not administered. 39. The nurses musculoskeletal assessment of a patient reveals involuntary twitching of muscle groups. How would the nurse document this observation in the patients chart? A) Tetany B) Atony C) Clonus D) Fasciculations D Feedback: Fasciculation is involuntary twitching of muscle fiber groups. Clonus is a series of involuntary, rhythmic, muscular contractions and tetany is involuntary muscle contraction, but neither is characterized as twitching. Atony is a loss of muscle strength. 40. A patient has been experiencing progressive increases in knee pain and diagnostic imaging reveals a worsening effusion in the synovial capsule. The nurse should anticipate which of the following? A) Arthrography B) Knee biopsy C) Arthrocentesis D) Electromyography C Feedback: Arthrocentesis (joint aspiration) is carried out to obtain synovial fluid for purposes of examination or to relieve pain due to effusion. Arthrography, biopsy, and electromyography would not remove fluid and relieve pressure. Chapter 40: Musculoskeletal Care Modalities 1. A nurse is caring for a patient who has had a plaster arm cast applied. Immediately postapplication, the nurse should provide what teaching to the patient? A) The cast will feel cool to touch for the first 30 minutes. B) The cast should be wrapped snuggly with a towel until the patient gets home. C) The cast should be supported on a board while drying. D) The cast will only have full strength when dry. D Feedback: A cast requires approximately 24 to 72 hours to dry, and until dry, it does not have full strength. While drying, the cast should not be placed on a hard surface. The cast will exude heat while it dries and should not be wrapped. 2. A patient broke his arm in a sports accident and required the application of a cast. Shortly following application, the patient complained of an inability to straighten his fingers and was subsequently diagnosed with Volkmann contracture. What pathophysiologic process caused this complication? A) Obstructed arterial blood flow to the forearm and hand B) Simultaneous pressure on the ulnar and radial nerves C) Irritation of Merkel cells in the patients skin surfaces D) Uncontrolled muscle spasms in the patients forearm A Feedback: Volkmann contracture occurs when arterial blood flow is restricted to the forearm and hand and results in contractures of the fingers and wrist. It does not result from nerve pressure, skin irritation, or spasms. 3. A patient is admitted to the unit in traction for a fractured proximal femur and requires traction prior to surgery. What is the most appropriate type of traction to apply to a fractured proximal femur? A) Russells traction B) Dunlops traction C) Bucks extension traction D) Cervical head halter C Feedback: Bucks extension is used for fractures of the proximal femur. Russells traction is used for lower leg fractures. Dunlops traction is applied to the upper extremity for supracondylar fractures of the elbow and humerus. Cervical head halters are used to stabilize the neck. 4. A nurse is caring for a patient who is in skeletal traction. To prevent the complication of skin breakdown in a patient with skeletal traction, what action should be included in the plan of care? A) Apply occlusive dressings to the pin sites. B) Encourage the patient to push up with the elbows when repositioning. C) Encourage the patient to perform isometric exercises once a shift. D) Assess the pin insertion site every 8 hours. D Feedback: The pin insertion site should be assessed every 8 hours for inflammation and infection. Loose cover dressings should be applied to pin sites. The patient should be encouraged to use the overhead trapeze to shift weight for repositioning. Isometric exercises should be done 10 times an hour while awake. 5. A nurse is caring for a patient who is postoperative day 1 right hip replacement. How should the nurse position the patient? A) Keep the patients hips in abduction at all times. B) Keep hips flexed at no less than 90 degrees. C) Elevate the head of the bed to high Fowlers. D) Seat the patient in a low chair as soon as possible. A Feedback: The hips should be kept in abduction by an abductor pillow. Hips should not be flexed more than 90 degrees, and the head of bed should not be elevated more than 60 degrees. The patients hips should be higher than the knees; as such, high seat chairs should be used. 6. While assessing a patient who has had knee replacement surgery, the nurse notes that the patient has developed a hematoma at the surgical site. The affected leg has a decreased pedal pulse. What would be the priority nursing diagnosis for this patient? A) Risk for Infection B) Risk for Peripheral Neurovascular Dysfunction C) Unilateral Neglect D) Disturbed Kinesthetic Sensory Perception B Feedback: The hematoma may cause an interruption of tissue perfusion, so the most appropriate nursing diagnosis is Risk of Peripheral Neurovascular Dysfunction. There is also an associated risk for infection because of the hematoma, but impaired neurovascular function is a more acute threat. Unilateral neglect and impaired sensation are lower priorities than neurovascular status. 7. A patient was brought to the emergency department after a fall. The patient is taken to the operating room to receive a right hip prosthesis. In the immediate postoperative period, what health education should the nurse emphasize? A) Make sure you dont bring your knees close together. B) Try to lie as still as possible for the first few days. C) Try to avoid bending your knees until next week. D) Keep your legs higher than your chest whenever you can. A Feedback: After receiving a hip prosthesis, the affected leg should be kept abducted. Mobility should be encouraged within safe limits. There is no need to avoid knee flexion and the patients legs do not need to be higher than the level of the chest. 8. A patient with a fractured femur is in balanced suspension traction. The patient needs to be repositioned toward the head of the bed. During repositioning, what should the nurse do? A) Place slight additional tension on the traction cords. B) Release the weights and replace them immediately after positioning. C) Reposition the bed instead of repositioning the patient. D) Maintain consistent traction tension while repositioning. D Feedback: Traction is used to reduce the fracture and must be maintained at all times, including during repositioning. It would be inappropriate to add tension or release the weights. Moving the bed instead of the patient is not feasible. 9. A patient with a total hip replacement is progressing well and expects to be discharged tomorrow. On returning to bed after ambulating, he complains of a new onset of pain at the surgical site. What is the nurses best action? A) Administer pain medication as ordered. B) Assess the surgical site and the affected extremity. C) Reassure the patient that pain is a direct result of increased activity. D) Assess the patient for signs and symptoms of systemic infection. B Feedback: Worsening pain after a total hip replacement may indicate dislocation of the prosthesis. Assessment of pain should include evaluation of the wound and the affected extremity. Assuming hes anxious about discharge and administering pain medication do not address the cause of the pain. Sudden severe pain is not considered normal after hip replacement. Sudden pain is rarely indicative of a systemic infection. 10. A nurse is caring for a patient who has a leg cast. The nurse observes that the patient uses a pencil to scratch the skin under the edge of the cast. How should the nurse respond to this observation? A) Allow the patient to continue to scratch inside the cast with a pencil but encourage him to be cautious. B) Give the patient a sterile tongue depressor to use for scratching instead of the pencil. C) Encourage the patient to avoid scratching, and obtain an order for an antihistamine if severe itching persists. D) Obtain an order for a sedative, such as lorazepam (Ativan), to prevent the patient from scratching. C Feedback: Scratching should be discouraged because of the risk for skin breakdown or damage to the cast. Most patients can be discouraged from scratching if given a mild antihistamine, such as diphenhydramine, to relieve itching. Benzodiazepines would not be given for this purpose. 11. The nurse is caring for a patient who underwent a total hip replacement yesterday. What should the nurse do to prevent dislocation of the new prosthesis? A) Keep the affected leg in a position of adduction. B) Have the patient reposition himself independently. C) Protect the affected leg from internal rotation. D) Keep the hip flexed by placing pillows under the patients knee. C Feedback: Abduction of the hip helps to prevent dislocation of a new hip joint. Rotation and adduction should be avoided. While the hip may be flexed slightly, it shouldnt exceed 90 degrees and maintenance of flexion isnt necessary. The patient may not be capable of safe independent repositioning at this early stage of recovery. 12. A patient is complaining of pain in her casted leg. The nurse has administered analgesics and elevated the limb. Thirty minutes after administering the analgesics, the patient states the pain is unrelieved. The nurse should identify the warning signs of what complication? A) Subcutaneous emphysema B) Skin breakdown C) Compartment syndrome D) Disuse syndrome C Feedback: Compartment syndrome may manifest as unrelenting, uncontrollable pain. This presentation of pain is not suggestive of disuse syndrome or skin breakdown. Subcutaneous emphysema is not a complication of casting. 13. The nurse educator on an orthopedic trauma unit is reviewing the safe and effective use of traction with some recent nursing graduates. What principle should the educator promote? A) Knots in the rope should not be resting against pulleys. B) Weights should rest against the bed rails. C) The end of the limb in traction should be braced by the footboard of the bed. D) Skeletal traction may be removed for brief periods to facilitate the patients independence. A Feedback: Knots in the rope should not rest against pulleys, because this interferes with traction. Weights are used to apply the vector of force necessary to achieve effective traction and should hang freely at all times. To avoid interrupting traction, the limb in traction should not rest against anything. Skeletal traction is never interrupted. 14. The orthopedic surgeon has prescribed balanced skeletal traction for a patient. What advantage is conferred by balanced traction? A) Balanced traction can be applied at night and removed during the day. B) Balanced traction allows for greater patient movement and independence than other forms of traction. C) Balanced traction is portable and may accompany the patients movements. D) Balanced traction facilitates bone remodeling in as little as 4 days. B Feedback: Often, skeletal traction is balanced traction, which supports the affected extremity, allows for some patient movement, and facilitates patient independence and nursing care while maintaining effective traction. It is not portable, however, and it cannot be removed. Bone remodeling takes longer than 4 days. 15. The nursing care plan for a patient in traction specifies regular assessments for venous thromboembolism (VTE). When assessing a patients lower limbs, what sign or symptom is suggestive of deep vein thrombosis (DVT)? A) Increased warmth of the calf B) Decreased circumference of the calf C) Loss of sensation to the calf D) Pale-appearing calf A Feedback: Signs of DVT include increased warmth, redness, swelling, and calf tenderness. These findings are promptly reported to the physician for definitive evaluation and therapy. Signs and symptoms of a DVT do not include a decreased circumference of the calf, a loss of sensation in the calf, or a pale-appearing calf. 16. A nurse is providing discharge education to a patient who is going home with a cast on his leg. What teaching point should the nurse emphasize in the teaching session? A) Using crutches efficiently B) Exercising joints above and below the cast, as ordered C) Removing the cast correctly at the end of the treatment period D) Reporting signs of impaired circulation D Feedback: Reporting signs of impaired circulation is critical; signs of impaired circulation must be reported to the physician immediately to prevent permanent damage. For this reason, this education is a priority over exercise and crutch use. The patient does not independently remove the cast. 17. A patient with a right tibial fracture is being discharged home after having a cast applied. What instruction should the nurse provide in relationship to the patients cast care? A) Cover the cast with a blanket until the cast dries. B) Keep your right leg elevated above heart level. C) Use a clean object to scratch itches inside the cast. D) A foul smell from the cast is normal after the first few days. B Feedback: The leg should be elevated to promote venous return and prevent edema. The cast shouldnt be covered while drying because this will cause heat buildup and prevent air circulation. No foreign object should be inserted inside the cast because of the risk of cutting the skin and causing an infection. A foul smell from a cast is never normal and may indicate an infection. 18. An elderly patients hip joint is immobilized prior to surgery to correct a femoral head fracture. What is the nurses priority assessment? A) The presence of leg shortening B) The patients complaints of pain C) Signs of neurovascular compromise D) The presence of internal or external rotation C Feedback: Because impaired circulation can cause permanent damage, neurovascular assessment of the affected leg is always a priority assessment. Leg shortening and internal or external rotation are common findings with a fractured hip. Pain, especially on movement, is also common after a hip fracture. A nurse is caring for a patient who has had a total hip replacement. The nurse is reviewing health education prior to discharge. Which of the patients statements would indicate to the nurse that the patient requires further teaching? A) Ill need to keep several pillows between my legs at night. B) I need to remember not to cross my legs. Its such a habit. C) The occupational therapist is showing me how to use a sock puller to help me get dressed. D) I will need my husband to assist me in getting off the low toilet seat at home. D Feedback: To prevent hip dislocation after a total hip replacement, the patient must avoid bending the hips beyond 90 degrees. Assistive devices, such as a raised toilet seat, should be used to prevent severe hip flexion. Using an abduction pillow or placing several pillows between the legs reduces the risk of hip dislocation by preventing adduction and internal rotation of the legs. Likewise, teaching the patient to avoid crossing the legs also reduces the risk of hip dislocation. A sock puller helps a patient get dressed without flexing the hips beyond 90 degrees. 20. A nurse is admitting a patient to the unit who presented with a lower extremity fracture. What signs and symptoms would suggest to the nurse that the patient may have aperoneal nerve injury? A) Numbness and burning of the foot B) Pallor to the dorsal surface of the foot C) Visible cyanosis in the toes D) Inadequate capillary refill to the toes A Feedback: Peroneal nerve injury may result in numbness, tingling, and burning in the feet. Cyanosis, pallor, and decreased capillary refill are signs of inadequate circulation. 21. A patient has suffered a muscle strain and is complaining of pain that she rates at 6 on a 10-point scale. The nurse should recommend what action? A) Taking an opioid analgesic as ordered B) Applying a cold pack to the injured site C) Performing passive ROM exercises D) Applying a heating pad to the affected muscle B Feedback: Most pain can be relieved by elevating the involved part, applying cold packs, and administering analgesics as prescribed. Heat may exacerbate the pain by increasing blood circulation, and ROM exercises would likely be painful. Analgesia is likely necessary, but NSAIDs would be more appropriate than opioids. 22. A patient has had a brace prescribed to facilitate recovery from a knee injury. What are the potential therapeutic benefits of a brace? Select all that apply. A) Preventing additional injury B) Immobilizing prior to surgery C) Providing support D) Controlling movement E) Promoting bone remodeling A, C, D Feedback: Braces (i.e., orthoses) are used to provide support, control movement, and prevent additional injury. They are not used to immobilize body parts or to facilitate bone remodeling. 23. A nurse is assessing the neurovascular status of a patient who has had a leg cast recently applied. The nurse is unable to palpate the patients dorsalis pedis or posterior tibial pulse and the patients foot is pale. What is the nurses most appropriate action? A) Warm the patients foot and determine whether circulation improves. B) Reposition the patient with the affected foot dependent. C) Reassess the patients neurovascular status in 15 minutes. D) Promptly inform the primary care provider. D Feedback: Signs of neurovascular dysfunction warrant immediate medical follow-up. It would be unsafe to delay. Warming the foot or repositioning the patient may be of some benefit, but the care provider should be informed first. 24. A physician writes an order to discontinue skeletal traction on an orthopedic patient. The nurse should anticipate what subsequent intervention? A) Application of a walking boot B) Application of a cast C) Education on how to use crutches D) Passive range of motion exercises B Feedback: After skeletal traction is discontinued, internal fixation, casts, or splints are then used to immobilize and support the healing bone. The use of a walking boot, crutches, or ROM exercises could easily damage delicate, remodeled bone. 25. A patient has just begun been receiving skeletal traction and the nurse is aware that muscles in the patients affected limb are spastic. How does this change in muscle tone affect the patients traction prescription? A) Traction must temporarily be aligned in a slightly different direction. B) Extra weight is needed initially to keep the limb in proper alignment. C) A lighter weight should be initially used. D) Weight will temporarily alternate between heavier and lighter weights. B Feedback: The traction weights applied initially must overcome the shortening spasms of the affected muscles. As the muscles relax, the traction weight is reduced to prevent fracture dislocation and to promote healing. Weights never alternate between heavy and light. 26. A nurse is planning the care of a patient who will require a prolonged course of skeletal traction. When planning this patients care, the nurse should prioritize interventions related to which of the following risk nursing diagnoses? A) Risk for Impaired Skin Integrity B) Risk for Falls C) Risk for Imbalanced Fluid Volume D) Risk for Aspiration A Feedback: Impaired skin integrity is a high-probability risk in patients receiving traction. Falls are not a threat, due to the patients immobility. There are not normally high risks of fluid imbalance or aspiration associated with traction. 27. A nurse is caring for a patient receiving skeletal traction. Due to the patients severe limits on mobility, the nurse has identified a risk for atelectasis or pneumonia. What intervention should the nurse provide in order to prevent these complications? A) Perform chest physiotherapy once per shift and as needed. B) Teach the patient to perform deep breathing and coughing exercises. C) Administer prophylactic antibiotics as ordered. D) Administer nebulized bronchodilators and corticosteroids as ordered. B Feedback: To prevent these complications, the nurse should educate the patient about performing deep-breathing and coughing exercises to aid in fully expanding the lungs and clearing pulmonary secretions. Antibiotics, bronchodilators, and steroids are not used on a preventative basis and chest physiotherapy is unnecessary and implausible for a patient in traction. 28. The nurse has identified the diagnosis of Risk for Impaired Tissue Perfusion Related to Deep Vein Thrombosis in the care of a patient receiving skeletal traction. What nursing intervention best addresses this risk? A) Encourage independence with ADLs whenever possible. B) Monitor the patients nutritional status closely. C) Teach the patient to perform ankle and foot exercises within the limitations of traction. D) Administer clopidogrel (Plavix) as ordered. C Feedback: The nurse educates the patient how to perform ankle and foot exercises within the limits of the traction therapy every 1 to 2 hours when awake to prevent DVT. Nutrition is important, but does not directly prevent DVT. Similarly, independence with ADLs should be promoted, but this does not confer significant prevention of DVT, which often affects the lower limbs. Plavix is not normally used for DVT prophylaxis. 29. A patient is scheduled for a total hip replacement and the surgeon has explained the risks of blood loss associated with orthopedic surgery. The risk of blood loss is the indication for which of the following actions? A) Use of a cardiopulmonary bypass machine B) Postoperative blood salvage C) Prophylactic blood transfusion D) Autologous blood donation D Feedback: Many patients donate their own blood during the weeks preceding their surgery. Autologous blood donations are cost effective and eliminate many of the risks of transfusion therapy. Orthopedic surgery does not necessitate cardiopulmonary bypass and blood is not salvaged postoperatively. Transfusions are not given prophylactically. 30. The nurse is helping to set up Bucks traction on an orthopedic patient. How often should the nurse assess circulation to the affected leg? A) Within 30 minutes, then every 1 to 2 hours B) Within 30 minutes, then every 4 hours C) Within 30 minutes, then every 8 hours D) Within 30 minutes, then every shift A Feedback: After skin traction is applied, the nurse assesses circulation of the foot or hand within 15 to 30 minutes and then every 1 to 2 hours. 31. A nurse is assessing a patient who is receiving traction. The nurses assessment confirms that the patient is able to perform plantar flexion. What conclusion can the nurse draw from this finding? A) The leg that was assessed is free from DVT. B) The patients tibial nerve is functional. C) Circulation to the distal extremity is adequate. D) The patient does not have peripheral neurovascular dysfunction. B Feedback: Plantar flexion demonstrates function of the tibial nerve. It does not demonstrate the absence of DVT and does not allow the nurse to ascertain adequate circulation. The nurse must perform more assessments on more sites in order to determine an absence of peripheral neurovascular dysfunction. 32. A nurse is caring for a patient in skeletal traction. In order to prevent bony fragments from moving against one another, the nurse should caution the patient against which of the following actions? A) Shifting ones weight in bed B) Bearing down while having a bowel movement C) Turning from side to side D) Coughing without splinting C Feedback: To prevent bony fragments from moving against one another, the patient should not turn from side to side; however, the patient may shift position slightly with assistance. Bearing down and coughing do not pose a threat to bone union. 33. A nurse is caring for an older adult patient who is preparing for discharge following recovery from a total hip replacement. Which of the following outcomes must be met prior to discharge? A) Patient is able to perform ADLs independently. B) Patient is able to perform transfers safely. C) Patient is able to weight-bear equally on both legs. D) Patient is able to demonstrate full ROM of the affected hip. B Feedback: The patient must be able to perform transfers and to use mobility aids safely. Each of the other listed goals is unrealistic for the patient who has undergone recent hip replacement. 34. A nurse is caring for a patient who is recovering in the hospital following orthopedic surgery. The nurse is performing frequent assessments for signs and symptoms of infection in the knowledge that the patient faces a high risk of what infectious complication? A) Cellulitis B) Septic arthritis C) Sepsis D) Osteomyelitis D Feedback: Infection is a risk after any surgery, but it is of particular concern for the postoperative orthopedic patient because of the risk of osteomyelitis. Orthopedic patients do not have an exaggerated risk of cellulitis, sepsis, or septic arthritis when compared to other surgical patients. 35. A patient is being prepared for a total hip arthroplasty, and the nurse is providing relevant education. The patient is concerned about being on bed rest for several days after the surgery. The nurse should explain what expectation for activity following hip replacement? A) Actually, patients are only on bed rest for 2 to 3 days before they begin walking with assistance. B) The physical therapist will likely help you get up using a walker the day after your surgery. C) Our goal will actually be to have you walking normally within 5 days of your surgery. D) For the first two weeks after the surgery, you can use a wheelchair to meet your mobility needs. B Feedback: Patients post-THA begin ambulation with the assistance of a walker or crutches within a day after surgery. Wheelchairs are not normally utilized. Baseline levels of mobility are not normally achieved until several weeks after surgery, however. 36. A patient has recently been admitted to the orthopedic unit following total hip arthroplasty. The patient has a closed suction device in place and the nurse has determined that there were 320 mL of output in the first 24 hours. How should the nurse best respond to this assessment finding? A) Inform the primary care provider promptly. B) Document this as an expected assessment finding. C) Limit the patients fluid intake to 2 liters for the next 24 hours. D) Administer a loop diuretic as ordered. B Feedback: Drainage of 200 to 500 mL in the first 24 hours is expected. Consequently, the nurse does not need to inform the physician. Fluid restriction and medication administration are not indicated. 37. A nurse is reviewing a patients activities of daily living prior to discharge from total hip replacement. The nurse should identify what activity as posing a potential risk for hip dislocation? A) Straining during a bowel movement B) Bending down to put on socks C) Lifting items above shoulder level D) Transferring from a sitting to standing position B Feedback: Bending to put on socks or shoes can cause hip dislocation. None of the other listed actions poses a serious threat to the integrity of the new hip. 38. A 91-year-old patient is slated for orthopedic surgery and the nurse is integrated gerontologic considerations into the patients plan of care. What intervention is most justified in the care of this patient? A) Administration of prophylactic antibiotics B) Total parenteral nutrition (TPN) C) Use of a pressure-relieving mattress D) Use of a Foley catheter until discharge C Feedback: Older adults have a heightened risk of skin breakdown; use of a pressure-reducing mattress addresses this risk. Older adults do not necessarily need TPN and the Foley catheter should be discontinued as soon as possible to prevent urinary tract infections. Prophylactic antibiotics are not a standard infection prevention measure. 39. A nurse is emptying an orthopedic surgery patients closed suction drainage at the end of a shift. The nurse notes that the volume is within expected parameters but that the drainage has a foul odor. What is the nurses best action? A) Aspirate a small amount of drainage for culturing. B) Advance the drain 1 to 1.5 cm. C) Irrigate the drain with normal saline. D) Inform the surgeon of this finding. D Feedback: The nurse should promptly notify the surgeon of excessive or foul-smelling drainage. It would be inappropriate to advance the drain, irrigate the drain, or aspirate more drainage. 40. A nurse is planning the care of a patient who has undergone orthopedic surgery. What main goal should guide the nurses choice of interventions? A) Improving the patients level of function B) Helping the patient come to terms with limitations C) Administering medications safely D) Improving the patients adherence to treatment A Feedback: Improving function is the overarching goal after orthopedic surgery. Some patients may need to come to terms with limitations, but this is not true of every patient. Safe medication administration is imperative, but this is not a goal that guides other aspects of care. Similarly, adherence to treatment is important, but this is motivated by the need to improve functional status. [Show More]

Last updated: 1 year ago

Preview 1 out of 1386 pages

Add to cart

Instant download

We Accept:

We Accept
document-preview

Buy this document to get the full access instantly

Instant Download Access after purchase

Add to cart

Instant download

We Accept:

We Accept

Reviews( 0 )

$22.00

Add to cart

We Accept:

We Accept

Instant download

Can't find what you want? Try our AI powered Search

OR

REQUEST DOCUMENT
217
1

Document information


Connected school, study & course


About the document


Uploaded On

Apr 22, 2020

Number of pages

1386

Written in

Seller


seller-icon
Kirsch

Member since 4 years

905 Documents Sold


Additional information

This document has been written for:

Uploaded

Apr 22, 2020

Downloads

 1

Views

 217

Document Keyword Tags


$22.00
What is Browsegrades

In Browsegrades, a student can earn by offering help to other student. Students can help other students with materials by upploading their notes and earn money.

We are here to help

We're available through e-mail, Twitter, Facebook, and live chat.
 FAQ
 Questions? Leave a message!

Follow us on
 Twitter

Copyright © Browsegrades · High quality services·